Cardio

अब Quizwiz के साथ अपने होमवर्क और परीक्षाओं को एस करें!

Patients with peripheral arterial disease will have all of the following findings EXCEPT: A. Increased ankle to brachial index (>1) B. Blanching of the legs upon elevation C. Dependent rubor D. Ischemic ulcers E. Pain in calf with ambulation that is relieved with rest

The correct answer is A. Decreased, not increased ankle-brachial index (<1.0) occurs in the setting of peripheral vascular disease due to decreased pulses in the lower extremity compared to the upper extremity.

A 28-year-old man presents to his primary care provider for his yearly physical examination. He currently smokes approximately one pack of cigarettes a day and is interested in quitting. He has no significant past medical history and has no allergies. He is interested in smoking cessation classes but would like additional help. Which of the following medications would be the most appropriate agent for treating nicotine dependence in this patient? A. Bupropion (Wellbutrin, Zyban) B. Buspirone (BuSpar) C. Clonazepam (Klonopin) D. Fluoxetine (Prozac) E. Nefazodone (Serzone)

The correct answer is A. Bupropion was approved by the U.S. Food and Drug Administration in 1996 as a treatment for nicotine dependence. The specific mechanism of action is unclear but is thought to relate to reducing craving for nicotine as well as reducing withdrawal symptoms after cessation. When used as a smoking cessation therapy, the medication is known as Zyban. This agent is an antidepressant that inhibits neuronal uptake of norepinephrine and dopamine.

Which of the following is the preferred treatment for patients with chronic heart failure with preserved ejection fraction? A. Diuretics B. Inotropic agents C. Vasodilators D. ACE inhibitors plus angiotensin-receptor blockers (ARBs) E. Calcium-channel blockers

The correct answer is A. Diuretics and salt restriction are the treatments for chronic heart failure with preserved ejection fraction. These patients do not need increased inotropic activity; rather, they will benefit from having less intravascular volume to move.

Patients with atrial fibrillation may develop their atrial fibrillation as a result of heart structural abnormalities. Abnormalities with which of the following heart structures is most likely to be associated with atrial fibrillation? A. Mitral valve B. Tricuspid valve C. Aortic valve D. Pulmonic valve E. Ventricular septum

The correct answer is A. Mitral valve stenosis is associated with atrial fibrillation as a result of causing pressure overload in the left atrium. This increased pressure in the left atrium causes a big, floppy left atrium, which may give rise to multiple areas of irritability leading to atrial fibrillation.

In addition to substernal chest pain lasting longer than 30 minutes, which of the following signs or symptoms is most consistent with myocardial infarction? A. Diaphoresis B. Stabbing pain C. Shortness of breath D. Palpitations E. Pain radiating into neck

The correct answer is A. Diaphoresis and substernal chest pain longer than 30 minutes are the physical examination signs that are most consistent with MI.

A patient with long-standing dilated cardiomyopathy is seen in the office. The patient has progressed to develop class IV heart failure and has an ejection fraction (EF) of 30%. The patient has also developed a left bundle branch block (LBBB) with a QRS duration of more than 120 milliseconds. Which of the following specific treatments/interventions is specifically indicated in this setting? A. Insertion of biventricular pacemaker (resynchronization therapy) B. Loop diuretic C. Potassium-sparing diuretic D. ACE I for vasodilation E. Hydralazine and isosorbide dinitrate for vasodilation

The correct answer is A. A resynchronization pacemaker is indicated for patients with dilated cardiomyopathy and decreased EF or QRS prolongation. Patients with LBBB and advanced heart failure have improvement of their clinical course when the normal cardiac conduction process (left ventricle firing prior to the right ventricle) is reinstituted.

A 63-year-old woman comes to the emergency department complaining of severe mid-abdominal pain. The patient reports that the pain has increased in intensity over the past few days. There has been no associated nausea or vomiting, no change in bowel habits, and no relief afforded by position changes. The woman is postmenopausal and does not take hormone replacement therapy. She has a 30-year history of hypertension and has been noncompliant with her calcium channel blocker and thiazide diuretic therapy. On examination, her abdomen is obese, but there is a suggestion of a nontender, pulsatile mass in the epigastric region. The remainder of the physical examination is normal. Imaging studies are done, and immediate surgical evaluation is indicated. Which of the following results were most likely obtained from the imaging studies? A. An abdominal aortic segment spanning 7 cm in diameter B. A liver span of more than 9 cm C. Multiple cysts within the liver D. Posterior herniation of a lumbar intervertebral disk E. Stones in the renal pelvis

The correct answer is A. Abdominal aortic aneurysms larger than 5 cm have greater than a 30% chance of rupturing in 3 years. As most ruptures result in death, and as the mortality of surgical repair has fallen sharply, surgical repair of the aneurysm is indicated. Aneurysms that are not repaired expand on average at about 0.4 cm per year. Repair of these aneurysms can be accomplished via either traditional open approaches or new endovascular approaches. With this patient having pain and a pulsatile abdominal mass, there is a concern about this aneurysm dissecting or rupturing. The larger the aneurysm, the higher the potential for rupturing.

A 73-year-old man comes to the emergency department complaining of abdominal pain. He describes a dull, aching, constant pain in his mid-umbilical region. The pain has persisted over the past few days with increasing intensity, and it is not relieved by changes in position or eating. The patient has a past medical history significant for hypertension and coronary artery disease. He had a myocardial infarction 3 years ago. The man has moderate peripheral vascular disease with a prior femoral-popliteal bypass graft on the left. On physical examination, his blood pressure is 180/100 mm Hg and his pulse is 86/min. He has a loud S4 on chest examination. Abdominal examination shows a pulsatile mass in his abdomen and venous stasis changes bilaterally on his lower extremities. Which of the following is the most appropriate diagnostic test at this time? A. Abdominal ultrasound (U/S) B. CT scan of the abdomen C. CT scan of the spine D. Lumbosacral (L/S) spine films E. Spinal MRI

The correct answer is A. Abdominal ultrasound (U/S) is the most cost-effective screening test for a suspected abdominal aortic aneurysm (AAA). Ultrasound of the abdomen is considered to be the definitive test to perform when AAA is suspected, as it has a sensitivity and specificity of almost 100%.

A patient with long-standing mitral valve prolapse (MVP) is seen in the office. She is concerned about potential complications from this condition. Which of the following complications from this condition is most concerning? A. Progressive mitral regurgitation B. Progressive mitral stenosis C. Pulmonary hypertension D. Systemic embolization E. Chronic diastolic heart failure

The correct answer is A. The most concerning complication of mitral valve prolapse is progressive mitral valve regurgitation. Typically, MVP does not lead to complications, but, if one occurs, damage to the chordae tendineae of the mitral valve can lead to mitral valve regurgitation.

A man is an unrestrained front-seat passenger in a car when it crashes; he sustains closed comminuted fractures of both femoral shafts. Shortly after admission, he develops a blood pressure of 80/50 mm Hg, pulse 110/min, and venous pressure of 0. He becomes pale, cold, and clammy. The rest of the physical examination and x-rays of the chest and pelvis are unremarkable. Sonogram of the abdomen performed in the emergency department is likewise negative. Which of the following most likely explains the patient's low blood pressure? A. Blood loss at the fracture sites B. Fat embolism C. Neurogenic shock from pain D. Unrecognized intracranial bleeding E. Unrecognized pericardial tamponade

The correct answer is A. After extensive trauma to certain areas of the body, enough blood may accumulate to send the patient into hypovolemic shock. One of these areas is the femur (as seen in the present patient); the others are the thorax, abdomen, and pelvis. These patients should be treated with massive resuscitation and transfusion to overcome their hypovolemia until reduction and fixation can be performed.

A 65-year-old patient has experienced several transient ischemic attacks over the past few months. Because his general health is poor, he is not considered an appropriate candidate for carotid endarterectomy. Examination reveals a regular rate on heart examination with a blood pressure of 130/86 mm Hg. The decision is made to treat him medically. Which of the following agents would be most appropriate for his therapy? A. Aspirin B. Coumadin (Warfarin) C. Dipyridamole (Persantine) D. Heparin E. Clopidogrel (Plavix) plus aspirin

The correct answer is A. Although treatment of transient ischemic attacks (actually prophylaxis against stroke) remains a controversial area, there are guidelines regarding this use. Daily aspirin therapy has been shown in prospective, randomized studies to reduce the incidence of stroke and death in patients who have transient ischemic attacks or carotid disease. If alternative therapy is needed, either because the patient cannot tolerate aspirin or because aspirin therapy has failed, the antiplatelet agent clopidogrel (Plavix) can be used.

A patient in the office complains of chest pain. A cardiac monitor is placed, and the patient is found to develop ventricular fibrillation. The patient is treated according to ACLS protocol with shock/CPR sequence followed by medication treatment with epinephrine. Which of the following medications is indicated for use next in this sequence? A. Amiodarone B. Lidocaine C. Procainamide D. Sotalol (Betapace) E. Sodium bicarbonate

The correct answer is A. Amiodarone is recommended as the first antiarrhythmic in VF protocol after epinephrine is given twice. In the setting of cardiac arrest with ventricular fibrillation or pulseless ventricular tachycardia, the patient is defibrillated twice and then is given epinephrine followed by amiodarone.

A 57-year-old woman comes to the emergency department complaining of a painful left foot. The patient describes the acute onset of foot tingling and numbness 3 days ago. Since that time the symptoms have progressed to constant pain and inability to bear weight on that foot. The patient's other medical history is significant for atrial fibrillation secondary to amyloidosis. She has been in atrial fibrillation intermittently for the past 3 years and has had attempted chemical and electrical cardioversion, but without long-term success. She currently takes no medications. On examination, she has an irregularly irregular pulse at 94/min. An electrocardiogram shows rhythm compatible with atrial fibrillation. Which of the following additional findings would most likely support the suspected diagnosis? A. Absent dorsalis pedis pulse on the left foot B. Lactate dehydrogenase level of 500 U/L C. Loss of light touch sensation in her affected limb D. Prothrombin time of 16 seconds (normal 11-13 seconds) E. Regular heart rhythm

The correct answer is A. An embolization as a result of atrial fibrillation that would cause the symptoms described by the patient would most likely lead to an absence of distal pulses in the affected leg, unless the embolism underwent autolysis.

A 64-year-old man comes to the office for a routine checkup. He has been taking metformin (Glucophage) for his diabetes. The patient's vital signs show an elevated blood pressure, which is confirmed by his daily blood-pressure measurements at home. Urine studies show evidence of mild proteinuria. The health care provider decides to add a new medication to his regimen to treat his hypertension. What is the best class of medication to add at this time? A. ACE inhibitors B. Beta-blockers C. Calcium channel blockers D. Diuretics E. Alpha-blockers

The correct answer is A. Angiotensin-converting enzyme (ACE) inhibitors are the drug class of choice for treating hypertension in patients who have diabetic nephropathy, because they improve renal function and delay end-stage renal disease. ACE inhibitors block the angiotensin-converting enzyme in the renin-angiotensin system, causing vasodilation and increasing blood flow to the kidneys.

A 20-year-old female is seen in the office with complaints of heart palpitations that occur without provocation. The patient notes that she is light-headed when these palpitations occur, but she has not had any true syncope. The patient has an ejection click heard in the right second intercostal space. If a patient requires treatment for this condition, which category of medications is prescribed? A. Beta-blockers B. Calcium-channel blockers C. ACE inhibitors D. Anticoagulants E. Direct aldosterone blockers

The correct answer is A. Beta-blockers are first-line therapy for symptomatic mitral valve prolapse. Beta-blockers are negative inotropic and negative chronotropic agents, which provides additional time for ventricular filling. Since beta-blockers block sympathetic overdrive, they are effective agents for symptomatic patients who have increased adrenergic tone with palpitations, tachycardia, and exaggerated heart rate response to exercise.

Patients with Brugada syndrome will have both right bundle branch block plus which of the following other EKG findings? A. ST segment elevation in V1 and V2 B. ST depression in V1 and V2 C. Shortened PR interval D. Shortened QT interval E. Prolonged QT interval

The correct answer is A. Brugada syndrome has right-axis deviation plus ST-segment elevation in V1 and V2. Patients with this condition are at risk for sudden cardiac death.

Acute coronary syndrome includes three distinct entities: unstable angina, STEMI, and non-STEMI (NSTEMI). Which of the following is the primary means of differentiating a patient with unstable angina from a patient with NSTEMI? A. Cardiac biomarkers B. EKG findings C. Site of myocardial damage D. Presenting signs or symptoms E. Response to nitrates

The correct answer is A. Cardiac biomarkers differentiate between unstable angina and NSTEMI, as both have similar EKG findings, but only NSTEMI will have positive cardiac biomarkers.

Management of heart failure depends upon whether the underlying cause has preserved or reduced ejection fraction of blood from the left ventricle. Which of the following medications is primarily used for patients whose heart failure is due to diastolic rather than systolic dysfunction? A. Beta blockers B. ACE inhibitors C. ARBs D. Digoxin E. Spironolactone (Aldactone)

The correct answer is A. Diastolic CHF may be treated with beta blockers and diuretics. Beta blockers decrease the force of contraction, which lowers myocardial oxygen demand and improves mortality. Diuretics lower intravascular and intracardiac volume, which can help with symptoms associated with heart failure with preserved ejection fraction.

Which of the following best describes why carotid sinus massage will result in the slowing of PSVT along with return of the patient to normal sinus rhythm? A. PSVT is caused by a reentry circuit that involves the AV node, and vagal stimulation from this maneuver will stimulate the vagus nerve to break the reentry cycle. B. Carotid massage will cause hypoventilation, which will stimulate the vagus nerve slowing the heart. C. Carotid massage will interfere with venous return of the heart, causing less blood back into the right atria, sufficiently slowing the heart and breaking the reentry circuit. D. PSVT is one arrhythmia that will not respond to carotid massage, so there is no effect in breaking the reentry circuit that occurs with this condition. E. Carotid sinus massage inhibits the activity of the vagus nerve, which permits the PSVT cycle to be broken as a result of this maneuver.

The correct answer is A. Carotid massage of the baroreceptors will cause vagus nerve stimulation of the heart to increase, resulting in an increased parasympathetic action on the heart. This stimulation will slow the heart's conduction through the AV node. When the speed of the impulse is slowed through the anterograde pathway, it will break the reentry cycle of PSVT by slowing the anterograde pathway to a speed that approaches the speed of the retrograde pathway, thus terminating the essential requirement that the reentry dysrhythmia needs to work.

A 59-year-old woman comes to the health care provider because of recurrent episodes of lightheadedness upon getting up in the morning and occasionally upon standing up from a chair. She reports that on two occasions she has "passed out" soon after getting up from bed. Her temperature is 37°C (98.6°F), blood pressure is 130/80 mm Hg, pulse is 70/min and regular, and respirations are 14/min. She takes a beta-blocker and a thiazide diuretic for moderate hypertension diagnosed 6 months ago. She had rheumatic fever as a child. A thorough physical examination, including chest auscultation, is unremarkable. Which of the following is the most likely cause of this patient's symptoms? A. Antihypertensive treatment B. Aortic stenosis C. Drop attacks D. Reflex syncope E. Transient ischemic attacks F. Vasovagal syncope

The correct answer is A. Clinical history is crucial in the diagnostic approach to syncope. Furthermore, measurement of orthostatic blood pressures and pulses should be performed first with the patient in a supine position, and then checked again sitting and standing. This patient's episodes of lightheadedness and syncope can be best explained as an effect of antihypertensive therapy. Orthostatic hypotension is one of the most frequent side effects of antihypertensive drugs and should always be considered in the differential diagnosis of syncope of unexplained origin.

A patient is seen in the office, and examination of the jugular pulse reveals a rise in the jugular venous pressure when inhaling. Cannon A waves are not seen in this patient. Which of the following conditions is most likely to cause this finding? A. Constrictive pericarditis B. Third-degree heart block C. Severe asthma D. Advanced right bundle branch block E. Severe hypotension

The correct answer is A. Constrictive pericarditis can cause Kussmaul sign, in which an increase in jugular venous pressure occurs with inhalation, rather than the normal fall in jugular venous pressure. Inhalation should cause a fall in jugular venous pressure when negative intrathoracic pressure draws blood into the thoracic cavity.

Which of the following physical examination findings is the most prominent physical finding in a patient who has constrictive pericarditis? A. Prominent JVD B. Rales at the posterior inferior bases C. Stridor D. S3 gallop E. Fixed split S2

The correct answer is A. Constrictive pericarditis occurs as a result of scarring in the pericardium with the loss of the normal elasticity in the pericardium. Jugular venous distention (JVD) is the most prominent sign for constrictive pericarditis, and this is seen in more than 90% of patients with this condition.

A health care provider is managing a patient with hypertrophic cardiomyopathy (HCM). In addition to avoiding vigorous exercise, which of the following medications would be contraindicated for use in a patient with HCM? A. Digoxin (Lanoxin) B. Metoprolol (Lopressor) C. Disopyramide (Norpace) D. Diltiazem (Cardizem) E. Verapamil (Isoptin)

The correct answer is A. Digoxin should be avoided in patients with HCM because it increases the force of contraction, which will worsen obstruction. Patients with HCM should have volume expansion, and they should also be given agents that have negative inotropic activity in order to lessen the outflow obstruction. While digoxin is a negative chronotropic agent, which would allow for increased time for ventricular activity, its positive inotropic activity would worsen the obstruction in the setting of HCM.

Health care providers need to have working knowledge with regard to patients with valvular heart disease. Which of the following is the most common (and earliest) sign or symptom that is seen in a patient with mitral stenosis? A. Dyspnea B. Palpitations C. Chest pain D. Syncope E. Cyanosis

The correct answer is A. Dyspnea is the most common symptom in a patient who has long-standing mitral stenosis. Another complication that occurs in the setting of mitral stenosis is pulmonary edema. As the blood in the left atrium cannot get through the narrow opening of the mitral valve, there is a back-up of blood into the pulmonary system, which results in the patient developing dyspnea. The initial dyspnea will be with exertion, but, with ongoing signs and symptoms, the patient will go on to develop progressive dyspnea to the point of dyspnea with rest. Typically, patients will have surgical intervention when symptoms progress.

A patient is seen with complaints of shortness of breath and chest discomfort. He feels better leaning forward and taking more shallow breaths. Examination reveals distant heart tones. Which of the following noninvasive imaging modalities is the preferred modality to perform in the evaluation of pericardial effusion and its hemodynamic consequences? A. Echocardiogram B. SPECT nuclear image C. PET scan of the thorax D. CT of the thorax E. MRI of the thorax

The correct answer is A. Echocardiogram is the best initial imaging study for detecting pericardial effusion. If the health care provider wishes to evaluate a patient for pericardial thickness, CT or MRI is preferred.

A patient is seen with multiple complaints. The patient is having increased shortness of breath along with ongoing progressive fatigue. He had a history of a holosystolic heart murmur as a child that was never followed up on because of lack of access to health care. Examination of the patient reveals central cyanosis along with digital clubbing that is seen in all of his nails. Which of the following descriptions is the most likely scenario explaining this patient's findings? A. Untreated ventricular septal defect increases pulmonary pressure causing eventual reversal of left-to-right shunting of blood leading to right-to-left shunting of blood B. Gradual relocation of the tricuspid valve to the right ventricle C. Machinery-like murmur that persists into the adult population D. Adult with congenital heart disease from transposition of the vessels E. Uncorrected tetralogy of Fallot with resultant right ventricular hypertrophy

The correct answer is A. Eisenmenger syndrome has uncorrected ventricular septal defect with eventual right-to-left shunt due to increased pulmonary pressures over time. When this right-to-left shunting occurs, deoxygenated blood is sent out into the periphery, and the condition is considered to be irreversible, leading to eventual death.

Which of the following medications used in the setting of non-STEMI (NSTEMI) and unstable angina acts as a factor Xa inhibitor? A. Fondaparinux (Arixtra) B. Abciximab (Reopro) C. Prasugrel (Effient) D. Ticagrelor (Brilinta) E. Bivalirudin (Angiomax)

The correct answer is A. Fondaparinux is a factor Xa inhibitor through its ability to selectively bind to antithrombin III, which potentiates factor Xa neutralization, which, in turn, inhibits thrombin formation. This medication is a synthetic selective factor Xa inhibitor and can be used in the setting of NSTEMI and unstable angina in order to stop new clot formation.

A patient is seen for a follow-up appointment in the office in order to obtain results of a continuous loop recorder. Analysis of this recorder reveals that the patient had two runs of nonsustained ventricular tachycardia that occurred while he was sleeping. The loop recorder was in place for a one-month period. The patient was asymptomatic during these episodes. The patient has had a previous myocardial infarction. Which of the following is the recommended treatment for this patient? A. ICD placement (implantable cardiac defibrillator) B. Amiodarone C. Procainamide D. Permanent pacemaker with overdriving capacity E. No further treatment

The correct answer is A. Implantable cardio-defibrillator is indicated for patients with nonsustained VT in the setting of underlying CAD. Patients with underlying cardiac disease who have ventricular tachycardia (sustained or not) are at increased risk for sudden cardiac death, which serves as an indication for placement of implantable cardiac defibrillator.

A patient is seen in the emergency department after complaining of symptoms related to bradycardia caused by atrioventricular block related to an infectious process. Which of the following infections is known to cause bradycardia? A. Lyme disease B. Gonorrhea C. Streptococcal pneumonia D. Aspergillosis E. Pertussis

The correct answer is A. Infection-related bradycardia may be due to Lyme disease, endocarditis, Chagas disease, syphilis, tuberculosis, diphtheria, or toxoplasmosis.

Which of the following locations of acute STEMI is most likely to result in the development of bradyarrhythmias and AV heart block? A. Inferior wall B. Anterior wall C. Septal wall D. Lateral wall E. None

The correct answer is A. Inferior wall myocardial infarction (MI) can lead to slowing of the heart due to increased vagal tone or discrete atrioventricular (AV) node infarction. The right coronary artery is most likely to provide circulation to the inferior and posterior wall. When the patient is right-heart dominant, the right coronary artery supplies blood to the AV node, and infarction of this vessel will result in the impairment of blood to the pacemaker of the heart, resulting in bradycardia or heart block.

A patient has a STEMI that is complicated by volume overload status. In addition to the use of loop diuretics, which of the following agents is recommended as additional first-line therapy for this condition? A. Nitroglycerin B. Dobutamine C. Nitroprusside D. Dopamine E. Norepinephrine

The correct answer is A. Nitroglycerin acts as a vasodilator, dilating the coronary artery and increasing pulmonary venous capacitance, which will help unload the vulnerable heart following acute STEMI. Nitroglycerin is also helpful for use in the setting of pulmonary edema, as the increased venous capacitance pulls blood out of the central circulation, which will treat volume overload in the setting of acute STEMI.

A patient with long-standing coronary artery disease, diabetes, smoking history, and chronic obstructive pulmonary disease presents to the office with complaints related to peripheral vascular disease. The patient is complaining of claudication affecting both calves and has buttock claudication, impotence, and weakened femoral pulses. Which of the following is the most likely diagnosis? A. Leriche syndrome B. Acute arterial occlusion C. Cholesterol embolization syndrome D. Deep vein thrombosis E. Chronic venous insufficiency

The correct answer is A. Leriche syndrome causes occlusion just above the bifurcation of the distal aorta, with the patient developing bilateral claudication, impotence, and weakened femoral pulses.

You are called to assess a patient in the ICU with the diagnosis of "shock." If the patient has neurogenic shock, which of the following signs or symptoms would be consistent with this particular type of shock? A. Low blood pressure but bradycardic B. Petechial rash C. Decreased urinary output to less than 20 cc per hour D. Distended neck veins E. Breath sounds only on one side of the chest

The correct answer is A. Low blood pressure should result in compensatory tachycardia. If bradycardia occurs in the setting of hypotension, neurogenic shock occurs, as there is a disconnect between the body's compensatory mechanisms to appropriately respond to the hypotension. Neurogenic shock involves a disconnect between the brain and spinal cord, which results in an inappropriate bradycardia occurring in the setting of hypotension.

Patients with chronic heart failure are treated with a variety of agents. To what class of medication does milrinone (Primacor) belong? A. Inotrope B. Loop diuretic C. Aldosterone antagonist D. Combined alpha- and beta-blocker E. Vasodilator

The correct answer is A. Milrinone is used in chronic heart failure with reduced ejection fraction and acts as inotrope without causing additional sympathetic tone. This agent inhibits cyclic AMP phosphodiesterase, which makes it acts like an inotrope and as a vasopressor.

Which of the following cardiac abnormalities is associated with an opening snap? A. Mitral stenosis B. Aortic stenosis C. Pulmonic stenosis D. Tricuspid stenosis E. Mitral valve prolapse

The correct answer is A. Mitral stenosis can cause opening snap, and it is the only valvular abnormality that can do so. An opening snap is due to the abrupt halt in leaflet motion in early diastole, after rapid initial rapid opening, due to fusion at the leaflet tips. It is best heard at the apex and lower left sternal border.

A patient is seen with acute ST-segment elevation myocardial infarction. The patient's pain does not respond to nitroglycerin treatment and morphine is being considered for additional therapy. In addition to causing pain relief as a result of its narcotic properties, which of the following mechanisms of action occurs with the use of morphine? A. Venodilation B. Decreased inotropic activity of the heart C. Decreased heart rate D. Coronary artery vasodilation E. Prevents the progression of thrombus

The correct answer is A. Morphine causes pain relief along with venodilation, which decreases myocardial oxygen demand and can result in relief of chest pain.

A patient is seen in the office with concerns about whether heart failure exists. Clinicians should learn to rely on their history and physical examination signs and not just laboratory findings in making the diagnosis of heart failure. Physical signs associated with chronic heart failure include all of the following EXCEPT: A. Wide pulse pressure B. Laterally displaced point of maximum impulse C. Positive hepatojugular reflex D. S3 E. Pulmonary rales that do not clear with coughing

The correct answer is A. Narrow (not wide) pulse pressure is associated with CHF. Pulse pressure involves the difference between systolic and diastolic blood pressure. Narrow pulse pressure means that the systolic blood pressure is not excessively higher than the diastolic blood pressure. When there is a decrease in left ventricular pulse pressure, the systolic blood pressure decreases. When there is poor perfusion to the kidneys with heart failure, there is increased intravascular pressure, which causes the diastolic pressure to increase. The combination of these factors causes the systolic blood pressure to fall and a relative increase in the diastolic blood pressure, which causes a narrow pulse pressure.

Which of the following patients should be evaluated for secondary rather than primary (essential) hypertension as the cause of hypertension? A. Patients who develop hypertension after age 55. B. Patients who have an increased intake of alcohol. C. Patients taking NSAIDs on a long-term basis. D. Patients who develop hypertension under age 40. E. Patients who develop hypertension after becoming obese.

The correct answer is A. Patients who develop hypertension prior to age 30 or after age 55 are more likely to have secondary rather than essential hypertension. The reason that this evaluation should occur is that eliminating the secondary cause of hypertension will result in curing hypertension.

A patient is seen in the office with chest pain. An EKG is performed immediately that shows inferior wall EKG changes, and the patient is noted to have hypotension, elevated jugular venous distension (JVD), and clear lungs. Damage to which of the following locations is most likely responsible for this presentation? A. Right ventricle B. Septal infarct C. Anterior wall D. Lateral wall E. Posterior wall

The correct answer is A. Right ventricular infarct patients will have clear lungs, JVD, inferior wall changes, and hypotension. It is important to identify this location of infarction because these patients are treated differently than those with infarctions located in other parts of the heart. Right ventricular infarctions should be suspected in the setting of inferior wall myocardial infarctions, as the right coronary artery supplies blood to both areas of the heart.

A patient is evaluated for potential angina and coronary artery disease. The risk for coronary disease is intermediate, and the patient undergoes an exercise treadmill test. Which of the following EKG findings is most commonly associated with exercise-induced ischemia? A. ST segment depression B. New-onset left bundle branch block C. ST segment elevation D. T wave tenting E. Early repolarization pattern

The correct answer is A. ST segment depression classically occurs with exercise-induced ischemia. When ST depression is seen in the setting of a patient having chest pain, the primary differential diagnosis is angina pectoris caused by coronary ischemia and non-STEMI myocardial infarction, which is diagnosed with this EKG finding along with positive coronary enzymes.

The term "shock" encompasses many distinct varieties. Which of the following is a main differentiating feature between hypovolemic shock and septic shock? A. Septic shock will have warm skin, and hypovolemic shock will have cool and clammy skin. B. Septic shock must have a rash, which is absent in hypovolemic shock. C. Only hypovolemic shock will have an increase in blood pressure with crystalloid administration. D. Only hypovolemic shock will have tachycardia. E. Only septic shock will cause a change in mental status.

The correct answer is A. Septic shock produces peripheral vasodilation with warm skin, while hypovolemic shock produces vasoconstriction with resultant cool and clammy skin.

A patient presenting to the office with a severe headache and markedly elevated blood pressure should have lowering of the blood pressure along with which of the following? A. CT of the brain B. MRI of the brain C. Urinalysis D. Echocardiogram E. Serum electrolytes

The correct answer is A. Severe headache and markedly elevated BP should have lowering of the BP along with CT of the head to look for subarachnoid hemorrhage.

Which of the following medication classes used in the management of chronic stable angina is associated with an increase in coronary mortality? A. Short-acting calcium-channel blockers B. Long-acting calcium-channel blockers C. Beta-1-specific beta-blockers D. Non-cardiac-specific beta-blockers E. Ranolazine (Ranexa)

The correct answer is A. Short-acting calcium-channel blockers used in the setting of chronic angina are associated with increased cardiac mortality and therefore should be avoided. As a rule, short-acting calcium-channel blockers are not typically used in clinical practice.

Patients that develop infection within the heart may develop injury to the heart valves and endocardium. This infection can occur either acutely or subacutely. Which of the following pathogens is most likely to be responsible for acute rather than subacute bacterial endocarditis? A. Staphylococcus aureus B. Streptococcus viridans C. Enterococcus species D. Streptococcus bovis E. Staphylococcus epidermidis

The correct answer is A. Staphylococcus aureus is the cause of acute rather than subacute endocarditis due to its virulence. S. aureus causes a more severe infection with bacteremia than other bacteria.

Several risk factors are known to put a patient at risk for the development of endocarditis affecting the heart. Which of the following pathogens is the most common cause of endocarditis in IV drug users? A. Staphylococcus aureus B. Streptococcus viridans C. Enterococcus species D. Streptococcus bovis E. Staphylococcus epidermidis

The correct answer is A. Staphylococcus aureus is the most common cause of endocarditis in drug users, as this pathogen is highly virulent and can be introduced by the injection or the material that is being injected. When endocarditis occurs in this setting, the tricuspid valve is most commonly affected.

A patient is seen with concerns regarding his lower extremity. The patient is worried about possibly having a deep vein thrombosis (DVT), as this problem tends to run in his family. Which of the following physical examination findings is the primary differentiating feature between superficial venous thrombosis, cellulitis, and lymphangitis? A. There are palpable indurated veins with superficial thrombophlebitis. B. There is no pain with cellulitis. C. Only superficial venous thrombosis has an identifiable underlying cause. D. There is only warmth with cellulitis. E. Cellulitis has to have a visible streak located proximal to the site of cellulitis

The correct answer is A. Superficial thrombophlebitis has a palpable indurated venous cord; lymphangitis has a proximal visible streak, which defines this condition as lymphangitis; and cellulitis has redness, warmth, and diffuse tenderness.

You are seeing a patient in the ICU as a medical consultant. Which of the following is the most common cause for the development of acute cardiogenic shock? A. Acute myocardial infarction B. Cardiac tamponade C. Tension pneumothorax D. Cardia arrhythmias E. Massive pulmonary embolism

The correct answer is A. The most common underlying cause of acute cardiogenic shock is myocardial infarction (MI). When MI occurs, impaired blood flow to the heart muscle in turn impairs pump function, resulting in acute cardiogenic shock.

A 28-year-old man seeks help for progressive edema affecting his right lower extremity. It first started about 2 years ago but has become much worse in the last 6 months. Physical examination shows pitting edema from the groin to the toes, as well as very prominent varicose veins and an area of chronic cellulitis above the medial malleolus. These findings are confined to the right side and are not present at all on the left. He also is noted to have a palpable thrill and an audible bruit over the right groin. His resting pulse is 115/min. He gives a history of having suffered a gunshot wound to his right groin 3 years ago, for which he had no specific therapy. Which of the following is the most likely diagnosis? A. Arteriovenous fistula B. Chronic lymphedema C. Deep thrombophlebitis D. Postphlebitic syndrome E. Superficial saphenous venous insufficiency

The correct answer is A. The bruit and the resting pulse rate are virtually pathognomonic for an arteriovenous fistula, particularly in view of the history of untreated penetrating trauma to the groin. Significant venous hypertension can develop in that setting, leading to the other presenting symptoms.

For patients with hypertension, which of the following is the most common initial eye finding? A. AV nicking B. Flame hemorrhage C. Hard exudates D. Increased disc-to-cup ratio E. Papilledema

The correct answer is A. The earliest eye manifestations of hypertension are AV nicking and cotton wool spots. Vessels are visible in the eye, and these vessels display the manifestations of the hypertensive process.

Which of the following is the door-to-drug goal for administration of fibrinolytic therapy following arrival at a hospital when a patient has a STEMI? A. 30 minutes B. 60 minutes C. 90 minutes D. 120 minutes E. 3 hours

The correct answer is A. The goal is 30 minutes for door-to-drug therapy in the setting of STEMI. Patients who arrive with acute chest pain are immediately risk stratified according to their EKG findings and history even if the cardiac enzymes have not yet become positive. The arrival of the patient to drug delivery time for STEMI patients is within 30 minutes.

Which of the following patients post-myocardial infraction (MI) should receive anticoagulants following discharge? A. Patients with large dyskinetic region following anterior wall MI B. Patients with ongoing chest pain C. Patients with unstable angina D. Patients with pulmonary hypertension E. Patients with large inferior wall MI

The correct answer is A. The indications for utilizing anticoagulants post-MI include mural thrombus, dyskinetic region after anterior wall MI, or severe left ventricular dysfunction. Patients with a large dyskinetic left ventricle are at high risk for the development of mural thrombosis in the left ventricle, so anticoagulants are used to prevent the development of a mural thrombosis.

The mortality rate per year is greatest with disease of which coronary anatomy? A. Left main coronary B. Right coronary C. Left circumflex D. Left anterior descending

The correct answer is A. The left main coronary artery is the primary arterial supply to the left ventricle. Blockage of this artery leads to anterior wall and lateral wall myocardial infarction. Mortality for patients who have left main coronary artery disease is more than 10 times greater than in patients who have one- or two-vessel disease involving the other coronaries.

Thiazide diuretics are recommended as first-line medication for patients with hypertension. Side effects of these medications include all of the following EXCEPT: A. Decreased LDL cholesterol levels B. Hyperglycemia C. Increased triglyceride levels D. Increased uric acid levels E. Metabolic alkalosis

The correct answer is A. The metabolic profile affected by thiazide diuretics is an increase in serum triglycerides and not due to an effect on the LDL cholesterol.

A 50-year-old man is admitted to the hospital after sustaining an acute non-ST segment myocardial infarction. He is not having any further chest pain and he has no dyspnea. Eight hours after this event, his blood pressure is 70/50 mm Hg and his pulse is 45/min. An electrocardiogram reveals sinus bradycardia rhythm. Which of the following is the most appropriate intervention? A. Administer atropine intravenously B. Administer dobutamine C. Administer a beta-blocker D. Insert a transvenous pacemaker E. Perform cardiac catheterization

The correct answer is A. The patient is hypotensive and bradycardic. This suggests a vagal response, and administering an agent that is vagolytic, such as atropine, is the correct treatment. Atropine is given for symptomatic bradycardia post-MI at a dosage of 0.5 mg every 5 minutes up to a total of 3 mg. As the vagus nerve is blocked, the patient's heart rate will increase.

An arteriogram is performed on a patient who has atherosclerosis. Luminal narrowing of which of the following vessels would compromise blood flow through the renal arteries? A. Abdominal aorta B. Celiac trunk C. Common iliac artery D. Inferior mesenteric artery E. Superior mesenteric artery

The correct answer is A. The renal arteries emerge from the abdominal aorta at about the level of the L1/L2 intervertebral disk and travel at nearly right angles to it (on the right, passing posterior to the inferior vena cava) to enter the hilum of the kidney.

A 55-year-old woman with a long-standing history of atrial fibrillation secondary to mitral regurgitation comes to the emergency department with a painful right foot. The patient reports that over the past few hours her foot has become more painful and now is nearly insensate. She describes the pain as burning and states that it is not relieved by any intervention. She takes warfarin (Coumadin), atenolol (Tenormin), digoxin (Lanoxin), and aspirin. On physical examination, her pulse is irregularly irregular. Her lungs are clear, and she has a loud holosystolic murmur heard best at the apex. Her right foot is gray and cool to the touch and has poor capillary refill. Dorsalis pedis and posterior tibial pulses are absent on the right. Her prothrombin time is 14.4 seconds (normal 11-13 seconds) and her INR is 1.4 with goal of anticoagulation being 2-3. Which of the following is the most appropriate course of action? A. Arrange for her to be seen urgently by a vascular surgeon in the emergency department now B. Arrange for her to be seen by a neurologist within the next few days C. Arrange for her to undergo an MRI of the head emergently D. Ask her to make an appointment to be seen in your office within 1 week E. Instruct her to soak her leg in warm water and to place a fitted stocking on her affected leg

The correct answer is A. The symptoms and signs that she is describing, particularly in the context of atrial fibrillation (AF), suggest peripheral embolization, which is a surgical emergency. The treatment of choice involves immediate embolectomy performed by a vascular surgeon, TPA infusion therapy, or insertion of a catheter with urokinase-directed therapy. After successful embolism removal or dissolution, anticoagulation is performed along with administration of heparin, which is given to prevent new clot formation. Her subtherapeutic prothrombin time and persistent AF on examination are supportive of this diagnosis.

A 55-year-old man with a recent syncopal episode is admitted to the hospital with congestive heart failure. His blood pressure is 160/100 mm Hg and pulse 90/min. He has a grade 2/6 harsh systolic ejection murmur. An echocardiogram reveals a thickened ventricular septum and systolic anterior motion of the mitral valve. Which of the following will most likely be found in this patient? A. Decreased murmur with hand grip B. Decreased murmur with Valsalva C. Delayed carotid upstroke D. Increased murmur with squatting E. Murmur radiating to carotid arteries

The correct answer is A. The thickened ventricular septum and the systolic anterior motion of the mitral valve suggest hypertrophic cardiomyopathy. The murmur is harsh and systolic and decreases when afterload increases as a result of hand grip exercise. Hand grip increases systemic vascular resistance, which allows the heart to fill with blood against increased afterload; this filled ventricle will lessen the obstruction that occurs with hypertrophic cardiomyopathy.

When treating a patient with hypertension with diuretics, which of the following advantages do thiazide diuretics have over loop diuretics? A. Longer duration of action B. Enhanced activity in the setting of chronic kidney disease C. More potent diuresis D. Much cheaper alternative E. Does not cause hypokalemia

The correct answer is A. Thiazide diuretics have a longer duration of action than loop diuretics, allowing them to be dosed on a once-daily basis and to be effective for blood pressure control for the entire 24-hour period.

A 4-year-old, apparently healthy child is examined by a health care provider. The health care provider hears a loud systolic ejection murmur with a prominent systolic ejection click. He also hears a soft, early diastolic murmur. Both murmurs are heard best at the upper right sternal border. An electrocardiogram shows left ventricular hypertrophy. The child is active and participates in normal activities without problems. Which of the following is the most likely diagnosis? A. Aortic valve stenosis B. Atrial septal defect C. Tetralogy of Fallot D. Transposition of great arteries E. Ventricular septal defect

The correct answer is A. This is aortic valve stenosis, which accounts for 5% of diagnosed cardiac defects but may actually be the most common congenital anomaly of the heart, because many minor cases are never diagnosed. Most cases are caused by bicuspid aortic valves and characteristically produce a systolic ejection murmur. An accompanying aortic insufficiency may produce an early diastolic murmur. The timing of surgical correction depends on the severity of the individual case.

A 54-year-old man comes to his health care provider complaining of intermittent palpitations. The patient reports that a few times over the past few months he has had episodes of "pounding in his chest" that are associated with shortness of breath and occasional chest pain. He is forced to sit down if he is standing, because of weakness and light-headedness. The patient has a history of hypertension and mitral valve prolapse. He takes nifedipine (Procardia, Adalat) and hydrochlorothiazide daily. While sitting in the office, the patient begins to complain of increasing shortness of breath and palpitations. His blood pressure is 85/50 mm Hg and his pulse is 110-130/min and irregularly irregular. Which of the following is the most appropriate management at this time? A. Call 911 for assistance B. Give the patient an oral dose of a beta-blocker C. Give the patient an oral dose of digoxin D. Give the patient an oral dose of a calcium-channel blocker E. Make arrangements to have the patient brought to the local ED for electrical cardioversion

The correct answer is A. This patient has acute atrial fibrillation (AF) with rapid ventricular response (RVR) and is consequently hypotensive. This is a medical emergency. Even the health care provider caring for this patient is ill equipped to deal with a potentially life-threatening episode of AF with RVR. Activating the emergency medical response system is always appropriate and ensures that trained persons who have additional equipment and medications appropriate to an emergency situation will be on hand as soon as possible. The patient will need to have IV access, intravenous fluid support, and medication or cardioversion as part of the treatment regimen. According to the Advanced Cardiac Life Support protocol, a patient who has atrial fibrillation with a rapid ventricular response who is hypotensive should be cardioverted, something that emergency medical personnel will have available to them.

A 64-year-old woman comes to her primary health care provider for management of her hypertension, which has been treated unsuccessfully for several years. She was recently hospitalized for pulmonary edema, and an echocardiogram at that time showed a moderately depressed ejection fraction. She was diagnosed with congestive heart failure. Her medications include a thiazide diuretic and a calcium channel blocker. She has an allergy to furosemide. Her review of systems is positive for two-pillow orthopnea and occasional paroxysmal nocturnal dyspnea. On physical examination, her blood pressure is 150/80 mm Hg and her pulse is 80/min and regular. Her lungs are clear, and there are no extra heart sounds. Her extremities are without edema. Which of the following is the most appropriate management at this time? A. Add an ACE inhibitor to her regimen B. Add an angiotensin II receptor blocking agent to her regimen C. Add hydralazine (Apresoline) to her regimen D. Increase the dose of her calcium channel blocker E. Increase the dose of her thiazide diuretic

The correct answer is A. This patient has both hypertension and congestive heart failure (CHF). An important concept to recognize in the treatment of medical conditions is that certain medications overlap syndromes and are efficacious in many areas. This "co-treatment" option maximizes each drug in a regimen and often addresses two or more issues simultaneously. In this case, ACE inhibitors have been shown to be very beneficial in prolonging the survival of CHF patients. ACE inhibitors are first-line therapy for patients with chronic heart failure and also have the additional benefit of being able to also improve the blood pressure.

A patient is actively coding with polymorphic ventricular tachycardia noted on the cardiac monitor. Which of the following is the most common underlying cause for the development of torsades de pointes? A. Prolonged QT interval B. Third-degree heart block C. Ventricular septal defect D. Sustained ventricular tachycardia E. Acute myocardial infarction

The correct answer is A. Torsades is most commonly caused by prolonged QT interval. Prolongation of the QT interval exposes the patient to the R-on-T phenomena during the relative refractory period of the T wave. If a strong enough impulse of conduction occurs on this relative refractory period because of a prolonged QT interval, the patient develops polymorphic ventricular tachycardia, which has a wavering baseline of electrical activity. The QT interval is corrected for the cardiac rate. Typically, a normal QT interval is less than 460 milliseconds in adult males and less than 480 seconds in adult females. The corrected QT interval is allowed to be somewhat longer in the setting of slow heart rates and should be shorter in the setting of faster heart rates.

A 16-year-old boy is brought to the emergency department by his parents because of an episode of severe chest pain several hours earlier. Review of symptoms reveals that the patient is recovering from a flu-like illness that started a week earlier. He had been complaining of fever, chills, abdominal discomfort, and feeling tired during the prior week. He has had a low-grade fever for 5 to 6 days and a sore throat, but pharyngeal swab cultures done at the primary care office had been negative and he was taking over-the-counter cold medication only. Physical examination reveals a well developed and well nourished young man in moderate distress. His temperature is 37.8°C (100.0°F), pulse is 120/min, and respirations are 28/min. A differential white blood cell count shows a normal number of neutrophils and marginally elevated lymphocytes. An electrocardiogram shows low voltage QRS complexes throughout the limb leads. Chest radiography is remarkable for increased pulmonary markings and an enlarged heart silhouette. Which of the following is the most appropriate next step in the management? A. Admit to monitored floor for further evaluation and management B. Discharge from emergency department after stabilization of vital signs C. Overnight observation in the emergency department D. Reassurance and discharge E. Recommend symptomatic antipyretic therapy, bed rest, a low salt diet, and discharge from emergency department

The correct answer is A. This patient has signs of myocarditis and should be admitted to a monitored hospital bed for further evaluation and management. Myocarditis is an inflammation of the myocardium. The etiology may be multiple, but it is most commonly viral, caused by adenovirus and Coxsackie B. Bacteria (diphtheria), Rickettsia, fungi, and parasites are also infectious causes of myocarditis. Connective tissue diseases, granulomatous diseases, and toxins may cause noninfectious myocarditis. The most common clinical presentation is heart failure. Older children may present with chest pain secondary to myocardial ischemia or concurrent pericarditis. Arrhythmias and sudden death are less common. The age of the patient also influences the clinical presentation, and it is more acute or fulminant in infancy. Viral myocarditis is usually preceded by a viral illness. In these cases, the patient may present with fever, heart failure, respiratory distress, and cyanosis. The erythrocyte sedimentation rate, creatine kinase, and lactate dehydrogenase all may be elevated. The presence of lymphocytosis or neutropenia on a differential white blood cell count supports the diagnosis of viral myocarditis. Serum viral titers are helpful when they are positive, and polymerase chain reaction may help identify the specific virus. Chest radiography shows an enlarged heart and pulmonary edema. Electrocardiography shows sinus tachycardia, reduced QRS complex, and abnormal S and ST waves. Echocardiography shows poor ventricular function and possible pericardial effusions, and absence of congenital heart disease and coronary artery involvement. The diagnosis is confirmed by endomyocardial biopsy. Treatment includes management of heart failure and arrhythmias. In the acute phase, the patient should be admitted to the hospital even if only mild signs of respiratory distress or congestive heart failure are present. Rapid progression to overt heart failure or hemodynamic collapse may occur. Pericardiocentesis is performed to alleviate the tamponade if present. The role of systemic steroids is controversial. If refractory to medical management, heart failure is ultimately treated with heart transplantation. Although spontaneous resolution may occur, most patients do very poorly without treatment.

A 67-year-old man comes to the health care provider complaining of increasing dyspnea on exertion. Over the past 3 weeks, he has noted increasing shortness of breath while walking to the bus stop three blocks from his home. He has also found it difficult to sleep comfortably while lying flat and now sleeps on three pillows. He has awoken several times in the past month with shortness of breath. He denies any history of chest pain. He does not smoke, and his cholesterol tests were normal 1 year earlier. On physical examination, he appears comfortable at rest. His blood pressure is 158/56 mm Hg, pulse is 86/min and regular, and respirations are 16/min. He has jugulovenous distention while being examined at 30 degrees. His lungs have bibasilar rales. On cardiac examination, there is a regular S1 and S2, with a blowing diastolic murmur heard loudest at the left sternal border. A I/VI systolic murmur is heard throughout the precordium. An extra heart sound immediately following the S2 is heard. There is mild lower extremity pedal edema bilaterally. Which of the following is the most likely cause of his symptoms? A. Aortic insufficiency B. Aortic stenosis C. Mitral regurgitation D. Mitral stenosis E. Tricuspid regurgitation

The correct answer is A. This patient has the typical symptoms of congestive heart failure with left ventricular failure. The physical examination has the characteristic findings for aortic insufficiency, including wide pulse pressure, blowing diastolic murmur, and an S3 consistent with left ventricular dilatation. The soft systolic murmur is caused by the volume overload and increased flow in the left ventricle and does not imply an additional valvular disorder.

A patient is seen in the office for an acute care visit after he underwent an arteriogram earlier in the day. He was told that he has significant atherosclerosis in his abdominal aorta. He is having a great deal of pain in his abdomen and in his right lower extremity. Examination reveals blue/black toes. Which of the following is the preferred treatment at this time? A. Supportive care, control of hypertension B. Thrombectomy C. Anticoagulation with low-molecular-weight heparin D. Anticoagulation with warfarin E. Infusion of urokinase as thrombolytic agent

The correct answer is A. Treat cholesterol embolization syndrome with supportive care and control of BP.

A 49-year-old man with a history of hypertension and hyperlipidemia has had a severe headache, blurry vision, and red urine for the past 2 days. He was in his usual state of health until 2 days ago. He has been compliant with his medications, including atenolol 50 mg/day and hydrochlorothiazide 12.5 mg/day. His temperature is 37.0°C (98.6°F), blood pressure 196/140 mm Hg, pulse 83/min, and respirations 10/min. The remainder of his physical examination is unremarkable. Which of the following is the most appropriate intervention at this time? A. Admit him to the hospital for evaluation and treatment B. Begin therapy with clonidine (Catapres) and recheck bp in 30 to 60 minutes C. Begin therapy with felodipine (Plendil) and recheck bp in 24 to 48 hours D. Increase the doses of his current medications and recheck bp in 1 week E. Schedule an outpatient CT scan of the head and recheck bp in 1 week

The correct answer is A. This patient is having a hypertensive emergency, which is defined as severe hypertension with end-organ damage. It happens for unknown reasons in approximately 1% of hypertensive patients. Our patient has hematuria, headache, and blurred vision. He needs to be admitted to the hospital and placed on intravenous blood pressure medications such as labetolol as soon as possible. Remember not to lower the blood pressure too far, too rapidly, because doing so might compromise cerebral, renal, or coronary perfusion. The goal for decreasing blood pressure is no more than 25% of the mean arterial blood pressure within the first minutes to hours.

A patient with long-standing mitral stenosis is seen in the office. Which of the following EKG abnormalities would most likely be present if an in-office EKG is performed? A. Left ventricular hypertrophy B. Atrial fibrillation C. First-degree AV heart block D. Second-degree type 1 AV heart block E. Second-degree type 2 AV heart block

The correct answer is B. Atrial fibrillation and left atrial enlargement are the most common EKG abnormalities seen in a patient with long-standing mitral stenosis. Patients with long-standing mitral stenosis will have pressure overload produced in the left atrium, which leads to left atrial enlargement. Patients with big, floppy left atria have multiple irritable sites in the left atrium, which leads to atrial fibrillation.

A 37-year-old African American man with a history of mild hypertension comes in for an annual examination. He has no complaints. He reports compliance with his low-salt diet. His only medication is hydrochlorothiazide, 25 mg each day. A thorough review of systems is negative. His temperature is 37.0°C (98.6°F), blood pressure 160/90 mm Hg, pulse 83/min, and respirations 10/min. Physical examination is within normal limits. Laboratory studies show: Na+ 142 mEq/L K+ 3.9 mEq/L Cl− 109 mEq/L HCO3− 22 mEq/L Glucose 96 mg/dL BUN 13 mg/dL Creatinine 0.6 mg/dL Which of the following is the most appropriate intervention at this time? A. Add a second antihypertensive medication B. Schedule for a captopril renal scan C. Send a 24-hour urine collection for cortisol D. Send a 24-hour urine collection for vanillylmandelic acid, metanephrine, and catecholamine E. Send urine and serum aldosterone levels

The correct answer is A. This patient most likely has essential hypertension. Secondary forms of hypertension should only be suspected in those patients who develop hypertension at extremes of age (less than 25 or more than 55 years of age), who have suspicious physical examination findings, or who are resistant to multiple blood pressure medications. This patient is a healthy man who is only on one medication and does not have any laboratory or physical examination stigmata of secondary hypertension. Simply adding another medication to his current regimen is appropriate at this time.

A 22-year-old man is evaluated for mitral regurgitation due to mitral valve prolapse. Examination reveals a tall, slender young man with long extremities and long tapering fingers. Pupillary dilation followed by slit-lamp examination reveals bilateral dislocation of the lenses of the eyes. This patient is potentially at increased risk for development of which of the following? A. Aortic dissection B. Lisch nodules C. Noncaseating granulomata D. Progressive dementia E. Rapidly progressive renal failure

The correct answer is A. This young man is displaying features of Marfan syndrome, a genetic (often autosomal-dominant) disease of connective tissue that affects the skeleton (tall stature, long fingers and toes, hyperextensible joints), eyes (often subluxation of the lenses bilaterally), and the cardiovascular system (cystic medial necrosis predisposing to aortic dissection or aortic valve incompetence). Marfan syndrome is caused by mutations in the fibrillin gene, leading to defects in the structure of elastic tissue.

A patient has a follow-up echocardiogram performed after a recent hospitalization. The report notes that this patient has tricuspid regurgitation. Which of the following is the most likely cause for tricuspid regurgitation to occur? A. Left ventricular failure B. Cor pulmonale C. Acute myocardial infarction D. Primary pulmonary hypertension E. Ventricular septal defect

The correct answer is A. Tricuspid regurgitation is typically due to right ventricular enlargement due to left ventricular failure, which is often the result of chronic heart failure. Chronic heart failure is a common condition that is the most common cause of hospital admissions. When the left ventricle starts to fail with either preserved or reduced ejection fraction, there is a backup of blood into the left atrium and into the pulmonary system. This will cause the right ventricle to hypertrophy (cor pulmonale), which leads to dilation of the right ventricle and then the right atrium. This dilation of the right side of the heart causes tricuspid regurgitation because the tricuspid valve cannot close and seal.

In addition to fixed splitting of S2, which of the following auscultatory findings occurs with atrial septal defect? A. Tricuspid regurgitant murmur B. Tricuspid stenosis murmur C. Aortic stenotic murmur D. Mitral regurgitant murmur E. Opening snap

The correct answer is A. Tricuspid regurgitation occurs in the setting of atrial septal defect due to increased blood flow on the right side of the heart.

Health care providers need to have working knowledge with regard to several congenital heart problems. In some circumstances, patients can survive longer periods of time despite having universally fatal types of congenital heart disease. Which of the following underlying cardiac conditions leads to the development of Eisenmenger syndrome? A. Large, uncorrected left-to-right shunts B. Acute pulmonary edema C. Acute pulmonary embolism D. Use of lithium carbonate during pregnancy E. Acute anterior wall STEMI

The correct answer is A. Uncorrected large left-to-right cardiac shunts (atrial septa defect, ventricular septal defect, patent ductus arteriosus) lead to right-sided heart overload and pulmonary hypertension. Eventually, pulmonary hypertension leads to reversal of this shunt, sending deoxygenated blood to the systemic circulation, which is what Eisenmenger syndrome is. When this right-to-left shunting occurs, there is little that can be done with regard to lengthening a person's survival.

A patient is seen in the hospital post myocardial infarction. The patient is suspected of having complications following this MI. Which of the following outcomes is most common if a patient has ventricular septal defect as a complication from an MI? A. Heart failure B. Mitral regurgitation C. Cardiac tamponade D. Embolic phenomena E. Arrhythmia

The correct answer is A. Ventricular septal defect can lead to heart failure or shock. This condition is more likely to occur in the setting of anterior wall MI, especially if the ventricular septum is involved. When this condition occurs, hypotension and pulmonary edema can result. VSD is heralded by new-onset holosystolic murmur, which is loudest at the lower left sternal border.

Patients with Wolff-Parkinson-White (WPW) syndrome will have a delta wave plus which of the following EKG changes? A. Shortened PR interval B. Suppressed PR interval C. Elevation of ST segment in the right-sided chest leads D. Narrowed QRS wave E. Left-axis deviation

The correct answer is A. WPW patients have a short PR interval and a wide QRS with a slurred upstroke known as a delta wave. This is a type of preexcitation syndrome in which an accessory pathway connects the atrium and ventricle. This accessory pathway results in shortening of the PR interval.

A 55-year-old man with no known cardiac history comes to the emergency department complaining of crushing substernal chest pressure that began 20 minutes prior to his arrival. He took sildenafil earlier in the evening prior to sexual intercourse. His past medical history is significant for bilateral inguinal hernia repairs 6 weeks ago, remote peptic ulcer disease, and prior cocaine and heroin addiction. On examination, he is diaphoretic and appears anxious. His blood pressure is 150/75 mm Hg with a pulse of 100/min. An electrocardiogram (ECG) obtained while he had severe chest pain revealed tall positive T waves and 1-mm elevation of the ST segments in leads V2 through V5. A serum creatine kinase (CK) drawn on presentation returned at 85 U/L (normal <250 U/L). Which of the following is the most likely diagnosis? A. Aortic dissection B. Myocardial infarction (MI) C. Pulmonary embolus (PE) D. Spontaneous pneumothorax E. Stable angina

The correct answer is B. The combination of substernal chest pressure with ECG findings of hyperacute T waves and ST elevation in this clinical setting suggests early anterior myocardial infarction. The myocardial damage causes time-dependent effects on the electrical properties of the myocardial cells. The earliest electrical evidence of myocardial injury is often development of hyperacute T waves, followed by elevation of the ST segments, inversion of the T wave, return of the ST segments to normal, and finally, the development of Q waves. It does not matter that the CK, a marker of myocardial injury, is not elevated, as it takes several hours for the CK to appreciably rise after myocardial damage has occurred. It does not matter that the serum CK, a marker of myocardial injury, is not elevated, as it takes several hours for the CK to appreciably rise after myocardial damage has occurred.

A health care provider examines a 2-month-old infant who had been born at term. There is a continuous murmur at the upper left sternal border. The murmur radiates over the lung fields anteriorly. The peripheral pulses in all extremities are full and show widened pulse pressure. Which of the following is the most likely diagnosis? A. Coarctation of the aorta B. Patent ductus arteriosus C. Peripheral pulmonic stenosis D. Persistent truncus arteriosus E. Ventricular septal defect

The correct answer is B. This is patent ductus arteriosus, which is a failure of closure of the duct between the pulmonary artery and the aorta. As many as 80% of significantly premature (<28-week gestation) infants have patent ductus arteriosus. In term infants, delayed closure is diagnosed if the murmur of the patent ductus (described in the question stem) is still present at 6-8 weeks of age. Indomethacin is given soon after birth in premature infants and is typically started within the first 12 hours of life. This treatment is given only during the first 3 days of life in an attempt to close the patent ductus. Infants should be evaluated for other cardiac disease, as a patent ductus arteriosus may be partially compensating for other cardiac anomalies. Infants who have heart failure require prompt surgical correction. Infants who do not have heart failure or complicating cardiac defects typically undergo elective surgery at 6 months to 3 years of age to reduce the risk for infective endocarditis later involving the patent ductus. If this condition is identified in the adult population, treatment consists of a percutaneous catheter to close or surgical ligation of this structure.

A 40-year-old woman is brought to the emergency department following a suicide attempt with imipramine (Tofranil). Her fiancé found her unresponsive, with an empty bottle of the imipramine at her side. The imipramine had been his, and the prescription had been filled that morning. Her past medical history is significant for hypertension, atrial fibrillation, diabetes, and asthma. Her medications include furosemide, procainamide, glyburide, prednisone, and albuterol. She has no known drug allergies. She is afebrile, has a blood pressure of 100/60 mm Hg, pulse of 62/min, and respirations of 22/min. A gastric lavage yields multiple pill fragments. She is confused and somnolent, and has shallow respirations. Her physical examination is otherwise unremarkable. On an electrocardiogram, which of the following abnormalities would most likely reflect possible cardiac toxicity? A. Left deviation of the QRS axis B. Prolongation of the QT interval C. Shortening of the PR interval D. ST-segment depression E. T-wave inversion

The correct answer is B. A prolongation of the QT interval is highly predictive of both cardiac and CNS toxicities from tricyclic antidepressant ingestion. This medication has high lethality associated with its overdose because of its cardiac effects.

A patient presents acutely. He is experiencing severe pain in his left lower extremity. He also notes that this lower extremity is cooler than the opposite side. Examination reveals no palpable pulse. Which of the following is the most likely diagnosis? A. Leriche syndrome B. Acute arterial occlusion C. Cholesterol embolization syndrome D. Deep vein thrombosis E. Chronic venous insufficiency

The correct answer is B. Acute arterial occlusion causes pain, pallor, pulselessness, paralysis, polar, and paresthesias.

Although consensus guidelines have changed for using prophylaxis against endocarditis, there are still certain circumstances in which prophylaxis must be given. For patients who have no known drug allergies, which of the following medications is considered to be the medication of choice used in endocarditis prophylaxis for at-risk patients undergoing GI or GU surgery? A. Azithromycin B. Amoxicillin C. Clarithromycin D. Ciprofloxacin E. Clindamycin

The correct answer is B. Amoxicillin is the most commonly used prophylactic agent against endocarditis, as it is able to cover gram-negative and gram-positive organisms found in the genitourinary (GU) and gastrointestinal (GI) tracks.

A patient has signs and symptoms consistent with heart failure. An echocardiogram is performed, and the patient is found to have infiltrative type of cardiomyopathy. The echocardiogram interpretation notes that the myocardium appears brighter with a sparkled appearance. Which of the following is the underlying cause for this appearance? A. Sarcoidosis B. Amyloidosis C. Hemochromatosis D. Scleroderma E. Carcinoid syndrome

The correct answer is B. Amyloidosis causes infiltrative cardiomyopathy with bright myocardium and sparkled appearance due to these starch-like deposits. As amyloid deposits into the myocardium, the cardiac myocytes are replaced, which leads to a dilated cardiomyopathy.

An elderly patient is seen in the office. When assessing the patient's pulse, the pulse is noted to be weak and delayed following closure of the aortic valve. Which of the following heart abnormalities is most likely to cause a parvus and tardus pulse in the carotid arteries? A. Mitral stenosis B. Aortic stenosis C. Pulmonic stenosis D. Tricuspid stenosis E. Mitral regurgitation

The correct answer is B. Aortic stenosis causes parvus and tardus pulse. Because of aortic stenosis, blood is only able to be ejected into the systemic circulation in a small jet because of the smaller opening. This causes a flattening of the blood, and it takes longer for the blood to get into the systemic circulation. This is what causes a parvus pulse. Since the blood flow is ejected over a longer period of time prior to the aortic valve closing, a tardus pulse is felt.

A patient presents to the office complaining of recurrent, bothersome palpitations. The patient has eliminated caffeine and any other potential stimulating agents. If the patient is in need of medications to treat these bothersome symptoms, which of the following agents is recommended for therapy? A. Calcium channel blockers B. Beta blockers C. ACE inhibitors D. Digoxin (Lanoxin) E. Alpha blockers

The correct answer is B. Beta blockers decrease sympathetic output making them ideal agents for treating symptomatic PACs.

You see a patient with hypertension who has just become pregnant. She previously was managed with metoprolol (Lopressor) prior to becoming pregnant. If this patient is continued on this class of medications during her pregnancy, which of the following complications can be seen in the baby? A. Large for gestational age B. Hypoglycemia C. Immature lung function D. Urinary retention E. Constipation

The correct answer is B. Beta-blocker use during pregnancy is associated with low birth weight and hypoglycemia in the offspring. Since sympathetic activity in the baby is suppressed with the use of beta-blockers in the mother, the baby may not be able to release counterregulatory hormones to combat hypoglycemia and raise the blood sugar.

A patient with a known aortic aneurysm is seen in the office for ongoing, follow-up care. At this point, the aneurysm is not large enough for elective surgery to be performed. Which of the following medical treatments is indicated in this setting? A. Calcium-channel blockers B. Beta-blockers C. ACE inhibitors D. Magnesium E. Vitamin D

The correct answer is B. Beta-blockers are the medication of choice for aortic aneurysm management until the aneurysm reaches the size needed for elective surgery. Beta-blockers control hypertension and also decrease the shearing force on the endovascular wall, helping to also prevent aortic dissection.

Which of the following medications is considered to be the treatment for atrial premature beats that are causing symptoms? A. Calcium-channel blockers B. Beta-blockers C. Adenosine D. Nitrates E. Diuretics

The correct answer is B. Beta-blockers are used for tachycardia related to premature atrial beats, as they decrease the sympathetic outflow that may contribute to these abnormal beats.

Which of the following is the primary advantage to using bivalirudin (Angiomax) versus unfractionated heparin plus glycoprotein IIb/IIIa inhibitors in the setting of a patient with non-STEMI (NSTEMI) who undergoes percutaneous coronary intervention (PCI)? A. Longer patency of the involved vessel B. Less chance for bleeding C. Less chance for spasm of the involved vessel D. Less chance for allergic reaction E. Less risk for myocardial rupture

The correct answer is B. Bivalirudin is associated with less bleeding in the setting of NSTEMI and cardiac catheterization than is the combination of heparin with glycoprotein IIb/IIIa inhibitors. The primary adverse action of anticoagulants used in the setting of PCI is bleeding, so providing treatment that involves less bleeding may provide an advantage.

A patient is seen in the office following the performance of a cardiac catheterization. Which of the following cardiac catheterization findings is considered to be an indication for having a coronary artery bypass graft procedure (CABG)? A. Atrial fibrillation B. Left ventricular dysfunction C. Left bundle branch block pattern D. Cor pulmonale E. 100% occlusion of left circumflex artery

The correct answer is B. CABG is indicated when there is left ventricular dysfunction, left main, or three-vessel disease. Diabetic patients also tend to have better outcomes when actual coronary artery bypass is performed rather than stenting because diabetic patients often have premature closure of these stents.

Which of the following medications that could potentially be used in the management of ischemic heart disease has been shown to have an increased mortality when used in this setting? A. Beta blockers B. Calcium channel blockers C. ACE inhibitors D. Aspirin E. Nitrates

The correct answer is B. Calcium channel blockers have increased mortality with ischemic coronary disease, which is why these agents are not considered first-line agents. At best, calcium channel blockers are second-line agents.

Elderly patients and patients of African American heritage respond to treatment of hypertension with both calcium-channel blockers and thiazide diuretics. When dihydropyridine calcium-channel blockers are used in the management of hypertension, which of the following is the most common adverse action? A. Bronchospasm B. Constipation C. Urinary retention D. Rash E. Visual disturbance

The correct answer is B. Dihydropyridine calcium-channel blockers cause edema and constipation due to vasodilation. Bronchospasm (choice A) is a side effect that is commonly associated with beta-blockers. Urinary retention (choice C) is seen with the use of alpha-agonists, which can increase rather than decrease blood pressure. Rash (choice D) is not associated with the use of calcium-channel blockers. Visual disturbances (choice E) may be a complication for phosphodiesterase inhibitors but not calcium-channel blockers.

Which of the following medications used in the setting of acute MI results in an increased in cardiac output and lowering of the pulmonary wedge pressure but does not raise the blood pressure? A. Nitroglycerin B. Dobutamine C. Nitroprusside D. Dopamine E. Norepinephrine

The correct answer is B. Dobutamine, an inotropic agent, works by stimulating beta-1 receptors. Dobutamine improves cardiac output and lowers pulmonary capillary wedge pressure but does not result in increase in the blood pressure.

Which of the following cardiac rhythms is most commonly seen in the setting of chronic heart failure? A. Sinus bradycardia B. Sinus tachycardia C. Ventricular tachycardia D. Ventricular fibrillation E. Asystole

The correct answer is B. Due to increased sympathetic tone and compensation, sinus tachycardia occurs in the setting of chronic heart failure.

A patient is seen in the emergency department with complaints of chest pressure, nausea, dyspnea, and diaphoresis. Which of the following is the primary means of differentiating a patient with STEMI from a patient with non-STEMI (NSTEMI)? A. Cardiac biomarkers B. EKG findings C. Site of myocardial damage D. Presenting signs or symptoms E. Response to nitrates

The correct answer is B. EKG findings differentiate between STEMI and NSTEMI, as both will have positive cardiac biomarkers, and they may also have similar histories.

A patient with long-standing bipolar I disorder who took lithium throughout her pregnancy has her child seen in the office. There is a concern that the use of lithium has caused complications in the baby's heart. Which valve is primarily involved with Ebstein anomaly? A. Mitral B. Tricuspid C. Aortic D. Pulmonic E. All are equally involved

The correct answer is B. Ebstein anomaly most commonly involves the tricuspid valve. Affected babies will need ongoing monitoring for this congenital heart problem.

Which of the following treatments is indicated to prevent further events in the setting of arteriosclerosis leading to peripheral vascular disease? A. Oral anticoagulation B. Statins C. Vasodilators D. Beta-blockers E. Calcium-channel blockers

The correct answer is B. Statins and antiplatelet agents are used to prevent further events in a patient with arteriosclerosis and peripheral vascular disease.

As part of the overall cardiac evaluation of a patient with chest pain without a clear underlying cause, patients may have further clinical intervention performed. Assessment of myocardial muscle viability with gadolinium-enhanced MRI is contraindicated in which of the following patients? A. Those with acute hepatitis B. Those with impaired renal function C. Those who are not able to exercise D. Those who have reactive airway disease E. Those with uncontrolled psoriasis

The correct answer is B. Gadolinium cannot be used in the setting of impaired renal function because the gadolinium will not be able to be cleared from the systemic circulation, leading to toxic accumulation and adverse reactions.

Which of the following electrolyte abnormalities is associated with the development of atrioventricular (AV) block? A. Hypercalcemia B. Hyperkalemia C. Hypernatremia D. Hypomagnesemia E. Hypochloremia

The correct answer is B. Hyperkalemia and hypermagnesemia are associated with AV block and bradycardia. Hyperkalemia affects repolarization of the ventricles and can also be toxic to the cardiac conduction system.

Heart failure may have its pathophysiology related to pump failure, but it can also be related to diastolic failure, which has preserved ejection fraction. When patients develop symptoms of heart failure but continue to have appropriate cardiac output from the left ventricle, the health care provider should consider underlying causes for this condition to occur. Which of the following conditions is the most common underlying cause for diastolic heart failure? A. Anemia B. Hypertension C. Diabetes D. Restrictive cardiomyopathy E. Infiltrative cardiomyopathy

The correct answer is B. Hypertension is the most common underlying cause of chronic heart failure. Hypertension causes increased left ventricular outflow resistance, and the left ventricle must increase left ventricular contraction to overcome this increased outflow resistance. This increased force of contraction leads to increased filling resistance, which directly leads to diastolic heart failure (also known as heart failure with preserved ejection fraction). With this diastolic heart failure, echocardiogram will show normal heart valves and normal ejection fraction.

Patients with prolonged QT intervals are at risk for the development of sudden cardiac death because of the tendency for torsades de pointes to occur. All of the following are known causes of prolonged QT being found on EKG EXCEPT: A. Hypokalemia B. Hyponatremia C. Hypocalcemia D. Congenital inheritance E. Certain medications such as class III antiarrhythmics

The correct answer is B. Hyponatremia does not affect QT length. Hyponatremia is really a problem with water overload rather than a lack of sodium in the body. Hyponatremia does not affect cardiac conduction or ventricular repolarization, so it does not affect the QT interval.

Health care providers who perform invasive clinical interventions need to be aware of the indications, contraindications, and mechanisms of action for each of these clinical interventions. Which of the following is the most likely outcome of placement of intraaortic balloon counterpulsation pump (IABP) that is used in the setting of cardiogenic shock? A. Reduces diastolic heart failure B. Maintains systolic blood pressure while reducing pulmonary capillary wedge pressure C. Improves urinary output D. Increases aortic regurgitation E. Increases mitral regurgitation

The correct answer is B. IABP improves cardiac output, maintaining systolic blood pressure, while reducing the pulmonary capillary wedge pressure. This device results in keeping the blood coming out of the heart in the systemic circulation and provides an additional forward thrust of this blood into the systemic circulation, which effectively increases the systolic blood pressure. When blood is thrust forward into the systemic circulation, the heart is unloaded, which results in lowering of the pulmonary capillary wedge pressure.

Different types of shock need to have different treatment. Which of the following is considered to be the mainstay of treatment for a patient with neurogenic shock? A. Lumbar puncture to remove excessive fluid B. IV fluid resuscitation C. Loop diuretics D. Vasopressors E. Nitrates

The correct answer is B. IV fluid resuscitation will correct one of the deficits in neurogenic shock, which will allow the patient to tolerate the bradycardia.

A 60-year-old woman with a long history of hypertension comes to the emergency department with a complaint of increasing and recurrent shortness of breath with minimal exertion. She is noted to have a prominent precordial impulse, and a chest radiograph reveals a prominent left ventricular shadow. A stress test is negative for ischemia. She is found to have left ventricular hypertrophy and a normal ventricular ejection fraction on echocardiogram. An S3 is heard on auscultation of the heart. Which of the following is the most likely underlying diagnosis causing this presentation? A. Chronic obstructive pulmonary disease (COPD) B. Diastolic dysfunction C. Myocardial ischemia D. Reactive airways disease E. Systolic congestive heart failure

The correct answer is B. Increased resistance to filling of one or more cardiac ventricles has been termed diastolic heart failure and can produce increased pulmonary capillary wedge pressures and respiratory complaints. In myocardial hypertrophy, impaired diastolic relaxation occurs. The key to differentiating systolic from diastolic types of heart failure is in the assessment of ejection fraction; it is normal and preserved with diastolic heart failure and impaired with systolic heart failure.

A patient is seen in the office and admits a history of IV drug use. He is febrile and has a newly discovered cardiac murmur. The patient is noted to have erythematous lesions noted on his palms and soles that are not causing symptoms. What are these lesions known as? A. Herald patches B. Janeway lesions C. Roth spots D. Target lesions E. Osler nodes

The correct answer is B. Janeway lesions are painless red lesions noted on the palms and soles that occur in the setting of infective endocarditis.

Patients who have certain metabolic measurements may have increased risk for type 2 diabetes and a higher potential for coronary artery disease. Components of the metabolic syndrome include all of the following EXCEPT: A. Low high-density lipoprotein (HDL) levels B. High low-density lipoprotein (LDL) levels C. High triglycerides D. Central obesity E. Insulin resistance

The correct answer is B. LDL level is not a component of metabolic syndrome, as this measurement is not typically directly measured. Even though high LDL is associated with an increased risk for cardiovascular risk, it is not part of what comprises metabolic syndrome.

Health care providers should be able to perform special maneuvers in order to make certain heart abnormalities more prominent. Which of the following maneuvers will cause the pericardial friction rub to become louder in the setting of acute pericarditis? A. Taking a deep breath B. Leaning forward C. Left lateral decubitus position D. Valsalva maneuver E. Recumbency with elevation of legs

The correct answer is B. Leaning forward exacerbates the pericardial friction rub in the setting of acute pericarditis. When the patient leans forward, the heart is brought into a more anterior plane, allowing the pericardial friction rub to become more prominent.

A patient having an acute MI is at increased risk for the development of stroke for at least the next 5 years following the acute event. Which of the following findings is most predictive for increasing the risk of stroke in this setting? A. Frequency of PVCs B. Left ventricular ejection fraction C. LDL level D. Diastolic blood pressure E. Pulmonary artery pressures

The correct answer is B. Left ventricular ejection fraction is the best predictor for stroke following acute MI. Patients with a depressed left ventricle are most likely to develop mural thrombus with evacuation of this thrombus into the systemic circulation resulting in stroke.

A patient is seen in the emergency department with crushing, tearing chest pain. The patient's lower extremity blood pressure is decreased, and the patient is noted to have a widened mediastinum on chest x-ray. Which of the following conditions is the most common underlying cause for a patient developing this condition? A. Trauma B. Long-standing hypertension C. Coarctation of the aorta D. Marfan syndrome E. Bicuspid aortic valve

The correct answer is B. Long-standing hypertension is present in 70% of patients with aortic dissection. Failure to control the blood pressure can lead to increased shearing of the intima and increase the risk for dissection.

Rheumatic fever may be a consideration for patients who are immigrants from countries that have this as an endemic condition. The Jones criteria are used in order to confirm the diagnosis of rheumatic fever. Which of the following signs or symptoms is part of the major rather than the minor Jones criteria? A. Polyarthralgias B. Subcutaneous nodules C. Elevated SED rate D. Evidence of preceding strep infection E. Prolonged PR interval

The correct answer is B. Major Jones criteria for rheumatic fever include carditis, migratory polyarthritis, chorea, erythema marginatum, and subcutaneous nodules.

During the performance of a supraclavicular node biopsy under local anesthesia, a hissing sound is suddenly heard, and the patient suddenly dies. At the time of the catastrophic event, the target node was under traction, and the final cut was being made blindly behind it to free it up completely. The patient, an otherwise healthy 24-year-old man, was inhaling at that moment. Which of the following most likely caused this patient's death? A. Arterial injury with air embolization B. Major vein injury with air embolism C. Sudden pneumothorax with lung collapse D. Sympathetic discharge E. Tracheal injury

The correct answer is B. Major veins at the base of the neck have negative pressure during inspiration and, if injured at that moment, will suck air rather than bleed. The air embolism then leads to sudden death.

A patient is seen in the office. Auscultation of the heart reveals an S3, soft S1, and wide split to S2. There is a holosystolic murmur at the apex. Which of the following is the most likely diagnosis? A. Atrial septal defect B. Mitral regurgitation C. Mitral stenosis D. Pulmonic stenosis E. Pulmonic regurgitation

The correct answer is B. Mitral regurgitation causes a holosystolic murmur located at the apex. Mitral regurgitation leads to a widely split S2 because of more blood on the right side of the heart and pulmonary system from the blood returning to the left atrium when the mitral valve fails to fully close during left ventricular contraction.

A 63-year-old woman comes to the emergency department complaining of chest pain. The patient states that the pain began during her morning walk. It started as a dull pressure over her breastbone and then radiated to her left arm. Over the next few minutes, it escalated in intensity and was not relieved by rest. She called 911 and was brought to the emergency department. Her past medical history is significant for hypertension and hyperlipidemia. Her medications include atenolol and simvastatin daily. On physical examination, her blood pressure is 190/100 mm Hg, and her pulse is 60/min. Which of the following is the most appropriate agent to lower her blood pressure? A. Intravenous norepinephrine B. Intravenous nitroglycerin C. Oral hydrochlorothiazide D. Oral metoprolol (Lopressor) E. Oral furosemide (Lasix)

The correct answer is B. Nitroglycerin is a potent vasodilator that acts predominantly on venous compliance to reduce preload to the heart. When administered intravenously, it can be rapidly titrated to produce optimized blood pressure control. IV nitroglycerin is effective when given in the setting of unstable angina. Nitroglycerin also has the ability to cause vasodilation in the coronary arteries, which will also improve blood flow to the heart.

Multiple medications may be used in the setting of acute MI. Nitroglycerin used in this setting has which of the following benefits that the other vasodilators and inotropes do not have? A. Less likely to lead to hypotension B. Improves coronary artery blood flow C. Raises cardiac output to the highest extent D. Improves blood flow to the kidneys E. Less likely to cause arrhythmias

The correct answer is B. Nitroglycerin is a vasodilator of the coronary arteries and has the ability to improve coronary artery blood flow to ischemic myocardium. Nitroglycerin acts as a venodilator, which can result in hypotension (choice A) as a complication.

A 14-year-old boy is brought to the health care provider with decreased exercise tolerance. He is up to date on all of his childhood immunizations and has been generally healthy until now. He is noted to have a grade III/VI systolic ejection murmur best heard at the left upper sternal border and a grade II/VI mid-diastolic murmur at the lower left sternal border. The first heart sound is normal. The second heart sound is widely split and fixed. A right ventricular impulse is palpated. On a chest roentgenogram, the pulmonary artery segment is enlarged, and pulmonary vascular markings are increased. An electrocardiogram shows right axis deviation. Which of the following congenital heart diseases does this patient most likely have? A. Aortic stenosis B. Atrial septal defect C. Coarctation of the aorta D. Patent ductus arteriosus E. Ventricular septal defect

The correct answer is B. One of the most common types of structural congenital heart disease to present in adolescence is atrial septal defect (ASD), and the most common presentation is a heart murmur. Some patients, however, present with arrhythmias, decreased exercise tolerance, or a paradoxic embolus.

Medical management of chronic heart failure can involve multiple types of medications including ACE inhibitors, beta-blockers, and various types of diuretics and inotropic agents. This treatment may result in different electrolyte abnormalities. Which of the following electrolyte abnormalities should be avoided in a patient taking both diuretics and digoxin (Lanoxin)? A. Hyponatremia B. Hypokalemia C. Hypocalcemia D. Hyperuricemia E. Hypomagnesemia

The correct answer is B. Patients taking digoxin who become hypokalemic have an increased tendency for digoxin toxicity, even with normal digoxin levels.

Which of the following EKG abnormalities is treated as an equivalent to an ST-segment elevation acute myocardial infarction in a patient who has acute onset of chest pain? A. Right bundle branch block B. Left bundle branch block C. Left anterior hemiblock D. Chaotic atrial tachycardia E. PR-segment depression

The correct answer is B. Patients who have acute signs and symptoms consistent with myocardial infarction pattern who display a new-onset left bundle branch block pattern are treated equivalently to patients who have ST-segment elevation myocardial infarction until cardiac enzyme patterns are completed. Under appropriate circumstances, these patients are eligible for percutaneous cardiac intervention and thrombolytic therapy (if they arrive within an appropriate timeframe and there are no contraindications). Patients who have acute myocardial infarction often have such damage to the left ventricle that the repolarization pattern is affected, which results in a left bundle branch block pattern on the EKG. Other causes of left bundle branch block include heart failure, aortic stenosis, and left ventricular aneurysm.

A patient is seen in the emergency department with chest pain. The patient's initial EKG reveals ST-segment elevation in anatomically contiguous leads. The patient's cardiac enzymes are normal, and repeated testing of the enzymes remains normal. The patient's pain spontaneously resolves. Which of the following is the most likely diagnosis? A. Non-STEMI B. Coronary artery spasm C. Unstable angina D. Chronic stable angina E. Acute pericarditis

The correct answer is B. Patients with Prinzmetal angina or variant angina have ST-segment elevation and normal cardiac enzymes in the setting of chest pain. Prinzmetal angina is caused by coronary artery vasospasm, which can temporarily interfere with blood delivery to the heart. Typically, these patients develop their symptoms at rest rather than with activity. Originally, these patients were thought to have completely benign disease caused by coronary vasospasm, but now there are some concerns that it is caused by injury to the endothelium of the blood vessel. These patients are treated with calcium-channel blockers, which raises the threshold for coronary artery vasospasm to occur.

Which of the following is the long-term treatment of choice for patients with chronic atrial fibrillation who are older than age 60 and who have underlying coronary heart disease? A. Aspirin B. Warfarin (Coumadin) C. Low-molecular-weight heparin D. Unfractionated heparin E. Clopidogrel (Plavix)

The correct answer is B. Patients with chronic atrial fibrillation along with some other cardiovascular risk factors should be treated with warfarin as the most effective long-term medication listed in the choices. Patients with nonvalvular atrial fibrillation can also be treated with direct oral anticoagulants.

Patients with secondary hypertension may have notable findings on physical examination. In addition to having an audible renal artery bruit, which of the following signs or symptoms is most likely to be seen in a patient with renovascular stenosis secondary to renal artery stenosis? A. Narrow BUN-to-creatinine ratio B. Hypokalemia C. Hyperglycemia D. Abdominal striae E. "E" sign or "3" sign seen on chest x-ray

The correct answer is B. Patients with renovascular hypertension develop hypokalemia secondary to activation of the renin-aldosterone system, which results in retention of sodium and loss of potassium.

A patient is seen in the office as follow up to an emergency medicine visit. The patient complained of chest pain with exertion, and he was referred to cardiology where he was diagnosed with syndrome X. Which of the following findings would be consistent with this conclusion on tests performed by cardiology? A. Spasm of the left anterior descending artery B. Clean (no plaque) of coronary arteries during cardiac cath C. Depressed left ventricular function during echocardiogram D. No myocardial ischemia seen during nuclear imaging E. Enlarged aortic root seen on echocardiogram

The correct answer is B. Patients with syndrome X have clear coronary arteries on cardiac catheterization, but they may have signs of ischemia on exercise testing or nuclear imaging. Patients have symptoms associated with ischemic chest pain, but there is no underlying significant coronary artery plaque. This is what constitutes syndrome X in cardiology.

A patient who sustained a STEMI is seen in the office following discharge. The patient has questions about one of the discharge medications, prasugrel (Effient). To which of the following classes of medications does this agent belong? A. Glycoprotein IIb/IIIa inhibitor B. Antiplatelet C. Low-molecular-weight heparin D. Vasodilator E. Nitrate

The correct answer is B. Prasugrel, clopidogrel, and ticagrelor are antiplatelet agents used post-MI, with prasugrel being used after percutaneous coronary intervention in order to keep the stent patent. These agents are further classified as P2Y12 antagonists, which reduce platelet activation and aggregation.

Health care providers need to have working knowledge of cardiac physiology. Pulmonary hypertension would be a major risk factor for the development of which of the following valvular heart disease types? A. Tricuspid stenosis B. Tricuspid regurgitation C. Pulmonary stenosis D. Mitral stenosis E. Mitral regurgitation

The correct answer is B. Pulmonary hypertension causes right ventricular dilation with resultant tricuspid regurgitation.

A patient has complaints related to fast heart beats. The patient has an in-office EKG performed, which reveals a short PR interval along with a delta wave. Which of the following is the treatment of choice for this patient? A. Immediate cardioversion B. Radiofrequency catheter ablation C. Digoxin (Lanoxin) D. Verapamil (Calan) E. Metoprolol (Lopressor)

The correct answer is B. Radiofrequency ablation is the definitive treatment for ablating Wolff-Parkinson-White (WPW) syndrome's accessory pathway in the bundle of Kent. Patients with WPW have an abnormal accessory pathway that leads to pre-excitement of the ventricle. As the conduction travels down this accessory pathway into the left ventricle, there is a slurred uptake to the R wave, which is known as a delta wave. Radiofrequency ablation has nearly a 100% success rate with less than a 1% in-hospital mortality. Treatment is performed for symptomatic patients with WPW or those patients with WPW who are at high risk for the development of sudden cardiac death.

Ranolazine (Ranexa) is an agent used in the management of patients with chronic angina who continue to have symptoms despite using traditional medical therapy. Which of the following is an adverse effect of this medication? A. Impaired renal function B. Prolongation of QT interval C. Pulmonary fibrosis D. Bronchospasm E. Hypothyroidism

The correct answer is B. Ranolazine can prolong the QT interval and is contraindicated in the setting of acute hepatitis and cirrhosis.

A patient is seen in the office for a routine appointment. The patient has a history of essential hypertension for the last 30 years, which has had variable control of the blood pressure. Which of the following auscultatory findings is most commonly seen in the setting of long-standing hypertension? A. S3 gallop B. S4 gallop C. Pronounced S1 D. Holosystolic murmur heard right second intercostal space E. Systolic murmur radiating into the carotids

The correct answer is B. S4 gallop is most commonly heard in setting of long-standing hypertension. This occurs from atrial contraction against a hypertrophied left ventricle.

A patient is seen in the office with complaints of claudication that occurs with ambulation. He notes the distance that he travels prior to developing the pain in his right calf. Which of the following risk factors is considered to be the most important risk factor involved with the development of this condition? A. Diabetes B. Smoking C. Hypertension D. Hyperlipidemia E. Sedentary lifestyle

The correct answer is B. Smoking is the most important risk factor for the development of peripheral vascular disease. Smoking is directly toxic to the vasculature and can also cause vasoconstriction of these vessels.

A 47-year-old woman with a history of rheumatic fever is examined by her health care provider. Physical examination is significant for a low-pitched, rumbling, diastolic murmur preceded by an opening snap. This murmur radiates into the apex of the heart. The affected valve can be best evaluated by auscultation at which of the following locations? A. Left second intercostal space B. Left fifth intercostal space C. Left lower sternal body border D. Right second intercostal space E. Right fifth intercostal space

The correct answer is B. Solid knowledge of cardiac anatomy and its clinical correlations are crucial for performing physical examinations. This question tests two facts. First it requires that you recognize the patient has mitral stenosis. Classic clues to this diagnosis are "low-pitched, rumbling, diastolic murmur," and "opening snap." The mitral valve is the most commonly affected valve in rheumatic fever, followed by the aortic and tricuspid valves. The mitral valve is most audible over the left fifth intercostal space at the midclavicular line.

The CHADS-VASc scoring system is used to determine whether anticoagulation is needed in the setting of atrial fibrillation. Which of the following categories is assigned 2 points instead of 1? A. Female gender B. Age 75 or higher C. Chronic heart failure D. Hypertension E. Diabetes

The correct answer is B. The CHADS-VASc scoring system awards 2 points for age 75 or higher, as well as for previous stroke or transient ischemic attack. The higher the number of points in this system, the greater the need for anticoagulation to be given on a long-term basis, as the benefits outweigh the risks.

On a routine physical examination, a midsystolic ejection murmur is detected in the pulmonic area of a 35-year-old woman. The cardiac examination also reveals a prominent right ventricular cardiac impulse and wide and fixed splitting of the second heart sound. EKG shows right axis deviation and chest radiograph shows enlargement of the right ventricle and atrium. Which of the following is the most likely diagnosis? A. Aortic stenosis B. Atrial septal defect C. Mitral regurgitation D. Mitral stenosis E. Pulmonary valve stenosis

The correct answer is B. The classic findings in atrial septal defect are a prominent right ventricular cardiac impulse, a systolic ejection murmur heard in the pulmonic area and along the left sternal border, and fixed splitting of the second heart sound. These findings are caused by an abnormal left-to-right shunt through the defect, creating a volume overload on the right side. The increase in volume on the right side creates the flow murmur, the dilatation of the right-sided chambers, and the delayed closure of the pulmonic valve, all of which are present in this case. The delayed closure of the pulmonic valve occurs because of the increased volume of blood that is delivered to the right ventricle.

For patients with heart failure who cannot tolerate an ACE inhibitor or ARB, hydralazine may be used as a substitute. When hydralazine is used in this setting, which additional agent is it classically paired with? A. Furosemide (Lasix) B. Isosorbide dinatrate (Isordil) C. Spironolactone (Aldactone) D. Metoprolol (Lopressor) E. Carvedilol (Coreg)

The correct answer is B. The combined use of hydralazine and nitrate therapy is to reduce cardiac preload and afterload by achieving both venous and arterial vasodilation. Hydralazine is an arterial vasodilator, and nitrates are predominantly venodilators. These vasodilatory effects can reduce intracardiac filling pressures, which may reduce adverse cardiac remodeling.

A 41-year-old man comes to his health care provider for a routine physical examination. He is new to this office and brings his previous medical record with him. He has no significant past medical history, but he does have a strong family history of cancer and heart disease. His father and his brother both had myocardial infarctions before age 55 years, and his sister, mother, and aunt had breast cancer. He exercises regularly and eats well, with most of his diet being low in saturated fat and cholesterol. He smokes one pack of cigarettes per week. His review of systems is unremarkable. He is very anxious and would like only minimal interventions done because of his good health. Which of the following is the most age-appropriate screening test in this patient? A. Fasting lipid profile B. Non-fasting total cholesterol level C. Oral glucose tolerance test D. Prostate examination E. Sigmoidoscopy

The correct answer is B. The current recommendations for routine, age-appropriate screening are based in some measure on data from clinical trials. Depending on the source of the recommendations, there is considerable variability in these recommendations. One current recommendation is that at least every 5 years a random cholesterol level should be checked. Typically, total cholesterol and HDL cholesterol can be drawn as a baseline in a nonfasting state. If abnormalities are found in these measurements, a fasting blood lipid (ideally a 12-hour fast) can be performed.

A patient sustains a myocardial contusion in a motor vehicle collision, during which time the patient's chest struck the steering wheel. The patient has chest pain and quickly develops jugular venous distention along with shortness of breath. Which of the following is the primary pathologic process associated with cardiac tamponade? A. Impaired cardiac outflow obstruction B. Impaired cardiac filling during diastole C. Cor pulmonale D. Increased venous capacitance in the lungs E. Systemic hypovolemia

The correct answer is B. The problem with cardiac tamponade is that there is impaired ventricular filling because of accumulation of pericardial fluid or compression by the lungs impairing cardiac filling. When the heart is compressed by fluid or increased pulmonary pressures, a cardiac tamponade will compress and increase pressure within the heart, which results in impaired filling of the heart.

A 59-year-old man who is scheduled for an abdominal aortic aneurysm (AAA) repair in 3 weeks presents to the health care provider's office. The patient's AAA was diagnosed last week via ultrasound. Imaging at that time revealed a 5.5-cm aneurysm of his abdominal aorta extending bilaterally into his iliac arteries. The patient also has moderate hypertension, with a mean daily blood pressure of 150/95 mm Hg. On physical examination, the patient appears in no distress. He weighs 274 pounds and is 5 feet 9 inches tall (BMI is 40.5). His lungs are clear, and he has a loud S4. His AAA is palpable as a pulsatile mass in his abdomen. He complains of very mild back pain. Which of the following is the most appropriate intervention to prepare this patient for surgery? A. Arrange long-term physical therapy B. Improve blood pressure control C. Initiate a weight loss program D. Prescribe a nonsteroidal anti-inflammatory drug (NSAID) E. Prescribe a regimen of regular aerobic exercise

The correct answer is B. The most important interventions involve lessening the chance for rupture of an abdominal aortic aneurysm (AAA). Other interventions include counseling for smoking cessation and improving blood pressure control.

A 45-year-old woman has a history of symptomatic ventricular couplets, for which she had been placed on amiodarone (Cordarone). A week after this intervention, she experiences a syncopal episode and is brought to the emergency department. The patient is awake and alert with a blood pressure of 110/70 mm Hg and pulse 90/min. Electrolytes are within normal limits. She is placed on a cardiac monitor and is noted to have a transient wide complex tachycardia. Which of the following is the most appropriate next step? A. Check the level of serum calcium B. Discontinue amiodarone (Cordarone) and observe C. Increase the amiodarone (Cordarone) dose D. Administer IV potassium E. Apply 360 J of defibrillating current

The correct answer is B. The patient is diagnosed with wide complex tachycardia because of the effects of amiodarone. This wide complex tachycardia leads to prolonged QT syndrome, which places the patient at risk for polymorphic ventricular tachycardia called torsades de pointes. This can be a fatal rhythm. Amiodarone is a Vaughn Williams Class III antiarrythmic that inhibits alpha and beta adrenergic stimulation and affects the calium, potassium, and sodium channels. Although this medication is an antiarrhythmic, it also can be pro-arrhtyhmic. The appropriate step is thus to discontinue the amiodarone and observe.

Which of the following is the recommended screening test for asymptomatic patients who are being assessed for hyperlipidemia? A. Serum triglyceride alone B. Total cholesterol and HDL cholesterol C. Fasting total cholesterol alone D. Fasting LDL E. Fasting total cholesterol, LDL cholesterol, HDL cholesterol, and triglyceride levels

The correct answer is B. The recommended screening test for hyperlipidemia is initially total cholesterol and HDL cholesterol. The most recent screening guidelines do not require that this test be done in a fasting state. Full lipid profiles can be repeated if the initial screening test is abnormal.

Although not a common entity, patients can develop tumors located in the heart. Which of the following is considered to be the treatment of choice for a patient with atrial myxoma? A. Antibiotic coverage during appropriate surgical interventions B. Excision C. Long-term anticoagulation D. Thrombolytic therapy to dissolve E. Placement of permanent pacemaker

The correct answer is B. The treatment of atrial myxoma is surgical excision. Atrial myxoma tumors can be a source of embolism, which can lead to stroke or acute arterial occlusion.

A patient is seen in the office for routine physical examination. During auscultation of the heart, the patient is noted to have increased splitting of S2. This splitting increases even more with inspiration. The patient is most likely to have which of the following conditions? A. Left bundle branch block B. Right bundle branch block C. Atrial septal defect D. Aortic stenosis E. Aortic regurgitation

The correct answer is B. There is increased S2 splitting with inspiration with right bundle branch block, pulmonic stenosis, and mitral regurgitation. When a patient inspires, increased negative intrathoracic pressure will cause more blood going into the right side of the heart. More blood on the right side of the heart will cause further delay in closure of the pulmonic valve, lengthening the split between A2 and P2.

A 25-year-old man comes to the health care provider with chest pain at rest that is not always related to exercise. He reports two prior episodes of fainting during exercise but has otherwise been healthy. He reports a similar history in other family members and notes that his father suddenly collapsed and died at age 50 after playing tennis. He denies use of cocaine or other recreational drugs and does not take any medications. Physical examination reveals a systolic ejection murmur that is loudest along the left sternal border. The rest of the physical examination is unremarkable. Echocardiography shows asymmetric septal hypertrophy without obstruction. Which of the following interventions would be most likely to decrease this patient's systolic murmur? A. Inhaling amyl nitrate B. Lying down C. Standing up D. Use of digoxin (Lanoxin) E. Valsalva maneuver

The correct answer is B. This patient has hypertrophic obstructive cardiomyopathy. This is a cardiac condition in which the interventricular septum hypertrophies excessively and asymmetrically. The hypertrophied septum and the anterior leaflet of the mitral valve produce left ventricular outflow obstruction. Most cases are inherited through an autosomal-dominant mode of transmission. The systolic ejection murmur is diminished when the patient lies down, because this maneuver increases cardiac size by increasing venous return and tends to diminish the intensity of the murmur. Afterload is increased, and venous return is increased to the heart. This increases the ventricular size and diminishes the murmur.

A patient is evaluated after developing ventricular fibrillation (VF) 3 weeks post-MI. Which of the following is the most likely underlying cause for VF to occur in this setting? A. Underlying aortic stenosis B. PVCs and other ectopic beats C. Development of ventricular septal defect D. Systolic pump failure E. Diastolic ventricular failure

The correct answer is D. Systolic pump failure should be suspected in the setting of a post-MI patient who develops VF several weeks after the event. Systolic pump failure occurs in the setting of anterior wall MI, and patients with the most impaired left ventricles are most likely to go on to develop VF.

A 38-year-old English professor complains of chest pain with exertion. The symptoms began approximately 4 months ago and have been accompanied by dyspnea. On several occasions, she became lightheaded and felt faint while exercising. She has no prior medical history. She does not smoke or drink. On physical examination, her blood pressure is 154/92 mm Hg, pulse is 82/min, and respirations are 16/min. She is afebrile. A head and neck examination is normal. There is no jugular venous distention. Her lungs are clear. Her carotid upstrokes are delayed and diminished in amplitude. On cardiac examination, there is a forceful apical impulse and a soft S2. There is a harsh, late-peaking, crescendo-decrescendo systolic murmur that is heard best at the right second intercostal space and radiates to the carotids. The remainder of the physical examination is unremarkable. An electrocardiogram reveals a normal sinus rhythm and left ventricular hypertrophy by voltage criteria. Which of the following is the most likely diagnosis? A. Aortic insufficiency B. Aortic stenosis C. Ischemia of the left anterior descending artery D. Ischemia of the left main coronary artery E. Mitral regurgitation

The correct answer is B. This patient's symptoms are suggestive of aortic stenosis. Her ECG does not reveal changes consistent with coronary artery ischemia making her symptoms most likely due to aortic stenosis. Aortic stenosis is associated with systolic murmur, with a crescendo-decrescendo pattern that peaks in midsystole and radiates into the carotids. In this patient's age group, aortic stenosis is most likely to result from a congenital bicuspid aortic valve or, less likely, as a sequelae of rheumatic valvular disease. The anginal-type symptoms develop in the presence of normal coronary arteries because there is increased oxygen demand by the hypertrophied left ventricle, which exceeds the coronary arteries' ability to meet demands.

Patients with congenital heart disease are now living into adulthood, and there are many types of complications that can occur as a result of these congenital heart defects. Which of the following valvular abnormalities is likely to occur as a result of a large ventricular septal defect that has not been repaired? A. Aortic stenosis B. Aortic regurgitation C. Mitral stenosis D. Pulmonic stenosis E. Atrial septal defect

The correct answer is B. Unrepaired large ventricular septal defect can lead to development and progression of aortic regurgitation because of increased heart volume and dilation of the left ventricle.

Valvuloplasty is indicated for mitral valve stenosis in which of the following scenarios? A. With a valve area of 4 cm2 B. Only after the patient has symptoms C. With a NYHA classification of Stage 1 D. With a left atrial pressure of 10 mm Hg E. Patient with history of rheumatic fever

The correct answer is B. Valvotomy is indicated in the symptomatic patient who has a mitral orifice less than 1.0 cm2/m2 BSA (body surface area).

Health care providers taking care of patients who are suspected of having acute deep venous thrombosis (DVT) need to have an understanding of the sensitivity and specificity of the tests that are used as part of this clinical assessment. These various diagnostic studies also have different sensitivities and specificities depending upon the suspected site for this occlusion. Which of the following diagnostic studies has the highest sensitivity for identifying DVT located in the calves? A. Doppler analysis and duplex ultrasound B. Venography C. Impedance plethysmography D. PET scan E. D-dimer

The correct answer is B. Venography is best at identifying distal (calf) DVT. Although not used as frequently as a diagnostic procedure, venography is still considered the gold standard in identifying DVT in the calf. This procedure is more sensitive and specific for calf DVT than any other modality. One complication of this procedure, however, is that it can actually cause phlebitis, which could lead to DVT.

A patient with a history of chronic heart failure is seen in the office. This patient has not felt well and has had poor oral intake over the last several days. Which of the following effects may occur if a volume-depleted patient with chronic heart failure is prescribed an ACE inhibitor? A. Worsening hyperkalemia B. Hypotension C. Acute liver failure D. Change in color vision E. Peripheral edema from the vasodilation

The correct answer is B. Volume-depleted patients with chronic heart failure who are given an ACE inhibitor will develop hypotension. ACE inhibitors cause vasodilation, making them useful to treat hypertension. Without sufficient volume support, however, patients who are already volume depleted will become significantly hypotensive.

A patient is assessed with new-onset hemoptysis. The patient undergoes a clinical evaluation, and the patient's hemoptysis is caused by a heart valve abnormality. Which of the following heart abnormalities is most likely to lead to the development of hemoptysis? A. Subacute bacterial endocarditis B. Acute bacterial endocarditis C. Mitral stenosis D. Mitral regurgitation E. Aortic stenosis

The correct answer is C. Mitral stenosis leads to hemoptysis because of increased pulmonary pressure leading to rupturing of small bronchial veins. Mitral stenosis leads to a pressure elevation in the pulmonary system leading to the damage to the pulmonary vessels and causes the highest intrapulmonary pressures when compared to all of the other heart valve abnormalities.

A patient is seen in the office with long-standing, asymptomatic mitral stenosis. The patient has recently developed atrial fibrillation (AF). Evaluation of the patient reveals favorable valve morphology without left atrial clot. There is no accompanying mitral regurgitation. Which of the following is the preferred management of this patient? A. Transcatheter mitral valve repair B. Mitral valve replacement C. Percutaneous mitral balloon commissurotomy D. Observation alone

The correct answer is C. Mitral valve commissurotomy is indicated for an asymptomatic patient with new-onset AF, very severe mitral stenosis, or stage D symptomatic patients with mitral stenosis. This procedure has known benefit in this situation and is less invasive than the other procedures that can be done to the mitral valve.

A patient has developed cardiogenic shock. Examination reveals that this patient has pulsus paradoxus, and he also has distended neck veins along with visible jugular venous distention during heart beats. Which of the following is the most appropriate therapy for this patient? A. Loop diuretics B. Morphine sulfate C. Pericardiocentesis D. Needle decompression in the second intercostal space E. Emergent open heart surgery to repair the ruptured myocardium

The correct answer is C. Patients with pulsus paradoxus and distended neck veins have cardiac tamponade. Cardiac tamponade is treated with pericardiocentesis or creation of a pericardial window.

A patient has a STEMI, which is treated with thrombolytic therapy. Two weeks after the myocardial infarction, the patient complains of chest pain that mimics his initial chest pain with his myocardial infarction. Dressler syndrome is suspected. Which of the following maneuvers is best at identifying a pericardial friction rub? A. Patient taking a deep breath B. Patient leaning on left lateral decubitus position C. Patient sitting up D. Placing the patient on an inversion table E. Listening over the carotids rather than over the chest

The correct answer is C. Pericardial friction rubs are heard best with the patient sitting up and leaning forward. Patients can sit forward with their elbows on their knees as the maneuver of choice to identify pericardial friction rubs because this position will cause maximum contact between the visceral and parietal pleura. Pericardial friction rubs are heard best with the diaphragm of the stethoscope pressing against the chest wall. It is heard best at the left sternal border and is described as scratchy in quality. It is consistent with the patient having pericarditis.

A 57-year-old man comes to his health care provider for a preoperative evaluation. He has been a long-time patient in this office and has been treated for hypertension and gastritis. He has been scheduled for an elective cholecystectomy in 2 days because of ongoing gallbladder symptoms. He currently takes omeprazole (Prilosec) for his gastritis and thiazide for his hypertension daily. He smokes two packs of cigarettes per day. His home blood pressure log shows that his systolic pressures range from 150 to 190 mm Hg, and his diastolic pressures range from 80 to 105 mm Hg, indicating that his blood pressure may be not adequately controlled for the surgical procedure. Which of the following medications is most appropriate in the perioperative period for added blood pressure control? A. Captopril (Capoten) B. Clonidine (Catapres) C. Metoprolol (Lopressor) D. Nifedipine (Adalat, Procardia) E. Prazosin (Minipress)

The correct answer is C. There is an extensive body of literature indicating that beta-blockers such as metoprolol given to non-cardiac surgical patients who are at risk for cardiac events are associated with a more favorable outcome in terms of postoperative cardiovascular morbidity and mortality. This patient has somewhat poorly controlled hypertension, as well as at least three cardiovascular risk factors (hypertension, tobacco, age). Ideally one would like to have better control of the blood pressure and to reduce any risk for adverse perioperative events before the patient has elective surgery. Beta-blockers can achieve both of these endpoints. These agents are available parenterally and can be given on a dose titration basis. Although the most recent blood pressure guidelines do not list beta-blockers as first-line agents for blood pressure control, these agents may have utility in the perioperative setting because of their cardiac morbidity benefit.

For patients hospitalized with hypovolemic shock, which of the following therapies is most commonly used to monitor for the effectiveness of treatment? A. Cardiac monitor to follow the patient's heart rate B. Pulse oximeter to monitor the systemic peripheral delivery of blood C. Foley catheter to measure urinary output D. Mental status examination to assess for cerebral perfusion E. Auscultation of lungs to assess for fluid overload

The correct answer is C. A Foley catheter should be placed in order to monitor urinary output for treatment response to hypovolemic shock. Urinary output is directly related to renal perfusion, so improved hydration status in the setting of hypovolemic shock will result in increased urinary output. The therapeutic goal for urinary output is at least 30 ml per hour.

A 2-month-old baby boy is brought to the clinic by his mother for a routine checkup. On physical examination, the health care provider hears a continuous machinery murmur near the left sternal border at the second interspace. What is the most likely diagnosis? A. Coarctation of the aorta B. Ventricular septal defect C. Patent ductus arteriosus D. Atrial septal defect E. Pulmonary stenosis

The correct answer is C. A continuous machinery murmur is characteristic of patent ductus arteriosus (PDA). PDA is also characterized by bounding pulses with wide pulse pressure.

A 25-year-old woman is discovered on physical examination to have a midsystolic click and a high-pitched heart murmur. Which of the following additional cardiovascular findings is she most likely to exhibit? A. Atrial fibrillation B. Decreased peripheral pulse pressure C. Premature ventricular contractions D. Slowed carotid upstroke E. Wide pulse pressure

The correct answer is C. A midsystolic click and a high-pitched heart murmur in a young woman is the classic presentation of mitral valve prolapse. Patients are usually asymptomatic but may have dyspnea, tachycardia, chest pain, syncope, eventual congestive heart failure, or, rarely, sudden death. Prolapse may coincide with tricuspid or pulmonary valve disease or with psychiatric conditions such as anxiety or depression. Complications may include atrial thrombosis, calcification, infective endocarditis, emboli to the brain, rupture of chordae, mitral regurgitation, arrhythmias, and premature ventricular contractions (PVCs).

What is the potential side effect that can occur when patient with chronic heart failure is treated with both ACE inhibitors and aldosterone-receptor blockers (ARBs)? A. Hyponatremia B. Hypocalcemia C. Hyperkalemia D. Low serum uric acid levels E. Hypoalbuminemia

The correct answer is C. ACE inhibitors and ARBs can each individually cause hyperkalemia through their effect on dropping glomerular filtration pressures. There is no benefit from using these agents together, and their use may cause hyperkalemia to occur more rapidly and more significantly. Although there are theoretic benefits to utilizing these agents together in the management of chronic heart failure, clinical trials do not support their combined use, and this practice has been largely abandoned.

Secondary prevention for coronary artery disease (CAD) involves multiple medications along with lifestyle modification. Which of the following medications used in this setting has its indication for patients with left ventricular ejection fraction being less than 40%? A. Aspirin B. PY212 platelet-receptor antagonist C. ACE inhibitor D. Beta-blocker E. Aldosterone antagonist

The correct answer is C. ACE inhibitors or angiotensin-receptor blockers (ARBs) are indicated for secondary CAD prevention when the left ventricle is failing: the more impaired the left ventricle, the higher the benefit for ACE inhibitors.

When explaining to patients the importance of limiting the amount of excess adipose fat, the clinician should explain that for patients with metabolic syndrome, excess adipose tissue leads to which of the following physiologic processes? A. Gluconeogenesis B. Triglyceride production C. Production of proinflammatory cytokines D. Increased estrogen clearance E. Endothelial hyperplasia

The correct answer is C. Adipose deposition leads to production of proinflammatory cytokines. This fatty tissue is not metabolically inert and accelerates the process of coronary artery disease and atherosclerosis.

Patients who survive STEMI may be treated with aldosterone antagonists such as spironolactone (Aldactone) and eplerenone (Inspra). Which of the following is a contraindication to their use? A. Acute hepatitis B. Psoriasis C. Renal failure D. Unstable angina E. Acute heart failure

The correct answer is C. Aldosterone blockers cannot be used with chronic renal failure or hyperkalemia. They are also known as potassium-sparing diuretics that will cause retention of potassium. Potassium retention also occurs in the setting of renal failure, so these agents would further worsen potassium levels.

A patient is seen in the hospital following acute STEMI. The patient is having bouts of ventricular tachycardia with a pulse, and the patient is otherwise stable. Which of the following clinical interventions is recommended for this patient? A. Immediate cardioversion B. Immediate defibrillation C. Amiodarone D. Epinephrine E. No treatment is necessary

The correct answer is C. Amiodarone is indicated for stable VT after MI. Following MI, the heart is irritable, and it is common for ventricular tachycardia to occur in this setting. When ventricular tachycardia occurs within 48 hours following acute MI, there is no associated poor prognosis.

A patient is seen in the office with complaints of fatigue, right upper quadrant fullness, and peripheral edema. Echocardiogram reveals bilateral biatrial enlargement and increased ventricular volume that has a "speckled" pattern. Systolic function is normal, but diastolic pressures are increased. Which of the following is the most likely diagnosis causing this presentation? A. Coxsackie B cardiomyopathy B. Hypertrophic cardiomyopathy C. Amyloidosis D. Alcohol-related cardiomyopathy E. Amiodarone-related cardiomyopathy

The correct answer is C. Amyloidosis causes infiltrative disease of the heart. The deposition of this abnormal starch-like substance in the cardiac myocytes causes a speckled pattern to occur on echocardiogram. These patients will have normal systolic function, increased diastolic pressures, and right-sided heart failure symptoms.

Patients with angina who undergo percutaneous coronary intervention (PCI) with drug-eluting stent implantation have increased vessel patency with the use of which of the following medications? A. Beta-blockers B. Calcium-channel blockers C. Antiplatelet agents D. Novel oral anticoagulants E. Low-molecular-weight heparin

The correct answer is C. Antiplatelet agents such as aspirin and P2Y12 receptor inhibitors are used to maintain stent placement following PCI.

A 55-year-old man with hypertension and a past medical history of myocardial infarction is prescribed atenolol (Tenormin). This medication will lower his blood pressure by A. centrally acting agents B. blocking the conversion of angiotensin I to angiotensin II C. decreasing cardiac output D. decreasing intravascular volume E. increasing renin release from the kidney

The correct answer is C. Atenolol is a beta-adrenergic receptor blocking agent used in the treatment of hypertension. Medications in this drug class lower blood pressure by reducing both cardiac output and decreasing renin release from the kidney (to a lesser extent). Beta blockers decrease heart rate and lessen cardiac contractility, which results in lowering of myocardial oxygen demand.

Which of the following dysrhythmias is most likely to occur in the setting of long-standing constrictive pericarditis? A. First-degree heart block B. Second-degree heart block type 2 C. Atrial fibrillation D. Atrial flutter E. Multifocal atrial tachycardia

The correct answer is C. Atrial fibrillation is seen as a long-standing complication of constrictive pericarditis. Constrictive pericarditis occurs as a result of scarring in the pericardium. This long-standing scarring results in increased atrial pressures with subsequent development of atrial fibrillation. Because of the fluid accumulating between the pericardial membranes, the EKG will also show low voltage.

Patients taking antihypertensive medications on a long-term basis should have an assessment of their clinical laboratory and physical examination effects. Which of the following complications is associated with the use of angiotensin-converting enzyme (ACE) inhibitors? A. Coronary artery vasospasm B. Hypercalcemia C. Hyperkalemia D. Peripheral edema E. Progression of renal failure in diabetic patients

The correct answer is C. Because of the lowering of glomerular filtration pressure in the kidneys, less potassium is filtered out into the kidney, causing potassium to be retained. ACE inhibitors cause hyperkalemia, cough, skin rash, and altered sense of taste.

It is important to stratify the type of myocardial infarction between STEMI and non-STEMI because treatments are different. Which of the following is the primary difference in management between treatments of STEMI versus treatment of non-STEMI? A. PCI alone is indicated in the setting of non-STEMI. B. CABG is indicated only in the setting of STEMI. C. Fibrinolytics are only indicated in STEMI. D. ACE inhibitors are only used in the setting of STEMI. E. Anticoagulants are only used in the setting of non-STEMI.

The correct answer is C. Fibrinolytics are not used in the setting of non-STEMI, as their use can worsen the partial obstruction and make an incomplete into complete obstruction.

You are seeing a patient in your practice who has long-standing hypertension. She was recently referred from her obstetrician because of a positive pregnancy test. Which of the following antihypertensive agent classes is considered to be safe for use during pregnancy? A. ACE inhibitors B. ARBs C. Beta-blockers D. Calcium-channel blockers E. Thiazide diuretics

The correct answer is C. Beta-blockers and hydralazine are considered safe to treat hypertension during pregnancy. Because drug manufacturers are reluctant to test medications during pregnancy due to unknown fetal complications, there are not many clinical trials verifying safety for use in pregnancy. In addition to beta-blockers and the vasodilator hydralazine, methyldopa, a central alpha agonist, has been used for many decades to reduce blood pressure in the setting pf preeclampsia.

First-line monotherapies for essential hypertension (without any other compelling underlying conditions) include all of the following EXCEPT: A. ACE inhibitors B. ARBs C. Beta-blockers D. Long-acting calcium-channel blockers (dihydropyridine) E. Thiazide diuretics

The correct answer is C. Beta-blockers are considered second-line antihypertensive therapy unless there are associated heart problems. As antihypertensive agents, they are not as effective in lowering blood pressure as the other agents favored under the most recent JNC 8 guidelines.

Which of the following cardiac conditions is associated with a bisferious (or bisferiens) pulse (rapid increase in carotid pulse with two upstrokes)? A. Aortic stenosis B. Mitral stenosis C. Hypertrophic cardiomyopathy D. Ventricular septal defect E. Atrial septal defect

The correct answer is C. Bisferiens pulse is associated with hypertrophic cardiomyopathy (HCM) and aortic regurgitation. A bisferiens pulse is described as a "notched" pulse. The rapid upstroke and prominent percussion wave result from rapid left ventricular ejection into the aorta during early systole. This is followed by a rapid decline as left ventricular outflow tract obstruction ensues, a result of midsystolic obstruction and partial closure of the aortic valve. The second peak is related to the emptying of blood when the obstruction lessens.

A 23-year-old man sustains multiple stab wounds to his left chest. At the time of emergency department admission, his blood pressure is 70/45 mm Hg, pulse 110/min, and central venous pressure 4 mm Hg. He is diaphoretic and anxious, and he is notably pale. It is quickly determined that he has a left hemopneumothorax and a chest tube is inserted. A total of 380 mL of blood is recovered initially, and another 120 mL is suctioned during the next hour. He is given 2 liters of Ringer's lactate, followed by 2 units of blood. Reassessment at the end of the first hour shows that his lung is expanded and his central venous pressure has gone up to 22 mm Hg, but his blood pressure is only 85/70 mm Hg, his pulse has gone up to 115/min, and he remains diaphoretic. Which of the following is the most likely diagnosis at this time? A. Continued bleeding B. Intrinsic cardiogenic shock C. Pericardial tamponade D. Tension pneumothorax E. Vasomotor shock

The correct answer is C. Cardiac/pericardial tamponade is caused by the accumulation of blood within the pericardial space, creating an increase in the intrapericardial pressure. This in turn results in restricted cardiac filling with a resultant decrease in cardiac output. Shock with high central venous pressure in a chest trauma victim suggests either pericardial tamponade or tension pneumothorax. The latter has been ruled out by the re-expanded lung. The problem was not initially obvious because of concomitant bleeding but became clear once blood volume was restored. Patients who have this condition will have hypotension, jugular venous distention, and distant heart sounds (known as Beck triad).

A patient is seen in the office. She has been diagnosed with type V familiar hyperlipidemia (elevated VLDL and chylomicrons). Which of the following is the pharmacologic treatment of choice if diet and exercise are NOT effective? A. Water-soluble statin B. Fat-soluble statin C. Gemfibrozil D. Cholestyramine (Questran) E. Garlic

The correct answer is C. Gemfibrozil belongs to the fibrate drug classification, which is used to treat both high triglyceride levels and high very-low-density lipoprotein (VLDL) and chylomicron levels. In addition to fibrate therapy, niacin may also be used to treat elevated VLDL and chylomicrons.

A 58-year-old man with history of atrial fibrillation is prescribed warfarin to prevent clot and embolism formation. His prothrombin time (PT) is regularly monitored. Administration of which of the following drugs would result in an increase in his PT and require readjustment of his warfarin dosage? A. Rifampin B. Carbamazepine (Tegretol) C. Ketoconazole D. Phenytoin (Dilantin)

The correct answer is C. Certain drugs can affect the metabolism of other drugs by either inducing or inhibiting hepatic microsomal enzyme cytochrome P450 activity. This means that the amount of circulating warfarin increased and that you are looking for a drug that inhibits the P-450 system (thereby decreasing warfarin metabolism). Of the drugs listed, only ketoconazole inhibits the P-450 system. Other important inhibitors include cimetidine (Tagamet) and isoniazid (INH).

A patient is seen in the office for an acute care visit after he underwent an arteriogram earlier in the day. He was told that he has significant atherosclerosis in his abdominal aorta. He is having a great deal of pain in his abdomen and in his right lower extremity. Examination reveals blue/black toes. Which of the following is the most likely diagnosis? A. Leriche syndrome B. Acute arterial occlusion C. Cholesterol embolization syndrome D. Deep vein thrombosis E. Chronic venous insufficiency

The correct answer is C. Cholesterol embolism syndrome is seen most commonly following peripheral bypass procedures, which results in distal seeding of the plaques containing cholesterol. This condition is provoked by surgery or radiologic interventions. Patients with this condition complain of abdominal and peripheral pain, and physical examination shows blue/black toes.

A patient is seen who has a history of congenital heart disease. The patient developed secondary hypertension as a result of this congenital issue. Blood pressure is significantly higher in the upper extremity compared to the lower extremity. There is a "3" sign noted in chest film. Which of the following locations is most commonly involved with this condition? A. Root of aortic valve B. Ascending aorta C. Distal to the left subclavian artery D. Top of the aortic arch E. Distal aorta into the abdominal aorta

The correct answer is C. Coarctation of the aorta is most commonly located just distal to the left subclavian artery, as this is the take-off point for the ligamentum arteriosum. This location explains why affected patients have increased blood pressure in the upper compared to the lower extremities. This location is also the reason why there may be a differential in blood pressure between the left and right arms.

In addition to myocardial infarction, which of the following is the most common cause of cardiac related death due to uncontrolled hypertension? A. Aortic stenosis B. Atrial fibrillation C. Chronic heart failure D. Pulmonary hypertension E. Ventricular septal defect

The correct answer is C. Congestive heart failure (CHF) and cardiovascular disease (CAD) are the most common lethal cardiac complications of hypertension. Uncorrected hypertension leads to increased stress to the heart as the heart attempts to pump harder in order to overcome this increased systemic vascular resistance. Uncontrolled hypertension leads to accelerated atherosclerosis, stroke, renal disease, and peripheral vascular disease.

A midnight salivary cortisol level laboratory test is indicated when evaluating a patient for which of the secondary causes of hypertension? A. Pheochromocytoma B. Primary aldosteronism C. Cushing syndrome D. Coarctation of aorta E. Renovascular hypertension

The correct answer is C. Dexamethasone suppression test, or midnight salivary cortisol level, is used in the assessment for Cushing syndrome because elevated cortisol levels will result in retention of sodium and increased intravascular volume.

A patient is seen in the office with concerns regarding cardiovascular risk factors. He has hypertension, diabetes, low levels of high-density lipoprotein (HDL) cholesterol, and a positive family history for coronary artery disease. Of his risk factors, which of the following is considered to have the highest risk for the development of coronary artery disease (CAD)? A. Male sex B. Hypertension C. Diabetes D. Low HDL cholesterol E. Positive family history for coronary artery disease

The correct answer is C. Diabetes has the highest prediction as a risk factor for CAD. Patients with diabetes have such an increased risk for coronary artery disease that patients with diabetes are classified as having the equivalence of heart disease.

Not all EKG changes occur because of problems within the heart. Which of the following medications is most likely to lead to ST-segment depression on the EKG? A. Metoprolol (Lopressor) B. Diltiazem (Cardizem) C. Digoxin (Lanoxin) D. Spironolactone (Aldactone) E. Enalapril (Vasotec)

The correct answer is C. Digoxin can cause a strain pattern with ST-segment depression due to increased inotropic ventricular contraction. The leads with the tallest R waves will have the largest amount of strain pattern being produced.

A 52-year-old man comes to the hospital complaining of palpitations. The patient reports that while cooking breakfast this morning, he felt his heart "racing in his chest" and was unable to catch his breath. He states that sitting down brought no relief. He called for an ambulance and he was brought to the emergency department. The man has no significant past medical history and takes no medications regularly other than ranitidine (Zantac) for occasional heartburn. On examination, the patient is quite thin but well developed and in mild distress. His pulse is 140/min and irregularly irregular. There are no murmurs, and the lung examination is clear. Which of the following findings on his echocardiogram would suggest a diagnosis of long-standing atrial fibrillation? A. Dilated left ventricle B. Dilated right ventricle C. Enlarged left atrium D. Hypertrophied ventricular septum E. Pericardial thickening

The correct answer is C. Echocardiography is an invaluable tool for assessing cardiovascular function in both normal and disease states. In the case of atrial fibrillation (AF), a chronic course versus an acute and self-limited course portends completely different treatment strategies and long-term prognoses. The most common cause of chronic AF is valvular disease, followed by congestive heart failure (CHF). Hyperthyroidism that is not adequately treated is another cause of atrial fibrillation and should be suspected in a patient who has new-onset AF. The most common anatomic correlate seen in patients who have AF (Framingham Heart Study) is an enlarged left atrium. Patients who have an enlarged left atrium are less likely to remain in normal sinus rhythm following cardioversion.

Which of the following types of hypertension most commonly occurs in the elderly population? A. Renovascular hypertension B. Isolated diastolic hypertension C. Isolated systolic hypertension D. Combined systolic and diastolic hypertension E. Pseudohypertension

The correct answer is C. Elderly patients develop isolated systolic hypertension because of stiffening of the vessels that occurs with aging. The new guidelines allow a slightly higher systolic blood pressure in the elderly population.

Patients with chronic heart failure are treated with a variety of agents. To what class of medication does eplerenone (Inspra) belong? A. Inotrope B. Loop diuretic C. Aldosterone antagonist D. Combined alpha- and beta-blocker E. Vasodilator

The correct answer is C. Eplerenone is direct aldosterone antagonist and is also classified as a potassium-sparing diuretic. Its mechanism of action is to directly antagonize aldosterone-specific mineralocorticoid receptors, thereby decreasing sodium and water reabsorption with resultant potassium retention.

Which of the following medications is used as a provocative agent when assessing a patient for possible Prinzmetal angina during coronary angiography? A. Metoclopramide (Reglan) B. Methacholine C. Ergonovine D. Isoproterenol E. Albuterol

The correct answer is C. Ergonovine is used to try to provoke vasospasm when assessing for Prinzmetal angina. This agent can cause spasm of the coronary artery and recreate Prinzmetal angina symptoms.

A 59-year-old man with a history of myocardial infarction presents to his health care provider complaining of shortness of breath. On examination, his pulse is 110/min and his respiratory rate is 22/min. He has rales in both lung fields, a normal sinus rhythm with an S3 gallop, and 2+ pitting ankle edema. A chest radiograph reveals cardiomegaly, and his ejection fraction on echocardiogram is calculated at 37%. Which of the following medications would alleviate this patient's symptoms by significantly reducing both the preload and afterload on the heart without affecting its inotropic state? A. Digoxin (Lanoxin) B. Diltiazem (Cardizem, Cartia, Dilacor) C. Enalapril (Vasotec) D. Furosemide (Lasix) E. Propranolol (Inderal)

The correct answer is C. Everything in this case points to congestive heart failure (CHF): dyspnea, elevated heart rate, S3, peripheral edema, and reduced ejection fraction. A drug that will alleviate the symptoms by decreasing both preload and afterload is necessary. Enalapril is an angiotensin-converting enzyme (ACE) inhibitor. ACE inhibitors (captopril, enalapril, lisinopril, benazepril, ramipril) work by blocking the conversion of angiotensin I to angiotensin II in the lungs. Angiotensin II is both a potent vasoconstrictor and a stimulator of aldosterone production. Aldosterone acts by promoting sodium (and thus water) reabsorption by the kidney. An ACE inhibitor will therefore promote vasodilatation (reducing afterload), as well as reduce intravascular volume (decreasing aldosterone, as less renin is being released). ACE inhibitors are used in the setting of poor left ventricular function as they also decrease cardiac remodeling of the left ventricle.

A young man sustains a gunshot wound to the base of his neck. He was shot point blank with a .38-caliber revolver. The entrance wound is above the left clavicle, below the level of the cricoid cartilage, and just lateral to the sternomastoid muscle. The exit wound is just above the spinous process of the right scapula. He has normal breath sounds on both sides, is awake and alert, is talking with a normal tone of voice, is neurologically intact, and is hemodynamically stable. Portable radiographs of the neck and chest taken in the emergency department show some air in the tissues of the lower neck but are otherwise non-diagnostic. Which of the following is the most appropriate next step in management? A. Observation for several hours B. CT scan of the lower neck and upper chest C. Angiogram, esophagogram, esophagoscopy, and bronchoscopy prior to surgical exploration D. Immediate surgical exploration of the lower neck through a collar incision E. Immediate surgical exploration of the upper chest through a median sternotomy

The correct answer is C. Gunshot wounds to the base of the neck need exploratory surgery, but the exact approach and incision are determined by a more accurate knowledge of the location and extent of the injuries. If time permits, diagnostic studies thus should precede surgical intervention. The major vessels, the tracheobronchial tree, and the esophagus are the potential targets that have to be investigated. The esophagram needs to be performed with a water-based solution in case the patient has damage to the mediastinum, which would result in leakage from the esophagus and potential mediastinitis.

Hyperlipidemia may occur as a primary or secondary abnormality. If a patient is identified as having secondary hyperlipidemia, correction of underlying causes can manage this hyperlipidemia and even prevent medications from being used in this setting. Secondary causes of hyperlipidemia include all of the following EXCEPT: A. Cushing syndrome B. Diabetes mellitus C. Hyperthyroidism D. Hypothyroidism E. Nephrotic syndrome

The correct answer is C. Hyperthyroidism is not a known risk factor for hyperlipidemia, but its opposite condition, hypothyroidism (choice D) is a known cause. Hypothyroidism contributes to hyperlipidemia from the slowdown of the basal metabolic rate, which results in the retention of lipids rather than their elimination.

A 43-year-old woman comes to the health care provider with a several-month history of dyspnea on exertion. She denies chest pain or a family history of coronary artery disease. She has a history of rheumatoid arthritis. Chest radiograph is remarkable for calcification of the heart border on the lateral film. She appears to have constrictive pericarditis. Which of the following physical examination findings would most likely be expected in this patient? A. Central cyanosis B. Opening snap and diastolic rumble C. Pericardial knock D. Systolic ejection murmur E. Widely split S2

The correct answer is C. In constrictive pericarditis, the lateral view of the chest may demonstrate calcification of the anterior pericardium. This may be seen in 50% of patients who have longstanding constriction. The pericardial thickening may be seen on an echocardiogram. A pericardial knock is heard after the aortic valve closes, which is caused by the sudden cessation of ventricular filling.

Patients having dilated cardiomyopathy will have which of the following changes on the EKG? A. Persistent ST-segment elevation B. Increased voltage in the chest leads C. Low amplitude of the QRS complexes D. Widening of the P wave in lead II E. Widening of the QRS complexes in leads V5 and V6

The correct answer is C. In dilated cardiomyopathy the EKG often shows sinus tachycardia, atrial fibrillation, ventricular arrhythmias, left atrial abnormality, diffuse nonspecific ST-wave abnormalities, intravascular conduction defects, or low voltage. The low voltage is caused by the impaired cardiac function of the heart because of the cardiomyopathy. Because the EKG measures the electrical strength of contraction, dilated cardiomyopathy will show impairment of the strength of contraction as the cardiac myocytes become less functional.

For patients needing a left ventricular assist device in the setting of chronic heart failure, which of the following medications/medication classes is essential to use as long as this device is in operation? A. Digoxin (Lanoxin) B. Amiodarone (Cordarone) C. Anticoagulation D. Beta blocker E. Loop diuretic

The correct answer is C. Patients with a left ventricular assist device need to have anticoagulation as part of their intervention because these devices are very thrombotic. These patients need to be treated with anticoagulant and antiplatelet agents to reduce the risk of thrombotic complications such as device thrombosis and embolic stroke. Since these patients are also at risk for bleeding complications, anticoagulation should be carefully timed and titrated, and these agents should be started prior to these devices being utilized.

Which of the following treatments is typically given for a patient with chronic pulmonary hypertension? A. ACE inhibitors B. Beta-blockers C. Diuretics D. Corticosteroids E. Alpha-blockers

The correct answer is C. In the setting of pulmonary hypertension, diuretics are prescribed in order to treat peripheral edema, while supplemental O2 made be needed as the disease progresses. With progressive disease, patients may need to limit physical activities because of the exertional dyspnea that occurs.

Patients placed on lidocaine as an antiarrhythmic medication can develop which of the following adverse reactions to this medication? A. Renal failure B. Coronary artery vasospasm C. Seizures D. Pulmonary fibrosis E. Urinary retention

The correct answer is C. Lidocaine's more common adverse actions include confusion, seizures, and respiratory arrest.

Patients being treated in primary care may have underlying psychiatric disorders whose treatment can result in various complications. Which of the following psychotropic medication classes has hypertension as a known complication and can even lead to malignant hypertension? A. Selective serotonin reuptake inhibitors (SSRIs) B. Selective norepinephrine reuptake inhibitors (SNRIs) C. MAO inhibitors D. Typical antipsychotics E. Atypical antipsychotics

The correct answer is C. MAO inhibitors can cause hypertension. Hypertension crisis can occur if this medication is taken with Demerol or with certain combinations of foods that contain tyramine.

Which of the following cardiac findings may occur in the setting of chronic stable angina during an episode of ischemia? A. Aortic stenotic murmur B. Aortic regurgitant murmur C. Mitral regurgitant murmur D. Holosystolic murmur secondary to ventricular septal defect (VSD) E. Mitral valve prolapse click

The correct answer is C. Mitral regurgitant murmur occurs as a result of papillary muscle ischemia, which can occur in the setting of angina.

Which of the following heart sounds is most likely to occur in the setting of mitral regurgitation? A. S3 gallop B. S4 gallop C. Pronounced P2 D. Pronounced A2 E. Opening snap

The correct answer is C. Mitral regurgitation causes pronounced P2 because the right ventricle needs to contract with more force in order to move the increased blood volume that occurs in the setting of mitral regurgitation.

Which of the following conditions is most commonly associated with multifocal atrial tachycardia? A. Mitral stenosis B. Mitral regurgitation C. COPD D. Hypertrophic cardiomyopathy E. Atrial septal defect

The correct answer is C. Multifocal atrial tachycardia is most commonly caused by COPD. This condition has three or more different P wave morphologies and causes an irregular heart rhythm and rate.

A 55-year-old man comes to the emergency department complaining of chest pain that radiates to his left arm and shortness of breath for the past hour. Which of the following enzymes is the most sensitive for acute muscle damage? A. CK-MB B. LDH C. Myoglobin D. AST E. Troponin T

The correct answer is C. Myoglobin identifies any muscle damage but is so sensitive that it is not routinely used in the evaluation of myocardial infarction. It is heme protein that is mainly found in muscle. It rises in 2-3 hours and is rapidly cleared by the kidneys. It may be used as a "rule out" for patients presenting urgently to a health care facility because of its sensitivity; if myoglobin is normal, there is little chance that any significant muscle damage is present. Any muscle damage in the body will cause myoglobin levels to increase.

Which of the following lipid-lowering agents should NOT be used by diabetic patients because of its tendency to worsen glucose control? A. Fat-soluble statins B. Water-soluble statins C. Niacin D. Fibrates E. Bile acid-binding resins

The correct answer is C. Niacin worsens glucose control and should be avoided in diabetic patients.

Patients with lipid disorders may have an elevation in the "bad" type of cholesterol or a decrease in the "healthy" type of cholesterol. The agents that may lower the LDL cholesterol may not raise the HDL cholesterol. Which of the following anti-lipid agents are most effective in raising the HDL cholesterol? A. Fat-soluble statins B. Water-soluble statins C. Niacin D. Fibrates E. Bile acid-binding resins

The correct answer is C. Niacin, a B3 vitamin, is most effective at raising the HDL cholesterol. The major downside to using this agent is that high doses are needed in order to have an effect, and patients may not tolerate these high doses because of and itching, burning sensation on the skin, and its tendency to raise serum glucose levels.

Which of the following agents used in the management of unstable angina and non-STEMI (NSTEMI) should be avoided if a patient recently took a phosphodiesterase-5 inhibitor? A. Beta-blockers B. Calcium-channel blockers C. Nitrates D. ACE inhibitors E. Aspirin

The correct answer is C. Nitrates given after phosphodiesterase inhibitor use will cause significant hypotension. This combination of agents can also cause death.

Thiocyanate toxicity, exhibited as blurred vision, tinnitus, and delirium, is most likely to occur with the use of which of the following agents that may be used in the setting of hypertension with acute MI? A. Nitroglycerin B. Dobutamine C. Nitroprusside D. Dopamine E. Norepinephrine

The correct answer is C. Nitroprusside's metabolite is thiocyanate, which can lead to cyanide poisoning. Sodium nitroprusside is given in a drop-to-drop IV delivery system, and a large amount of medication is needed in order to lead to the development of thiocyanate toxicity.

Which of the following medication classes is indicated for managing vasospastic peripheral vascular disease such as Raynaud phenomena? A. Nitrates B. Beta-blockers C. Alpha-1 antagonists D. ACE inhibitors E. Vasodilators such as hydralazine (Apresoline)

The correct answer is C. Nondihydropyridine calcium-channel blockers and alpha-1 antagonists are preferentially used in the management of Raynaud phenomenon.

Which of the following drugs antagonizes both the vascular and cardiac actions of norepinephrine (NE)? A. Atenolol (Tenormin) B. Esmolol (Brevibloc) C. Labetalol (Trandate) D. Metaproterenol (Alupent) E. Prazosin (Minipress)

The correct answer is C. Norepinephrine (NE) is an agonist at alpha-1, alpha-2, and beta-1 receptors. NE exerts its vascular actions via alpha (predominantly alpha-1) receptors and its cardiac actions via beta-1 receptors. Labetalol is a selective antagonist at alpha receptors and nonselective antagonist at beta receptors and therefore could prevent all actions of NE.

Adult patients with congenital heart disease are increasingly seen in primary care offices as a result of improved survival with congenital heart disease. It is important that health care providers have working knowledge with regard to congenital heart disease as they will be evaluating adults with these conditions. Patients with patent ductus arteriosus (PDA) have an abnormal connection between the descending aorta and which of the following structures? A. Right atrium B. Right ventricle C. Pulmonary artery D. Left atrium E. Left ventricle

The correct answer is C. PDA connects the descending aorta and pulmonary artery with resultant overload to the left side of the heart. Patients who have this connection will have a machinery type murmur that is heard just below the left clavicle.

A patient is seen in the office with complaints of pain in her right calf. At her job, she stands for long periods of time. Examination reveals a tender, palpable venous cord, which she says is the area of tenderness. Which of the following is considered the most important element of treatment for a patient with this condition? A. Anticoagulation B. Antibiotics C. Analgesia D. Application of Unna boot E. Greenfield filter

The correct answer is C. Patients with superficial thrombophlebitis are not at increased risk for the development of deep vein thrombosis and subsequent pulmonary embolism, so treating the patient symptomatically is the treatment of choice.

Which of the following medications is only indicated for treatment of a patient with non-STEMI (NSTEMI) who undergoes percutaneous coronary intervention (PCI) as part of the management of this condition? A. Aspirin B. Clopidogrel (Plavix) C. Prasugrel (Effient) D. Ticagrelor (Brilinta) E. Bivalirudin (Angiomax)

The correct answer is C. Prasugrel, a PY212 receptor-antagonist, is only indicated when a patient with NSTEMI undergoes PCI.

Patients with various types of valvular heart disease will need advice with regard to whether antibiotic prophylaxis is needed prior to high-risk surgical procedures. Which of the following is the primary recommendation for prophylaxis against infective endocarditis (IE) in the setting of mitral valve prolapse (MVP)? A. Patient having co-existing mitral stenosis B. Patient having co-existing mitral regurgitation C. Patient having previous bout of endocarditis D. Patient using illicit IV drugs E. Patient having pulmonary hypertension

The correct answer is C. Previous endocarditis in the setting of MVP is the only indication for IE prophylaxis.

A 45-year-old woman comes to the emergency department complaining of recurring episodes of chest pain that each last a few minutes since she awoke this morning. She denies shortness of breath, radiation of the pain, or chest pain on exertion. She reports that she had similar episodes within the last month. The chest pain occurs at rest and not with activity. The patient states that she has been healthy otherwise, takes no medications, and has no family history of coronary artery disease. Her vital signs and physical examination are within normal limits. Electrocardiogram shows transient ST-segment elevation. Serial cardiac enzymes reveal no abnormalities over a 24-hour period. Cardiac catheterization shows no significant areas of plaque or stenosis. Which of the following classes of drugs may be most effective in this patient? A. ACE inhibitors B. Beta-blockers C. Calcium channel blockers D. Diuretics E. Alpha-adrenoceptor antagonists

The correct answer is C. Prinzmetal angina, or variant angina, is classically characterized by angina without the associated precipitating factors of angina pectoris. It is caused by a transient coronary artery spasm and displays a transient ST elevation on electrocardiogram. It usually has no other diagnostic findings. Calcium channel blockers have been shown to treat and prevent episodes of coronary vasospasm by decreasing contractility of cardiac muscle and producing vasodilation.

A 71-year-old retired physician complains of increasing calf pain when walking uphill. The symptoms have gradually increased over the past 2 months. His past medical history is significant for a transient ischemic attack 6 months ago. Over the past month, his blood pressure has worsened despite previous control with diltiazem, hydrochlorothiazide, and propranolol. His other medications are isosorbide dinitrate and aspirin. On physical examination, his blood pressure is 152/90 mm Hg and his pulse is 66/min. There is a right carotid bruit. His lower extremities are mildly cool and have diminished pulses at the dorsalis pedis. An ACE inhibitor is added to help control his blood pressure. Which of the following is the most appropriate in the management of his leg pain? A. Decrease diltiazem (Cardizem) dose B. Decrease hydrochlorothiazide dose C. Decrease propranolol (Inderal) dose D. Increase aspirin dose E. Increase isosorbide dinitrate (Isordil) dose

The correct answer is C. Propranolol, a beta-blocker, is considered to be relatively contraindicated in patients who have peripheral claudication. Although beta-blockers are excellent medications in patients who have hypertension, the beta blockade may predispose to arterial vasoconstriction peripherally. In addition, beta-blockers are negative inotropic agents that decrease the force of contraction of the heart. Because the heart is not contracting as strongly, there is less blood being propelled into the peripheral vascular system. The dose of beta-blockers in this patient should therefore be reduced or gradually eliminated.

A woman is brought into the emergency department following an automobile accident in which her chest was hit by the steering wheel. Her blood pressure is 120/90 mm Hg. When she inhales, her systolic blood pressure drops to 100 mm Hg. This finding defines which of the following terms? A. Pulsus alternans B. Pulsus bisferiens C. Pulsus paradoxus D. Pulsus parvus E. Pulsus tardus

The correct answer is C. Pulsus paradoxus is defined as a fall in systolic blood pressure >10 mm Hg on inspiration. It can be associated with cardiac tamponade and chronic obstructive pulmonary disease (COPD).

A 71-year-old man comes to his health care provider for follow-up of a recent emergency department visit. The patient has a 2-year history of mild congestive heart failure in the setting of longstanding hypertension. He reports that yesterday he sought care at the local emergency department for palpitations and shortness of breath. He was told that his heart was "fibrillating," but later the fibrillation had "stopped on its own." His medications include a thiazide diuretic and an ACE inhibitor. On physical examination he appears well and in no distress. His blood pressure is 130/80 mm Hg and pulse 100/min and irregular. His lungs have scant bibasilar rales and no gallops are appreciated. He has a grade 2 holosystolic murmur heard best at the apex. His jugular venous pressure (JVP) is 10 cm at 30 degrees. An electrocardiogram taken in the office reveals atrial fibrillation at a rate of 94/min with normal ST segments. Which of the following is the most appropriate next step in management? A. Discontinue the ACE inhibitor B. Initiate amiodarone therapy C. Initiate beta-blocker therapy D. Initiate digoxin (Lanoxin) therapy E. Initiate furosemide (Lasix) therapy

The correct answer is C. Recall that in the treatment of medical conditions, certain medications overlap syndromes and are efficacious in many areas. This "co-treatment" option maximizes the benefits of each drug in a regimen and often addresses 2 or more issues simultaneously. Ischemia-mediated arrhthymia is the most common cause of death in a person who has congestive heart failure (CHF). Beta-blockers will control the rate in atrial fibrillation, as well as provide a mortality benefit for the CHF. Beta-blockers are both anti-ischemic and antiarrhythmic. In this case, ACE inhibitors have been shown to be very beneficial in prolonging the survival of patients who have CHF, as they are effective as vasodilators. They are also useful antihypertensive agents. Given this, discontinuing his ACE inhibitor (choice A) is clearly incorrect. This patient requires rate control for his atrial fibrillation so that acute heart failure does not recur. Short of restoring this patient's atrial contractions, rate control is the best method to ensure adequate management of atrial fibrillation. Digoxin, with or without a nodal agent such as a beta-blocker, has been shown to be reasonably effective at rate control, but it is less effective in high adrenergic states. Beta-blockers such as metoprolol (Lopressor) and the combined alpha- and beta-blocker carvedilol (Coreg), however, have been clearly shown to lower mortality with CHF, in addition to just controlling the heart rate.

For patients who have heart failure, which of the following findings would show that there is a need for placement of a resynchronization pacemaker? A. ST segment elevation B. Prolonged QT interval C. Wide QRS complex D. Bradycardia with multiple escape PVCs E. Polymorphic ventricular tachycardia

The correct answer is C. Resynchronization pacemaker is indicated for patients with wide QRS complex. The use of a resynchronization pacemaker is an effective therapy in patients with heart failure and dyssynchrony identified as a prolonged QRS duration. CRT can improve exercise tolerance and New York Heart Association (NYHA) functional class and reduce both mortality and the need for hospitalization in patients in sinus rhythm. Allowing the heart to have its electrical pathways to be restored, wherein the left ventricle precedes contraction of the right, ventricle will help to alleviate symptoms of heart failure. Patients with a failing left ventricle and decompensated heart failure may also have increased risk for sudden cardiac death. When patients have resynchronization pacemaker placement, an automatic cardiac defibrillator can be placed at the same time.

A patient is seen in the office and admits a history of IV drug use. He is febrile and has a newly discovered cardiac murmur. The patient is noted to have oval retinal hemorrhages with a clear, pale center. What are these lesions known as? A. Herald patches B. Janeway lesions C. Roth spots D. Target lesions E. Osler nodes

The correct answer is C. Roth spots are located in the retina and are oval with clear, pale center in the setting of infective endocarditis.

Patent ductus arteriosus can occur as a result of an infant being born prematurely. This condition also has an increased incidence because of congenital infection. Which of the following prenatal infections is most likely to lead to patent ductus arteriosus? A. Measles (rubeola) B. Mumps C. Rubella (German measles) D. Parvovirus E. Herpes simplex

The correct answer is C. Rubella is one of the TORCH (toxoplasmosis, other [syphilis, varicella-zoster, parvovirus B19], rubella, cytomegalovirus, and herpes) viruses that adversely affects the fetus when the mother acquires the infection during pregnancy. In addition to causing patent ductus arteriosus, the baby may also develop congenital cataracts and deafness.

A patient with a long-standing atrial septal defect (ASD) is evaluated. The patient wishes to discuss the potential role for closure of his secundum type of ASD. Which of the following comorbidities would serve as a contraindication for closure of an ASD? A. Chronic liver disease B. Chronic renal disease C. Significant pulmonary hypertension D. Mitral stenosis E. Right ventricular hypertrophy

The correct answer is C. Significant pulmonary hypertension is a contraindication for closing an atrial septal defect because of the need to support the additional volume and maintain cardiac output that ASD provides.

A patient is seen in the office with progressive heart failure requiring medications following the use of an ACE inhibitor and beta blocker. Spironolactone (Aldactone) has been shown to be effective in the management of class III and class IV heart failure. There has also been mortality benefit when this this agent is used. Which of the following is the primary contraindication for using this medication? A. Liver failure B. Hypokalemia C. Renal failure D. Diabetes mellitus E. COPD

The correct answer is C. Spironolactone cannot be used in the setting of renal failure because renal failure causes hyperkalemia. As a potassium-sparing diuretic, this agent will further increase potassium levels. Spironolactone has evidence-based improvement in mortality when used in the setting of class III and class IV heart failure. Spironolactone is cleared in the kidney and the only monitoring recommendation is with serum creatinine and potassium.

A blood pressure of 135/85 would be what stage of hypertension? A. Normal B. Elevated C. Stage 1 D. Stage 2

The correct answer is C. Stage 1 hypertension is defined by systolic blood pressure of 130 to 139 or diastolic blood pressure of 80 to 89.

It is vital for health care providers to identify risk factors for aortic aneurysms. Non-modifiable risk factors such as male sex and aging can contribute to the development of aortic aneurysms. There may be other reasons why aortic aneurysms develop. Which of the following medical conditions puts a patient at increased risk for the development of aortic aneurysms? A. Chlamydia B. Gonorrhea C. Syphilis D. Mycoplasma infection E. Parvovirus B19 infection

The correct answer is C. Syphilis puts patients at risk for thoracic and lower aortic aneurysms, as this infection can affect the vasculature, including the aorta.

A 1-week-old baby girl has a coarctation of the aorta just distal to the subclavian arteries. The blood pressure distal to the constriction is 50% lower than normal. Which of the following is increased in this infant? A. Blood flow in the lower body B. Glomerular filtration rate C. Plasma levels of renin D. Renal excretion of sodium E. Renal excretion of water

The correct answer is C. The aorta is constricted at a point beyond the arterial branches to the head and arms but proximal to the kidneys. Collateral vessels in the body wall carry much of the blood flow to the lower body, and the arterial pressure in the lower body is about 50% lower compared with the pressure in the upper body. The lower than normal pressure at the level of the kidneys causes renin to be secreted with the eventual conversion to angiotensin I and angiotensin II. The renin-aldosterone system results in salt and water retention, so that within a few days to weeks the arterial pressure in the lower body (at the level of the kidneys) increases to normal, but in doing so, the blood pressure in the upper body has increased to hypertensive levels. This is the underlying pathophysiology as to why coarctation of the aorta is a secondary cause of hypertension. Correction of the coarctation will then allow the kidney to be normally perfused, and this will result in a "cure" for the patient's hypertension.

A 25-year-old woman involved in an automobile accident is admitted as an emergency patient. A major artery severed in her leg caused an estimated 600 mL blood to be lost. Her blood pressure is 90/60 mm Hg. Which of the following would be expected to increase in response to hemorrhage? A. Arteriolar diameter in skeletal muscles (dilation of arterioles) B. Sodium excretion C. Sympathetic nerve activity D. Vagal nerve activity E. Water excretion

The correct answer is C. The decrease in blood pressure caused by hemorrhage activates the baroreceptor reflex, which tends to increase sympathetic nerve activity and decrease parasympathetic (vagal) nerve activity (choice D). This is the body's main compensation for acute volume loss. The increase in sympathetic nerve activity constricts arterioles in skeletal muscle (choice A) and elsewhere in the body.

A cyanotic infant is discovered to have a ventricular septal defect, an overriding aorta, right ventricular hypertrophy, and complete pulmonic stenosis. Which of the following accompanying congenital anomalies improves survival in this patient? A. Bicuspid aortic valve B. Ostium secundum defect C. Patent ductus arteriosus D. Patent foramen ovale E. Preductal coarctation of aorta

The correct answer is C. The ductus arteriosus connects the aorta with the pulmonary artery. If it remains patent after birth, it allows oxygenated blood to flow from the aorta to the pulmonary artery. In this patient, who has tetralogy of Fallot with complete right ventricular outflow obstruction, this anastomosis is a crucial source of blood to the pulmonary vasculature.

A 2-year-old boy is shot in the arm in a drive-by shooting. His brachial artery is partially transected and there is copious bleeding. Emergency medical technicians are able to control the site of bleeding by local pressure and the child stops losing blood, though he is hypotensive and tachycardic. Intravenous fluid resuscitation is urgently needed, but several attempts at starting peripheral intravenous lines are unsuccessful. Which of the following is the best alternative route in this situation? A. Central line via subclavian puncture B. Hypodermoclysis C. Intraosseous cannulation in the proximal tibia D. Percutaneous femoral vein cannulation E. Saphenous vein cutdown

The correct answer is C. When intravenous lines cannot be established, the preferred alternate route is intraosseous (IO) cannulation. This is done by placing a trocar in the bone marrow of a long bone. The site of choice in children is the proximal tibia; alternative sites are the distal tibia and proximal femur. Careful attention should be made in regard to IO location in order to avoid injury to the growth plate which could stunt bony growth as the child grows older.

A patient develops documented ventricular tachycardia. The patient survives this episode and asks the health care provider what caused him to develop this dysrhythmia. The most common cause for the development of ventricular tachycardia is which of the following? A. Complete heart block B. End-stage aortic stenosis C. Coronary artery disease with prior MI D. Untreated hypertension E. Left ventricular thrombus

The correct answer is C. The most common cause of ventricular tachycardia is CAD and MI. Patients with structurally normal hearts do not typically develop ventricular tachycardia. Patients with VT even when it is paroxysmal should have evaluation for underlying heart disease because VT may be a marker for coronary artery disease.

A 55-year-old woman has been known for years to have mitral valve prolapse. She has now developed exertional dyspnea, orthopnea, and atrial fibrillation. She has an apical, high-pitched, holosystolic heart murmur that radiates to the axilla and back. Because of her deterioration, surgery has been recommended. Which of the following is the most appropriate procedure? A. Aortic valve replacement B. Mitral commissurotomy C. Mitral valve annuloplasty D. Mitral valve replacement E. Both aortic and mitral valve replacement

The correct answer is C. The physical findings are classic for mitral valve insufficiency. Whenever possible, repair of the native mitral valve is preferable to replacement. The way to repair an insufficient valve is to tighten the annulus, bringing the leaflets closer to one another, allowing better coaptation of the valve. Repair of the mitral valve, rather than replacement, allows better preservation of the left ventricle and no need for long-term anticoagulation.

Which of the following diuretics acts at the distal tubule of the nephron? A. Ethacrynic acid (Edecrin) B. Furosemide (Lasix) C. Hydrochlorothiazide (Microzide) D. Mannitol (Osmitrol) E. Spironolactone (Aldactone)

The correct answer is C. The thiazide diuretics (e.g., hydrochlorothiazide, chlorothiazide, benzthiazide) promote diuresis by inhibiting reabsorption of NaCl, primarily in the early distal tubule.

A patient is seen in the office who is newly diagnosed with hypertension. Several agents are considered for therapy, and risks versus benefits are discussed with the patient along with a discussion about what comorbid conditions already present in the patient. Which of the following is a contraindication for using thiazide diuretics in the management of hypertension? A. Bronchospasm B. Hyperkalemia C. Gout D. Heart failure E. Peripheral vascular disease

The correct answer is C. Thiazide diuretics have a compelling contraindication with gout.

Caring for patients with shock requires that the health care provider understands the body's physical examination signs and symptoms and the body's physiologic response to shock. All of the following are typically seen with shock EXCEPT: A. Oliguria B. Change in mental status C. Lactic acidosis D. Right upper quadrant pain E. Hypotension

The correct answer is D. Shock causes hypotension, tachycardia, oliguria, lactic acidosis, and change in mental status. There is no particular reason that a patient would have right upper quadrant pain in the setting of shock. Right upper quadrant tenderness can occur in the setting of right-sided heart failure.

A 71-year-old white man comes to the emergency department because of blurry vision and blood-tinged urine. He states that he has a long history of hypertension treated with a beta-blocker, an ACE inhibitor, and a calcium channel blocker. He reports that he has had 3 days of blurry vision and urine that is "cola" colored. The symptoms began when he ran out of his medications 3 days ago and he has "not had the time" to get the prescriptions refilled. His physical examination is remarkable for a blood pressure of 200/110 mm Hg in both arms and funduscopic examination showing flame hemorrhages and papilledema. Urinalysis shows red blood cells and some dysmorphic red blood cell casts. He has an abdominal bruit. Which of the following is the most appropriate next step in management? A. Order an ECG and observe the patient B. Give the patient his usual daily doses of BP medications and observe him C. Give the patient intravenous medication to reduce his BP D. Order an abdominal CT scan to rule out aortic aneurysm E. Perform a bedside cystoscopy to evaluate the hematuria

The correct answer is C. This patient has a hypertensive emergency, as evidenced by both the presence of elevated systolic and diastolic blood pressure and evidence of end-organ damage. The blood pressure numbers themselves are a good reason for urgency, but the presence of increased intracranial pressure and renal failure require that this patient be triaged as an emergency. Elevated blood pressure alone, in the absence of symptoms or end-organ damage, rarely requires emergency therapy. The therapy for a hypertensive emergency requires IMMEDIATE LOWERING of the blood pressure by approximately 20-30 mm Hg by intravenous administration of medication. The blood pressure should not be reduced to normotensive levels because of the risk for watershed cerebral infarcts with such a dramatic reduction.

A 7-year-old girl is brought to the office because of fever, headache, and malaise that has been increasing over the previous 7 days. She had been away at her grandparents' house in the countryside for vacation and had come down with a sore throat and fever. Her grandmother had given her some over-the-counter cold medication and kept her in bed for a few days, but when she had not been getting better, they had brought her back to the city. Her mother noted a faint red rash that has been waxing and waning on her chest, back, and shoulders, that would become very red after a hot bath. The girl's past medical history is unremarkable and she has not been on any medications recently. On physical examination, the patient is in moderate distress with a temperature of 39.4°C (102.9°F), pulse of 120/min, and respirations of 30/min. She complains of chest discomfort and repeatedly coughs during the examination. Deep breaths cause moderate chest pain, so her breathing is shallow. She also has difficulty swallowing. The tonsils are edematous, erythematous, and covered with purulent exudate. Her ankles are swollen, erythematous, and with limited range of motion. The rash her mother described is present only on the right shoulder in the form of an annular erythematous plaque, 5 cm in diameter. A rapid strep test done in the office is positive. Which of the following complications is the most likely to develop in this patient? A. Aortic valve disease B. Complications are unlikely to happen in this patient C. Mitral valve disease D. Pulmonary valve disease E. Tricuspid valve disease

The correct answer is C. This patient has acute rheumatic fever with signs of evolving carditis. Mitral valve disease is the most common complication, occurring in up to 70% of cases. Acute rheumatic fever is caused by infection with group A β-hemolytic streptococci. It is most commonly seen in children aged 5-15 years because they are most susceptible to infection with group A β-hemolytic streptococci. Skin infections with group A β-hemolytic streptococci do not predispose to rheumatic fever, but upper respiratory infections do. Acute rheumatic fever usually presents 1-5 weeks after a preceding streptococcal pharyngitis. Diagnosis is based on the Jones criteria that include clinical signs and symptoms.

A patient is seen in an acute care facility who is diagnosed with acute pulmonary edema. The patient was initially given furosemide, morphine, oxygen, nitroglycerin, and norepinephrine. The patient's systolic blood pressure is greater than 100. Which of the following medications is now indicated as the next step in his management? A. Metoprolol (Lopressor) B. Atenolol (Tenormin) C. Diltiazem (Cardizem) D. Captopril (Capoten) E. Spironolactone (Aldactone)

The correct answer is D. In the setting of acute pulmonary edema in a patient who has failed treatment with first-line mediations, a health care provider should add an ACE inhibitor such as captopril in order to produce further vasodilation. This treatment can be tolerated as long as the patient's blood pressure is being supported with vasopressors such as norepinephrine already being used.

A 66-year-old woman with newly diagnosed type 2 diabetes comes to the health care provider for a blood pressure check. At a previous visit 3 months ago, her blood pressure was 140/95 mm Hg and she was advised to attempt a moderate weight loss with a low-impact exercise program and diet modification. Unfortunately her weight has increased 2 kg (4.4 lb) since her last visit. Her blood pressure is now 144/95 mm Hg. She is mildly resistant to starting medication at this time. Laboratory studies show: Na+ - 140 mEq/L K+ - 3.9 mEq/L Cl− - 109 mEq/L HCO3− - 22 mEq/L Glucose - 176 mg/dl BUN - 23 mg/dl Creatinine - 1.2 mg/dl Urinalysis shows: Color - Clear Leukocyte esterase - Negative Nitrite - Negative Blood - Negative Microscopic - No cells, no casts Protein - 3+ Which of the following is the most appropriate intervention at this time? A. Begin therapy with atenolol (Tenormin) B. Begin therapy with labetalol (Trandate) C. Begin therapy with lisinopril (Zestril, Prinivil) D. Begin therapy with verapamil (Calan, Isoptin, Verelan) E. Continue lifestyle modification

The correct answer is C. This patient has diabetes, hypertension, and proteinuria. Her blood pressure must be well controlled to prevent progression of nephropathy. ACE inhibitors have been shown to prevent progression of diabetic nephropathy in type 1 diabetics and likely have similar preventive effects for type 2 diabetics. ACE inhibitors such as lisinopril should be first-line therapy for diabetic patients who have hypertension. This patient's potassium is not elevated, so this class of medications is not contraindicated.

A 12-year-old girl presents to her health care provider with an erythematous macular skin rash in a bathing suit distribution. She has a history of streptococcal sore throat a few weeks ago. If rheumatic fever is under consideration as a diagnosis, which of the following signs would be necessary to make this diagnosis as this sign is part of the Jones major criteria? A. Elevated erythrocyte sedimentation rate B. Leukocytosis C. Migratory polyarthritis D. Prolonged PR interval on EKG E. Temperature >38°C (100.4°F)

The correct answer is C. This patient's macular skin rash in a bathing suit distribution is also known as erythema marginatum, one of the five major Jones criteria for diagnosing rheumatic fever. The other four major criteria are migratory polyarthritis, Sydenham chorea, subcutaneous nodules, and pancarditis. In context of prior streptococcal infection, the presence of two of five major criteria or one major plus two minor criteria (choices A, B, D, or E) is sufficient to establish the diagnosis. Other minor criteria include previous rheumatic fever and elevated C-reactive protein. A mnemonic for the major criteria is CANCER (not pleasant, but easy to remember): Chorea, Arthritis, Nodules, Carditis, Erythema marginatum, which is consistent with Rheumatic fever diagnosis.

A patient is evaluated with sepsis that is evaluated clinically. Echocardiogram reveals vegetations on the tricuspid valve that are consistent with tricuspid endocarditis. Which of the following is the most common cause of this condition? A. Acute MI B. Cor pulmonale C. IV drug use D. Ebstein anomaly E. Atrial septal defect

The correct answer is C. Tricuspid endocarditis most commonly due to intravenous (IV) drug use. The bacteria can be injected from the skin due to inadequate skin preparation, or bacteria may be introduced as a result of nonsterile IV drugs being injected. There are many contaminants that are part of these illicit drugs being injected. With the increased use of heroin, there has been a steady increase in the number of tricuspid endocarditis. Tricuspid endocarditis is best identified with echocardiogram.

A patient with a long-standing, large, unrepaired ventricular septal defect has pressure measurements performed and is found to have increased pulmonary vascular resistance. If these physiologic changes do result in changes on the EKG, which of the following changes will be seen? A. Right ventricular hypertrophy B. Left ventricular hypertrophy C. Biventricular hypertrophy D. Second-degree heart block type 2 E. Third-degree heart block

The correct answer is C. Unrepaired ventricular septal defect can result in high pulmonary vascular pressures, which can lead to biventricular hypertrophy on EKG.

Which of the following medications should be avoided in the setting of an acutely dissecting aorta? A. Propranolol (Inderal) B. Labetalol (Trandate) C. Vasodilators D. Sodium nitroprusside E. Calcium-channel blockers

The correct answer is C. Vasodilators should be avoided in the setting of aortic dissection because they can increase sheer strength of the aorta, which promotes further tearing of the dissection.

A patient is seen in the office. She has multiple medical issues and is taking glyburide, metformin (Glucophage), Lisinopril (Zestril), chlorthalidone (thiazide diuretic), and cholestyramine (Questran). She has been diagnosed with hyperlipidemia due to an increase in her serum cholesterol. Which of the following agents is most likely contributing to this hyperlipidemia? A. Glyburide B. Metformin (Glucophage) C. Lisinopril D. Chlorthalidone E. Cholestyramine

The correct answer is D. Chlorthalidone is a long-acting thiazide diuretic. Thiazide diuretics, estrogen, steroids, and beta-blockers may contribute to hyperlipidemia.

A patient admitted to the emergency department with chest pain is diagnosed with myocardial infarction. On discharge, the patient is prescribed aspirin but develops an allergic hypersensitivity reaction. Clopidogel (Plavix) is then prescribed. Which of the following is the mechanism of action of this drug? A. It binds to the active site of cyclo-oxygenase via acetylation B. It blocks the binding of plasmin to fibrin C. It hinders the production of thromboxane A2 D. It inhibits adenosine diphosphate-induced platelet aggregation E. It stimulates platelet adenylyl cyclase

The correct answer is D. Clopidogrel (Plavix) is an antiplatelet drug that inhibits adenosine diphosphate-induced platelet aggregation and interacts with platelet glycoprotein IIb/IIIa, a fibrinogen receptor that links platelets together and leads to platelet plug formation. The drug thus prevents fibrinogen from binding to platelets. The drug is useful both for patients who have coronary disease and for those who have cerebrovascular disease such as stroke. Of course, hemorrhagic stroke should first be ruled out before it is deemed safe to use any anticoagulant. Clopidogrel is used in both acute coronary syndrome (for patients not having an open bypass) and for thrombotic event prevention. For stroke prevention, it is used when aspirin or warfarin (Coumadin) cannot be used. Clopidogrel is indicated for use following placement of drug eluting stent during PCI therapy and this agent is ideally used for one year to maintain stent patency.

A 50-year-old female police officer experiences an episode of substernal chest pain while chasing a suspect on foot. She arrives in your clinic 4 hours later, describing the pain as a pressure sensation and nonradiating, lasting 5 to 10 minutes and relieved by resting after the pursuit. It was associated with shortness of breath, but no nausea, vomiting, diaphoresis, or palpitations. Her physical examination and resting ECG are within normal limits. She currently smokes cigarettes. Which of the following is the most appropriate initial course of action? A. Empirically place her on an ACE inhibitor B. Cardiac catheterization C. Upper gastrointestinal series D. Exercise stress test E. CT scanning of the coronaries

The correct answer is D. Patients who have a new onset of chest pain or a change in the previously established pattern of angina are classified as having unstable angina. Anginal symptoms are classically described as prolonged or new chest pain; EKG in these patients can be normal or show signs of ST-segment depression. Cardiac enzymes are normal. Exercise stress testing is the most widely used diagnostic test initially, even though the overall sensitivity of the test is only about 75%. Exercise stress testing interpretation should include patient symptoms during and following exercise, exercise capacity, hemodynamic response, and EKG response to exercise. If the patient has a pre-exercise EKG showing left bundle branch block, the ensuing exercise-induced EKG will be difficult to interpret, which might result in the patient having stress imaging as part of the evaluation strategy.

A patient is seen in the emergency department because of acute decompensation of heart failure because of dietary indiscretion caused by eating a jar of pickles. For this patient, which of the following treatments is the most important part of the management of this decompensated condition? A. Beta blockers B. ACE inhibitors C. Nitrates D. Diuretics E. Digoxin (Lanoxin)

The correct answer is D. The best treatment for acutely decompensated heart failure is diuretics, as they are best at decreasing the preload. Unloading the heart in the setting of decompensated heart failure is key at allowing the heart to recover and change from decompensated to compensated heart failure. Diuretics will help with congestive symptoms, and less intravascular and intracardiac volume lowers myocardial oxygen demand. In order to preserve ventricular function, inotropes such as dobutamine may be added in order to support the pump function of the heart with acute decompensated heart failure. A search for the underlying cause of the decompensation should be made, with correction of the underlying cause being employed as key to this treatment.

A patient presents to the office with complaints related to his chest. He admitted to drinking too much green beer on St. Patrick's Day, and he is diagnosed with "holiday heart syndrome." Which of the following cardiac dysrhythmias is he diagnosed with? A. Ventricular bigeminy B. Nonsustained ventricular tachycardia C. Atrial flutter D. Atrial fibrillation E. Multifocal atrial tachycardia

The correct answer is D. "Holiday heart" is the term to describe the situation in which alcohol use precipitates atrial fibrillation. Atrial fibrillation can occur in about 60% of binge alcohol users, and atrial fibrillation can still occur without alcohol-induced cardiomyopathy. Heavy alcohol consumption has a direct correlation with an increased incidence of atrial fibrillation.

Patients with STEMI who develop hemodynamically stable ventricular tachycardia should be treated with which of the following? A. Immediate defibrillation B. Immediate cardioversion C. 300 mg amiodarone bolus followed by drip D. 150 mg amiodarone bolus followed by drip E. 1 mg epinephrine

The correct answer is D. A dose of 150 mg of amiodarone is indicated for hemodynamic stable ventricular tachycardia following MI.

In preparation for an inguinal hernia repair, a 22-year-old man has a spinal anesthetic placed. The level of sensory block turns out to be much higher than had been planned, and shortly thereafter his blood pressure drops to 75/20 mm Hg. He looks warm and flushed, and his central venous pressure is near 0 mm Hg. He has no history of blood pressure abnormalities. Which of the following should be included in his therapy? A. Diuretics and fluid restriction B. Whole blood and clotting factors C. Inotropic agents and cardiac assist pump D. Vasoconstrictors and intravenous fluids E. Vasodilators and intravenous fluids

The correct answer is D. A high spinal anesthetic can produce vasomotor shock by inducing widespread vasodilation. Vasoconstrictors are the appropriate therapy, but because the capacity of the vascular tree is also increased under these circumstances, filling it up with additional volume is also helpful.

A patient is seen in the office with complaints regarding peripheral arterial disease. The patient has been walking longer distances in order to improve the claudication symptoms, but he knows that he will need to start medications. Which of the following medications used in the management of peripheral arterial disease works by both suppressing platelet aggregation and by directly dilating arterioles? A. Aspirin B. Clopidogrel (Plavix) C. Ticlopidine (Ticlid) D. Cilostazol (Pletal) E. Nitroglycerin

The correct answer is D. Cilostazol is a platelet and phosphodiesterase-3 inhibitor. It suppresses cyclic AMP, resulting in vasodilation. It is contraindicated in the setting of chronic heart failure due to increased mortality.

Which of the following is the treatment utilized for patients who develop second-degree type 2 heart block? A. Atropine B. Adenosine C. Dopamine D. Permanent pacemaker E. Norepinephrine

The correct answer is D. A permanent pacemaker is indicated for second-degree heart block type 2, as this block is below the atrioventricular (AV) node, where atropine will not work.

Heart failure is the most common medical reason for admission to hospitals in the United States. Providers need to understand the pathophysiology of this condition in order to develop a treatment regimen. For patients with chronic heart failure, which of the following criteria is the most compelling reason for adopting ACE inhibitors as treatment for this condition? A. Fluid overload B. Pulmonary congestion C. Peripheral edema D. Poor left ventricular function E. Poor relaxation of the left ventricle

The correct answer is D. ACE inhibitors are indicated when there is low EF. In fact, ACE inhibitors as vasodilators have their major effect in the setting of a poorly functioning left ventricle, especially if the EF is 40% or lower. As centrally acting vasodilators, these agents drop the left ventricular outflow resistance allowing the left ventricle to empty. As a result of enhanced ability of the left ventricle to empty with the use of ACE inhibitors and ARBs, there is less left ventricular remodeling, which improves mortality in the setting of chronic heart failure.

A patient is seen in a cardiology practice for further evaluation of suspected angina and coronary artery disease (CAD). The patient's baseline EKG reveals left bundle branch block (LBBB). Which of the following diagnostic interventions is recommended in this setting? A. Traditional exercise treadmill test B. Cardiac catheterization C. Single-photon emission computerized tomography (SPECT) scanning D. Adenosine radionuclide scanning E. Multigated acquisition (MUGA) scan

The correct answer is D. Adenosine or dipyridamole (Persantine) radionuclide testing is recommended in the setting of suspected CAD with baseline LBBB. The presence of LBBB causes interference with left ventricular depolarization, which makes the EKG difficult to interpret.

Which of the following is contraindicated in the setting of torsades de pointes? A. Magnesium B. Isoproterenol C. Lidocaine D. Amiodarone E. Overdrive pacemaker

The correct answer is D. Amiodarone is a Vaughn Williams class III medication that acts to suppress ventricular arrhythmias in the setting of cardiac arrest. It can be used to suppress both atrial and ventricular arrhythmias. Medications like amiodarone that prolong the QT interval are contraindicated in the setting of torsades because prolonged QT is the problem underlying this condition.

A patient is seen with chest pain, dyspnea, and increased pain when taking a deep breath. The patient says that leaning forward seems to help the pain. Auscultation of the heart reveals a three-phase friction rub. Common causes of this condition include all of the following conditions EXCEPT: A. Viral infections B. Radiation to the mediastinum C. Connective tissue disease such as rheumatoid arthritis D. Amiodarone (Cordarone) E. Uremia

The correct answer is D. Amiodarone primarily causes pulmonary and thyroid toxicity and is not commonly associated with pericarditis. When amiodarone causes cardiac toxicity, the primary problems are bradycardia, arrhythmia, and conduction disturbances.

A 15-year-old girl is brought to a pediatric cardiology clinic with a complaint of chest pain. She states the pain has come and gone over the past year but has increased in frequency over the past few weeks. She describes it as a sharp pain over her left chest. Physical examination reveals a healthy appearing 15-year-old girl. Her temperature is 37.2°C (99°F), pulse 90/min, and respirations 20/min. Lung examination is normal. Cardiac examination reveals a late systolic murmur preceded by a click at the apex. No heave or rub is present. Electrocardiogram and chest radiograph are unremarkable. Which of the following is the most likely diagnosis? A. Atrial septal defect B. Mitral regurgitation C. Mitral stenosis D. Mitral valve prolapse E. Tricuspid regurgitation

The correct answer is D. An apical click followed by a late systolic murmur is classic for mitral valve prolapse. It can be a source of subjective chest pain in children. Mitral valve prolapse is more common in females. Antibiotic prophylaxis is no longer routinely recommended by the AHA prior to dental procedures unless prolapse is accompanied by severe regurgitation.

An elderly homeless man who regularly resides outside the emergency department is acting strangely. He is often seen drinking alcohol and asking for money, but today he seems disoriented, confused, and weak. It is bitterly cold and there is snow on the ground, so one of the health care providers is concerned that the combination of cold weather and alcohol would predispose the patient to hypothermia. Two emergency medical technicians help bring the man inside so he can be fully evaluated. His temperature is 32.0°C (89.6°F), blood pressure 80/40 mm Hg, pulse 53/min, and respirations 10/min. Electrocardiogram would most likely show which of the following findings? A. Delta wave B. Depressed ST segments C. Diffuse broadening of T waves D. Osborn waves E. U waves

The correct answer is D. An upward deflection following the R wave (classically in lead II) describes an Osborn wave (or J wave of Osborn), which is pathognomonic of hypothermia. The Osborn wave appears on the EKG following ST-segment elevation and is seen when the patient's body temperature is 32-33°C (89-91°F). Although these findings are pathognomonic for hypothermia, their absence does not rule out heart involvement in a patient who has hypothermia. As the body's metabolism slows because of the hypothermia, bradycardia is also typically seen.

A patient with a history of the secundum type of atrial septal defect since birth is seen in the office. Which of the following EKG abnormalities is most likely to be seen in the setting of atrial septal defect? A. Poor R-wave progression across the precordium B. Second-degree heart block type 1 C. Second-degree heart block type 2 D. Right bundle branch block E. Left bundle branch block

The correct answer is D. Atrial septal defect causes right bundle branch block (RBBB) and right axis deviation on an EKG. The atrial septal defect classically causes a left-to-right shunting of the blood, which results in more blood having to be moved out of the right side of the heart. This increased right-sided blood volume can result in both RBBB and right axis deviation.

You are seeing a patient in consultation who was admitted with unstable angina. All of the following medications are indicated as part of his therapy EXCEPT which of the following? A. Clopidogrel (Plavix) B. Aspirin C. Low-molecular-weight heparin D. Calcium channel blockers E. Glycoprotein IIb/IIIa inhibitors

The correct answer is D. Calcium channel blockers and fibrinolytics should be avoided in the setting of unstable angina. Patients with unstable angina typically have partial rather than complete occlusion. Calcium channel blockers do not cause coronary artery vasodilation, so these agents will not help in the setting of partial coronary artery occlusion. If fibrinolytics are used in the setting of unstable angina, a partial occlusion could be converted to a total occlusion downstream, which will worsen the clinical condition of the patient.

A patient is newly diagnosed with isolated systolic hypertension, and treatment is planned. Treatment of isolated systolic hypertension includes both diuretics plus which of the following classes of medications? A. ACE inhibitors B. Beta-blockers C. Direct renin inhibitors D. Calcium-channel blockers E. Angiotensin receptor blockers

The correct answer is D. Calcium-channel blockers and diuretics are indicated in the setting of isolated systolic hypertension, as these agents shrink volume (diuretics) or act as vasodilators (calcium-channel blockers).

Patients with ventricular tachycardia may have significant signs and symptoms that occur as a direct result of having this cardiac condition. Which of the following physical signs or symptoms is most commonly associated with ventricular tachycardia? A. Hepatojugular reflex B. Bounding pulse C. Lateral displacement of the point of maximum impulse D. Cannon A waves E. Irritability on the part of the patient

The correct answer is D. Cannon A waves occur during VT because of AV dissociation. When the atria contracts against a closed tricuspid valve, the blood in the atria is reflected back into the jugular veins causing the cannon A waves to occur. Ventricular tachycardia results in atrioventricular dissociation with lack of coordination between the atrial and ventricular contraction.

All of the following are recommended treatments for long-standing lymphedema in patients EXCEPT: A. Meticulous foot hygiene to prevent infection B. Compression boots C. Compression stockings D. Diuretics E. Leg elevation

The correct answer is D. Compression of the lower extremity along with elevation may be helpful for lymphedema management. Diuretics, however, should be avoided in order to prevent intravascular volume depletion. Diuretics do not alter the natural course of lymphedema and are associated with more risks than benefits.

A 22-year-old construction worker is brought to the emergency department after being pinned by a steel beam that fell on his legs and mid-torso. The patient was brought to the hospital by the EMS team who had extricated him. At the scene, his Glasgow score was 15/15, and he was alert and oriented. On arrival the patient appears pale and bloody and in a significant amount of pain, but still alert. Examination reveals bilaterally crushed lower extremities with a visibly pulsating bleed. Emergency laboratory data are as follows: Sodium - 143 mEq/L (135-145 mEq/L) Potassium - 5.2 mEq/L (3.5-5.2 mEq/L) Bicarbonate - 20 mEq/L (20-29 mEq/L) Chloride - 98 mEq/L (96-106 mEq/L) Urea nitrogen (BUN) - 37 mg/dL (7-20 mg/dL) Creatinine - 1.1 mg/dL (0.8-1.4 mg/dL) Creatine phosphokinase - 15,300 U/L (52-200 U/L) Aspartate aminotransferase - 112 IU/L (<40 IU/L) Alanine aminotransferase - 99 IU/L (<40 IU/L) Hematocrit - 38% (41-50%) Arterial blood gas on 40% oxygen pO2 - 75 mm Hg (75-100 mm Hg) PCO2 - 37 mm Hg (35-45 mm Hg) pH - 7.33 (7.35-7.45) Which of the following is the most appropriate intervention at this time? A. 12-lead electrocardiogram B. Echocardiogram C. Fractionated creatine kinase levels D. Intravenous crystalloid and bicarbonate E. Right upper quadrant ultrasound

The correct answer is D. Crush injuries such as the one sustained by this patient often result in a massive release of muscle contents. Creatine kinase (CK) is an enzyme released by dead or damaged muscle into the blood. By itself it is harmless, but it is a marker for myoglobin, which is directly nephrotoxic. Myoglobin, which is not measured by conventional assay, is released after damage to muscle. Aldolase levels can also be used to assess the degree of rhabdomyolysis. Much data exist showing early intervention with copious alkalinized IV crystalloid can prevent renal damage.

Which of the following factors is recognized as contributing to atherosclerotic risk? A. Decreased lipoprotein A B. Hypohomocysteinemia C. Decreased fibrinogen levels D. Decreased fibrinolytic activity E. Hypoinsulinemia

The correct answer is D. Decreased fibrinolytic activity or an elevated plasminogen activator inhibitor helps to modulate the balance between fibrinogen factors. Fibrinolytic activity helps to clear areas of plaque, which contributes to atherosclerosis.

Fibrinolytic and thrombolytic therapy used in the setting of STEMI has benefits but also real risks associated with their use. Patients seen in the emergency department with STEMI who receive fibrinolytic therapy may have this therapy up to how long following the onset of chest pain? A. 3 hours B. 6 hours C. 9 hours D. 12 hours E. 24 hours

The correct answer is D. Fibrinolytics can be used up to 12 hours after onset of symptoms in the setting of STEMI with benefits exceeding risks for this time period.

Heart failure can occur with either preserved or reduced ejection fraction from the left ventricle. Because the etiology of these two types of heart failure may be different along with different treatments being employed, it is important for the clinician to differentiate between these types of heart failure. Which of the following clinical or diagnostic findings is used to differentiate systolic heart failure from diastolic heart failure? A. Central venous pressure B. Loudness of S2 C. Whether there is a paradoxical closing of the aortic and pulmonic valves D. Ejection fraction percentage E. Whether the patient is symptomatic with exercise versus symptomatic with lying down

The correct answer is D. Differentiate between systolic and diastolic heart failure by ejection fraction. Diastolic heart failure is known as heart failure with preserved ejection fraction, whereas systolic heart failure is known as heart failure with preserved ejection fraction.

Which of the following patient groups is most likely to benefit from diuretics in the setting of hypertension? A. Those with chronic kidney disease B. Those with comorbid liver disease C. Those with orthostatic hypotension D. African American population E. Young Caucasian population

The correct answer is D. Diuretics to treat hypertension are most effective in elderly and African American populations, as they have low renin output, which does not negate the effects of having a lower intravascular volume.

Which of the following vasoactive medications used in the setting of acute MI has the potential to improve blood flow to the kidneys when used in a low-dose formulation? A. Nitroglycerin B. Dobutamine C. Nitroprusside D. Dopamine E. Norepinephrine

The correct answer is D. Dopamine stimulates alpha and beta-1 receptors in order to act as an inotropic and vasopressor agent. When it is given at a low dose at less than 5 microgram per kg per minute, dopamine has the potential to improve renal blood flow due to its ability to cause vasodilation of the mesenteric receptors. When dopamine is given at higher doses, this effect is not seen, and only pure vasopressor activity occurs without preservation of renal blood flow.

Patients with hypertension may be treated with a variety of agents. To which of the following classes of medication does ethacrynic acid (Edecrin) belong? A. Thiazide diuretics B. Potassium-sparing diuretics C. Combined alpha-beta blockers D. Loop diuretics E. Direct renin inhibitors

The correct answer is D. Ethacrynic acid is a loop diuretic. It is most useful in situations in which the patient has a sulfa allergy because there is no sulfa ring with this agent.

A patient is seen in the office after having medication-induced cardioversion in the setting of chronic atrial fibrillation. The patient has continued to have normal sinus rhythm following cardioversion. Which of the following treatments is now indicated? A. Ablation of the AV node and permanent pacemaker placement B. High-dose BBs in order to maintain normal sinus rhythm C. Calcium-channel blockers to prevent coronary vasospasm D. Anticoagulation for at least 3 weeks E. Anticoagulation for at least 3 months

The correct answer is D. Expert guidelines recommend that patients continue anticoagulants for at least 3 weeks following conversion to normal sinus rhythm to prevent thrombosis.

A patient is seen in the office. You are reconciling her medications and see that she is taking fish oil and a fibrate agent. Which of the following lipid disorders is she most likely to have? A. Low HDL B. High LDL C. High total cholesterol D. High triglycerides E. All are equally possible.

The correct answer is D. Fibrates and fish oil are used to treat high triglyceride levels.

Which of the following lipid-lowering agents is most likely to cause gynecomastia, gallstones, weight gain, and myopathies as side effects of their use? A. Fat-soluble statins B. Water-soluble statins C. Niacin D. Fibrates E. Bile acid-binding resins

The correct answer is D. Fibrates cause gynecomastia, weight gain, gallstones, and myopathies as their primary adverse action. These side effects are more prominent if fibrates are used concurrently with statins.

Patients who cannot take ACE inhibitors or angiotensin-receptor blockers (ARBs) in the setting of chronic heart failure may be placed on hydralazine (Apresoline). What additional agent is given in this setting? A. Loop diuretic B. Thiazide diuretic C. Calcium-channel blocker D. Nitrate E. Beta-blocker

The correct answer is D. Hydralazine plus isosorbide dinitrate are used in combination for chronic heart failure management. For vasodilation to occur, both of these agents need to be used simultaneously to achieve the same effects.

A patient is seen in the office with greatly elevated blood pressure. Which of the following is the primary discriminating feature as to whether this presentation is a hypertensive emergency versus hypertensive urgency? A. Level of elevation of the systolic BP B. Level of elevation of the diastolic BP C. Level of elevation for the mean arterial pressure D. Whether the patient has signs of end organ damage E. Whether the patient had a previous diagnosis of hypertension prior to this presentation

The correct answer is D. Hypertensive emergency has increased BP plus signs of end organ damage, while hypertensive urgency may have increased BP readings without end organ damage being evident.

A 1-day-old infant girl appears dusky in the newborn nursery during feeding. Oxygen is immediately administered by nasal cannula. Shortly afterward, she develops tachypnea. On physical examination her blood pressure from the right upper arm is 50/30 mm Hg, pulse 180/min, and respirations 60/min. Echocardiogram is consistent with hypoplastic left heart syndrome. Which of the following would likely be found on auscultation? A. Continuous ductal murmur, bounding pulses B. Continuous ductal murmur, poor peripheral pulses C. Holosystolic murmur, poor peripheral pulses, quiet 2nd heart sound D. No murmur, precordial hyperactivity, loud second heart sound E. No murmur, precordial hyperactivity, quiet second heart sound

The correct answer is D. Hypoplastic left heart (HLH) syndrome is a group of closely related cardiac anomalies characterized by underdevelopment of the left cardiac chambers, atresia or stenosis of the aortic and/or the mitral orifices, and hypoplasia of the aorta. These anomalies are an especially common cause of heart failure in the first week of life. The left atrium and ventricle often exhibit endocardial fibroelastosis. Pulmonary venous blood traverses a patent foramen ovale, and a dilated and hypertrophied right ventricle acts as the systemic as well as pulmonary ventricle; the systemic circulation receives blood by way of a patent ductus arteriosus. Infants who have HLH syndrome develop poor perfusion, and metabolic acidosis when systemic blood flow decreases. When the ductus closes, inadequate blood flow to the body occurs because the ductus is the only path for blood to flow from the right ventricle to the body. Even if the ductus remains open, when the infant is given oxygen, the oxygen will dilate the vasculature of the pulmonary circulation, and blood will preferentially flow to the lower pressure pulmonary system, depriving the systemic circulation of adequate perfusion. ECG usually shows right axis deviation, right atrial and ventricular enlargement, and nonspecific ST- and T-wave abnormalities in the left precordial leads. Chest radiography may show only slight enlargement shortly after birth, but with clinical deterioration there is marked cardiomegaly with increased pulmonary vascular markings. Echocardiography is diagnostic and will show a diminutive aortic root and left ventricular cavity and absence or poor visualization of aortic and mitral valves. An infant who has HLH syndrome has a hyperdynamic precordium because the enlarged right ventricle is contracting against systemic pressure. The infant also has a loud second heart sound (S2) because the pulmonary artery acts as the aorta by pumping blood to the systemic circulation through the ductus arteriosus. The high end-systolic pressure markedly enhances S2. The flow from the right ventricle to the pulmonary artery is not turbulent; therefore, there is usually no significant murmur heard on auscultation. When the ductus closes, or when the pulmonary vasculature resistance falls, the flow to the systemic circulation will decrease, causing greatly diminished peripheral pulses. Management of the hypoplastic left heart syndrome patient includes infusion of prostaglandin E1 and administration of room air while on a ventilator (instead of oxygen, which will dilate the vasculature of the pulmonary circulation, causing blood to flow into the pulmonary system rather than the systemic circulation). Prostaglandin E1 may open the ductus arteriosus and restore systemic blood flow. Administration of room air or even hypobaric oxygen (FiO2 less than 21%) and the use of muscle relaxants can prevent hyperventilation and subsequent pulmonary vasodilation, thus reversing systemic hypoperfusion and metabolic acidosis.

A patient is seen in the emergency department with crushing, tearing chest pain. The patient's lower extremity blood pressure is decreased, and the patient is noted to have a widened mediastinum on chest x-ray. Which of the following is considered to be the first-line medication/medication class that is used in the management of this condition? A. Lisinopril (Zestril) B. Clonidine (Catapres) C. IV calcium-channel blockers D. IV beta-blockers E. IV nitroglycerin

The correct answer is D. IV beta-blockers are first-line agents for treating aortic dissection, as these agents decrease inotropic activity of the heart, which lowers the blood pressure from the heart. These agents also decrease sympathetic tone, which also is helpful in the setting of aortic dissection.

A 52-year-old man is discharged from the hospital after an uncomplicated myocardial infarction. Several weeks later, he visits his primary care provider complaining of insomnia, anorexia, and depressed mood. He appears to be clinically depressed. He denies any current chest pain or shortness of breath. Which of the following would be the most appropriate medication to initiate for this patient? A. Methylphenidate (Ritalin, Concerta, Metadate) B. Nortriptyline (Pamelor) C. Phenelzine (Nardil) D. Sertraline (Zoloft) E. Thioridazine (Mellaril)

The correct answer is D. In a patient who has cardiac complications, a selective serotonin reuptake inhibitor (SSRI) such as sertraline has been demonstrated to be the safest and most effective medication for the treatment of clinical depression. It has overtaken tricyclics in the management of depression in patients who have pre-existing heart disease, because it is not associated with prolongation of the QT interval that the tricyclics are known to cause. When compared to tricyclics, SSRIs also have a safer profile if a patient overdoses.

Which of the following clinical conditions is most likely to be associated with prerenal type of renal failure? A. Acute glomerulonephritis B. Bladder tumor C. Prostatic hyperplasia D. Shock E. Transplant rejection

The correct answer is D. It is important to remember that not all oliguria is caused by intrinsic renal disease. Shock, of any etiology, is an example of a prerenal cause of oliguria in which poor renal perfusion leads to inadequate urine output. Other examples of prerenal causes include hepatorenal syndrome, postoperative oliguria, and dehydration. Postrenal causes of oliguria usually involve bilateral obstruction and may be caused by disease of the ureters, bladder neck, or urethra.

A 72-year-old man is scheduled to have elective sigmoid resection for diverticular disease. He has a history of heart disease and had a documented myocardial infarction 2 years ago. He currently does not have angina, but he lives a sedentary life because "he gets out of breath" if he exerts himself. Physical examination reveals jugular venous distention. His hemoglobin level is 9 g/dL (normal 14-17 g/dL in men). If surgery is indeed needed, which of the following should most likely be done prior to the operation? A. Evaluate the patient as a candidate for coronary revascularization B. Place the patient on intensive respiratory therapy C. Order a transfusion to increase the patient's hemoglobin D. Treat the patient for congestive heart failure E. If at all possible, wait 6 months before performing surgery

The correct answer is D. Jugular venous distention in this setting is indicative of congestive heart failure, a condition that would make elective surgery very risky. Medical treatment for congestive heart failure can reduce the risk. This patient has right-sided heart failure, which is most commonly caused by left-sided heart failure.

In the management of chronic heart failure, which of the following diuretics continues to have an effect in the setting of impaired renal function? A. Thiazide diuretics B. Potassium-sparing diuretics C. Direct aldosterone antagonists D. Loop diuretics E. No diuretic works in the setting of impaired renal function

The correct answer is D. Loop diuretics continue to work in the setting of impaired renal function. As renal function continues to decline with chronic heart failure, only loop diuretics will be able to produce urine, while the other classes of diuretic typically lose their effectiveness.

When patients develop symptoms as a result of acute myocardial infarction, there may be auscultatory findings consistent with this acute infarction. Which of the following cardiac murmurs occurs as a complication of STEMI? A. Aortic stenosis B. Aortic regurgitation C. Mitral stenosis D. Mitral regurgitation E. Atrial septal defect

The correct answer is D. Mitral regurgitation and ventricular septal defects are complications of STEMI. Mitral regurgitation occurs as a result of infarction of the posterior chordae tendineae rupture, which ordinarily helps the mitral valve to close. If the single vessels supplying it is occluded, the mitral valve is unable to appropriately close, which means that left ventricular contraction will result in blood flowing back into the left atrium from mitral regurgitation (also known as mitral insufficiency). If there is an infarction of the ventricular septum, this tissue is weakened from being infarcted, and ventricular septal defect can occur.

Which of the following valvular abnormalities most commonly occurs in the setting of acute myocardial infarction? A. Aortic stenosis B. Aortic regurgitation C. Mitral stenosis D. Mitral regurgitation E. Pulmonic stenosis

The correct answer is D. Mitral regurgitation occurs from mitral papillary muscle injury. The posterior chordae tendineae of the mitral valve has a single blood supply, typically the right coronary artery. If this artery is occluded, the patient will have infarction to this papillary muscle with resultant pulling away of this chordae tendineae and acute onset of mitral regurgitation. Mitral regurgitation occurs commonly in acute MI with 50% of patients experiencing it. Mitral regurgitation is heralded with an acute systolic murmur, which radiates into the axilla. This condition is likely to occur 2-7 days following inferior MI.

A 70-year-old man is seen in the office. He was diagnosed with a 4.5-cm abdominal aortic aneurysm 2 years ago, and he is seen for follow-up every 6 months. Which of the following diagnostic studies is typically serially performed in this setting? A. CT without contrast B. CT with contrast C. MRA D. Ultrasound E. Plain films

The correct answer is D. Serial ultrasounds to assess the progression of an abdominal aortic aneurysm provide excellent anatomical information, are reproducible, and do not involve x-ray exposure.

A female patient is seen in the office with complaints of palpitations and dyspnea. She is also having episodes of presyncope. The patient is suspected of having mitral valve prolapse as a result of her presentation. Which of the following maneuvers will cause the murmur of mitral valve prolapse to increase? A. Squatting B. Use of beta-blocker C. Sustained hand grip maneuver D. Valsalva maneuver E. Lying down

The correct answer is D. Mitral valve prolapse murmur will increase when there is less blood in the heart because of a reduced left ventricular chamber size, so the Valsalva maneuver decreases the amount of blood in the heart, which will cause the mitral valve prolapse murmur to increase. The Valsalva maneuver typically lessens most other heart murmurs as there is less blood coming into the heart. The Valsalva maneuver will increase the intensity of two murmurs: mitral valve prolapse and hypertrophic obstructive cardiomyopathy.

The diagnosis of myocardial infarction is based upon patient history, cardiac enzyme findings, and EKG findings. One way that a STEMI is diagnosed is with ST-segment elevation in anatomically contiguous leads. There are other ways in which STEMI can be diagnosed without having this type of ST-segment elevation. Which of the following new EKG findings is considered to be the equivalent of STEMI? A. Right bundle branch block B. Left anterior hemiblock C. Left posterior hemiblock D. Left bundle branch block E. Second-degree heart block type 2

The correct answer is D. New-onset left bundle branch block is equivalent to STEMI in the setting of a patient having acute onset of chest pain. This diagnosis is best made when this symptomatic patient has a previous EKG not showing a left bundle branch block pattern. Left bundle branch block pattern results in abnormal repolarization of the left ventricle and is consistent with STEMI when this occurs in the setting of chest pain and positive cardiac isoenzymes.

Nitrates are often used in the management of angina pectoris and ischemic heart disease. Side effects of nitrates include all of the following EXCEPT: A. Tolerance B. Orthostatic hypotension C. Syncope D. Peripheral edema E. Headache

The correct answer is D. Nitrates cause headache, tolerance, orthostasis, and syncope. These effects are the result of nitrates causing increased venous capacitance of blood with less fluid coming back into the heart. There may be associated dilation of the coronary arteries, but this medication does not cause peripheral vasodilation so there is no associated peripheral edema.

Providers need to be aware that various medical conditions can lead to a strain on the heart as part of its pathophysiology. Which of the following conditions is a known cause of high-output heart failure? A. Cushing disease B. Addison disease C. Crohn disease D. Paget disease E. Acromegaly

The correct answer is D. Paget disease causes increased vascularity to the bone as bone remodeling occurs. This increased blood flow to the remodeling bone results in high output heart failure. Anemia is another cause of high output cardiac failure because the body compensates for lack of available blood cells by increasing the circulation of those cells by causing a reflex tachycardia.

Which of the following is a primary differentiating feature between a patient having a STEMI and a patient having acute pericarditis on an EKG? A. Patients with acute pericarditis will have 1st-degree heart block. B. Patients with pericarditis will have right-axis deviation. C. Patients with pericarditis will fully respond to aspirin therapy. D. Patients with STEMI will have reciprocal ST-T wave changes. E. Patients with STEMI will have concurrent Q waves with ST-segment elevation.

The correct answer is D. Patients with STEMI will have ST-segment elevation in anatomically contiguous leads and will have reciprocal changes in anatomically opposite leads. Patients with acute pericarditis will have diffuse ST-wave changes without reciprocal changes occurring.

You are called to assess a patient in the ICU with the diagnosis of "shock." If the patient has cardiogenic shock, which of the following signs or symptoms would be consistent with this particular type of shock? A. Low blood pressure but bradycardic B. Petechial rash C. Decreased urinary output to less than 20 cc per hour D. Distended neck veins E. Breath sounds only on one side of the chest

The correct answer is D. Patients with cardiogenic shock have distended neck veins as a result of the heart not being able to meet systemic demands, with pump failure and the backing up of blood into both the pulmonary system and the right side of the heart. When blood backs up into the right side of the chest, distended neck veins are noted.

A patient is seen in the office with complaints related to his chest. He is complaining of chest discomfort, shortness of breath, and fatigue. He is stable and is sent for a chest x-ray. The patient's x-ray interpretation notes that the patient has a water bottle appearance in the cardiac silhouette. Which of the following is the most likely diagnosis? A. Ruptured aortic aneurysm B. Thoracic aortic aneurysm C. Constrictive pericarditis D. Pericardial effusion E. Ruptured myocardium

The correct answer is D. Pericardial effusion consists of fluid accumulating between the visceral and parietal pericardium. When this fluid accumulates around the heart, there is a change in the overall appearance of the heart, with the heart appearing in the shape of a water bottle.

A 63-year-old retired airline pilot comes to the office for a first time visit. He is switching primary care facilities because he was very unhappy with the care that he previously received. He brings his medical record, which documents longstanding cirrhosis and portal hypertension caused by previous alcohol abuse. He has no history of gastrointestinal bleeding. He has a very long medication list, and he reports that his pharmacist told him that his medications "were wrong" and that his health care provider was prescribing drugs improperly. On review of his medication list, a number of medication incompatibilities are found. Co-administration of which of the following is likely to cause this patient the greatest harm if not corrected? A. Lactulose and oral potassium B. Prednisone and inhaled albuterol (Ventolin, ProAir) C. Propranolol (Inderal) and isosorbide dinitrate (Isordil) D. Spironolactone (Aldactone) and oral potassium E. Spironolactone (Aldactone) and hydrochlorothiazide

The correct answer is D. Pharmacology is one of the most important applied basic science topics in clinical medicine. Possible drug interactions are so important a concern that entire organizations and computer cross-check systems have been created to address this issue. A health care provider must still know, however, the consequences and potential interactions of every medicine prescribed. Spironolactone is a potassium-sparing diuretic. The mistake of calling this drug a "diuretic" and cueing your brain to think "replenish electrolytes" could cause this patient to die from hyperkalemic cardiac arrest. Spironolactone is increasingly used as an add-on agent for chronic heart failure, as there has been mortality benefit demonstrated with its use. Spironolactone also is a direct aldosterone antagonist and can be beneficial in the setting of cirrhosis with ascites, to lessen fluid accumulation.

A 60-year-old alcoholic female who appears malnourished presents to her health care provider complaining of shortness of breath and gasping for air on awakening. Cardiac examination reveals an S3 heart sound, a diastolic murmur, and jugular venous distention. Pulmonary rales and peripheral edema are evident. An echocardiogram would be expected to reveal which of the following? A. Carotid pulse tracing with spike and dome configuration B. Bilateral atrial enlargement and ventricular thickening C. Depressed left ventricular function with pericardial effusion D. Left and right ventricular dilatation with poor contraction throughout E. Left ventricular hypertrophy with asymmetric septal hypertrophy

The correct answer is D. Primary acquired cardiomyopathy can occur in the peripartum period or from tachycardia-induced dilated cardiomyopathy. Secondary, toxicity-related cardiomyopathy can occur as a result of alcohol, doxorubicin use, or from heavy metal or chemical exposure. Peripartum dilated cardiomyopathy typically occurs in obese, multiparous women older than age 30. Patients who have pre-eclampsia are at increased risk. When a patient is peripartum, this condition has to occur in the third trimester or within 5 months postpartum. Echocardiogram will show normal or decreased wall thickness, poor wall thickening in systole, left ventricular dilation in a spherical pattern, and diminished stroke volume. Patients who have this disorder tend to present with fatigue and dyspnea. The patient in this case has dilated cardiomyopathy caused by ethanol abuse. Malnourishment often accompanies severe alcoholism and implies thiamine deficiency, which can lead to heart disease (wet beriberi). Suspect this diagnosis in any alcoholic patient presenting with symptoms and signs of congestive heart failure. In this patient, an echocardiogram would be expected to reveal bilateral ventricular dilatation with impaired contraction throughout both chambers.

Although rheumatic fever is not commonly seen in people who are native to the United States, there should be continual surveillance for rheumatic fever because of our ability to travel to endemic areas and receive patients from those countries. Rheumatic fever has long-term consequences affecting the heart valves. Which of the following valvular abnormalities most commonly results from rheumatic fever? A. Tricuspid stenosis B. Aortic stenosis C. Pulmonic stenosis D. Mitral stenosis E. Aortic regurgitation

The correct answer is D. Rheumatic heart disease typically occurs 10 to 20 years after initial infection. The mitral valve is more commonly involved than the aortic valve, which is the second most common valve that is involved. While the initial manifestation of rheumatic heart disease is mitral regurgitation, mitral stenosis occurs with time as the mitral valve calcifies and becomes stenotic with time. About 50% of patients who develop carditis at the time of rheumatic fever go on to develop significant valvular involvement and heart murmur.

You are interpreting an EKG. You note that the patient's R wave in V1 is larger than the S wave in that same lead. You also note that the R waves decrease in size across the precordial leads. Which of the following other EKG findings would most likely be seen? A. Left axis deviation B. Left bundle branch block C. Q waves in inferior leads D. Right axis deviation E. Wide P waves in lead II

The correct answer is D. Right ventricular hypertrophy (RVH) typically has a large R wave in V1, with the R wave height being greater than the depth of the S wave. With right ventricular hypertrophy, the predominant axis can shift to the right as a result of the increased electrical vectors that occur with right ventricular activity. RVH can also result in right bundle-branch block because it will take longer for this hypertrophied muscle to contract.

Special maneuvers to identify specific cardiac abnormalities are an important part of the assessment of patients with cardiac conditions. In order to understand the effects of the different physical examination maneuvers, the clinician needs to know how these maneuvers alter blood flow to or from the heart. Clinicians also need to also know what additional factors play a role in affecting the various cardiac conditions. Which of the following maneuvers or medications will cause the murmur associated with hypertrophic cardiomyopathy to increase? A. Squatting B. Use of beta blocker C. Sustained hand grip maneuver D. Valsalva maneuver E. Lying down

The correct answer is D. The Valsalva maneuver interferes with venous return of blood to heart and makes the hypertrophic obstructive cardiomyopathy (HCM) murmur worse. The obstruction associated with HCM worsens with less blood in the heart itself, so anything that decreases the amount of blood in the heart will make the obstruction worse and put the patient at risk for sudden cardiac death.

An elderly man presents with complaints of dizziness, headaches, diarrhea, nausea and vomiting, weakness, palpitations, and a change in vision with a yellowish to blue tint to his vision. He is taking multiple medications. He has a history of chronic heart failure and hypertension. His wife states that he has had a few episodes of confused, delirious behavior over the past few weeks. Which of the following agents might be responsible for this man's symptoms? A. Allopurinol (Zyloprim) B. Hydralazine (Apresoline) C. Niacin D. Digoxin (Lanoxin) E. Spironolactone (Aldactone)

The correct answer is D. The collection of symptoms described includes dizziness, headaches, diarrhea, nausea and vomiting, weakness, palpitations, and a change in vision with a yellowish to blue tint are classic side effects of digoxin (Lanoxin). EKG changes, such as biventricular tachycardia, may also occur. Digoxin is a medication that has been shown to improve symptoms in patients who have heart failure, as it is a positive inotropic and negative chronotropic agent. Digoxin also lessens hospitalization for patients who have chronic heart failure but it does not improve mortality. Patients who are hypokalemic are especially prone to the development of digoxin toxicity.

In patients who have significant atherosclerosis, plaques can break off and cause occlusion. Which of the following is the most common site for acute arterial occlusion to occur? A. Ascending aorta B. Descending aorta C. Iliac artery D. Common femoral artery E. Popliteal artery

The correct answer is D. The most common site for arterial occlusion is in a common femoral artery. This anatomical location serves as a large bifurcation from the abdominal aorta, with the blood flow being turbulent here.

A 72-year-old woman with no prior medical history comes to the emergency department because of a 3-hour episode of crushing substernal chest pain. The pain radiates to her arm and neck. An electrocardiogram reveals ST-segment elevation in leads II, III, and aVF. The patient has no obvious contraindication to anticoagulation. Which of the following is the treatment that should be currently pursued? A. Avoidance of thrombolytic treatment, given the patient's age B. Administration of IV fluids C. Administration of aspirin and heparin only D. Administration of thrombolytic therapy, heparin, and aspirin E. Cardiac surgery to bypass the occluded vessel

The correct answer is D. The patient is having an acute ST-segment elevation inferior wall myocardial infarction. The infarct occurs as a result of an atherosclerotic plaque with thrombus formation, leading to complete coronary artery obstruction. Lysis with a thrombolytic agent has been shown to decrease mortality from early post-myocardial infarction. Aspirin prevents both platelet aggregation and reocclusion of the reperfused vessels. When aspirin and lytic therapy are given in the setting of ST-segment elevation MI, heparin is traditionally added to the regimen to stop new clots from forming.

A 45-year-old man suddenly loses consciousness and falls to the ground. His pulse is lost. He has been previously healthy and has been on no medication. There is no obvious evidence of trauma. Electrocardiogram reveals wide complex tachycardia at a rate of 200/min. CK-MB levels are elevated. Which of the following is the most appropriate intervention? A. Obtain vital signs B. Administer a bolus of intravenous lidocaine C. Administer a thrombolytic agent D. Perform defibrillation E. Perform synchronous cardioversion

The correct answer is D. The patient is in pulseless ventricular tachycardia and is hemodynamically unstable as illustrated by the loss of consciousness. Pulseless ventricular tachycardia is treated ini the same way as ventricular fibrillation. He should be emergently defibrillated with 200 joules of energy initially.

A patient is seen in the office, and examination of the heart reveals a diastolic murmur heard best at the second intercostal space to the right of the sternum. There is a wide pulse pressure noted on the patient's blood pressure. Risk factors for the development of this condition include all of the following EXCEPT: A. Male sex B. Long-standing syphilis C. Ankylosing spondylitis D. Long-standing hypertension E. Bicuspid aortic valve

The correct answer is D. The risk factors for aortic regurgitation include cystic medial degeneration, male sex (75%), bicuspid valve, and dilated aortic root, such as that seen with ankylosing spondylitis or syphilis. Long-standing hypertension is not a risk factor for aortic regurgitation.

A 56-year-old smoker with no previous medical history comes to the emergency department complaining of chest pain and shortness of breath with exertion. He is admitted to the hospital for further evaluation. An exercise stress test supports the diagnosis of coronary artery disease, and the patient undergoes a heart catheterization. There is diffuse coronary artery disease but no clearly stentable lesions. The cardiologist decides that medical management of this patient's coronary disease is appropriate at this time. The patient has no allergies and no other medical conditions. His blood pressure and pulse over 3 days are as follows: Day 1 bp 146/96 mm Hg, pulse 80/min Day 2 bp 150/90 mm Hg, pulse 86/min Day 3 bp 140/96 mm Hg, pulse 73/min Which is the most appropriate bp medication for this patient? A. Diltiazem (Cardizem, Dilacor, Cartia) B. Hydrochlorothiazide (Microzide) C. Lisinopril (Prinivil, Zestril) D. Metoprolol (Lopressor, Toprol-XL) E. Nifedipine (Adalat, Procardia)

The correct answer is D. There is clear and convincing evidence that beta-blockers are indicated in the treatment of patients who have coronary artery disease. This is partly because in addition to lowering blood pressure, beta-blockers keep the heart rate slow, which decreases strain on the heart by decreasing myocardial oxygen demand and increases myocardial perfusion. Because this patient has coronary artery disease and hypertension, the best choice for this patient is metoprolol. There is mortality benefit for patients with CAD who are given beta blocker therapy.

A 15-month-old boy is brought to the emergency department by his parents because he suddenly turned blue and had difficulty breathing while he was playing in the back yard of their home. The parents have noticed several times in the previous 2-3 months that he had developed a bluish discoloration around the lips but did not think much of it, as it was wintertime. Recently, however, the boy had increasing fatigability and would stop in the middle of playing to catch his breath by sitting down or squatting. This morning when he suddenly turned blue and started behaving in a very flustered manner, they rushed him to the emergency department. The parents deny any other significant medical conditions or allergies to medication. They have a 4-year-old daughter who is in good health. On physical examination, the patient is in the fifth percentile for height and weight. His lips and fingertips are bluish in color and he frequently stops to catch his breath while playing in the examination room. There is a hint of clubbing of his fingers. Vital signs are within normal limits, but a complete blood count shows a red blood cell count of 6 x 1012/L and a hematocrit of 66%. A chest radiograph shows a boot-shaped heart with an uptilted apex and clear lung fields. On auscultation, there is a harsh systolic ejection murmur and a single S2 is heard. Which of the following is the most likely diagnosis? A. Endocardial cushion defect B. Myocarditis C. Patent ductus arteriosus D. Tetralogy of Fallot E. Transposition of the great vessels

The correct answer is D. This boy has a congenital heart defect known as tetralogy of Fallot. Tetralogy of Fallot is defined as pulmonary stenosis, ventricular septal defect, dextroposition of the aorta (overriding), and right ventricular hypertrophy. It is the most common type of cyanotic congenital heart disease. Symptoms depend on the size of the ventricular septal defect and the degree of the right ventricular outflow tract obstruction. Acyanotic (pink) tetralogy occurs when there is sufficient pulmonary blood flow caused by mild obstruction and the shunting across the ventricular septal defect is balanced. Typically, however, patients present with cyanosis, delayed growth and development, and dyspnea. Paroxysmal hypercyanotic attacks (hypoxic, blue, or tet spells) manifest with episodes of restlessness, cyanosis, and gasping respirations. Clubbing of the fingers and toes occurs secondary to chronic hypoxia. A loud, harsh, systolic ejection murmur is heard. S2 is single or very soft because of the pulmonary stenosis. Chest radiographs reveal a boot-shaped heart (coeur en sabot) with uptilted apex. Lung fields are clear, reflecting decreased pulmonary blood flow. Electrocardiography shows right ventricular hypertrophy and right-axis deviation. An echocardiogram reveals the anatomic abnormalities. Medical management includes maintaining the ductus open in severe right-sided obstructive lesions, but surgical correction is the definitive treatment. Blue spells are treated by placing the child in knee-chest position, sedation, oxygen, and avoiding acidosis. Beta blockade with propranolol, appropriate fluid status, and maintaining the hematocrit at 55 to 65% are also helpful. Complications have become less frequent as the surgical correction is typically performed early. Cerebral thrombosis occurs with extreme polycythemia and dehydration. Patients are usually younger than 2 years of age. Brain abscess, although less common than thrombosis, is more often seen in patients older than 2 years of age. Patients are at higher risk for bacterial endocarditis.

A 2-month-old infant is evaluated by a pediatric specialist health care provider. The infant was noted at birth to have an upper left sternal border ejection murmur. The infant at that time was not cyanotic but slowly developed cyanosis over the next 2 months. At the time of the examination, an electrocardiogram showed right axis deviation and right ventricular hypertrophy. A chest radiograph showed a small heart with a concave main pulmonary artery segment and diminished pulmonary blood flow. Surgical repair is recommended. Which of the following is the most likely diagnosis? A. Complete atrioventricular canal defect B. Hypoplastic left ventricle C. Isolated atrial septal defect D. Tetralogy of Fallot E. Transposition of the great arteries

The correct answer is D. This is tetralogy of Fallot, in which severe obstruction of right ventricular outflow and a ventricular septal defect allow unoxygenated blood to pass from the right side of the heart to the left. In severe cases, cyanosis presents at birth; in milder cases (such as this infant has), it develops more slowly. The upper left sternal border ejection murmur is caused by right ventricle outflow obstruction. The ECG and chest radiographic findings described in the question stem are typical for older infants who have tetralogy of Fallot. Early surgical repair is now recommended for tetralogy of Fallot.

A 70-year-old hypertensive man arrives at the emergency department complaining of shortness of breath. His history is significant for chronic hypertension, paroxysmal nocturnal dyspnea, and nocturia. Physical examination reveals evidence of pulmonary and peripheral edema. The patient is admitted, and furosemide is administered. A low-sodium diet is ordered. The purpose of this dietary restriction is to A. decrease tubular reabsorption of sodium. B. increase extracellular water. C. increase intracellular water. D. reduce extracellular water. E. reduce intracellular water.

The correct answer is D. This hypertensive patient has symptoms and signs of congestive heart failure (paroxysmal nocturnal dyspnea, nocturia, and pulmonary and peripheral edema), for which dietary sodium restriction is recommended to reduce water retention. Most of the sodium in the body exists in locations outside cells, such as plasma, interstitial fluid, cerebrospinal fluid, intraocular fluid, semen, peritoneal fluid, and pleural fluid. The concentration of sodium in these fluids is tightly regulated by the kidney's action on blood plasma. When dietary sodium is severely restricted, the losses in the kidney (and small losses in sweat) exceed the intake, and the total body sodium is decreased. The body then tries to maintain serum sodium concentration by excreting more water via the kidneys. Most of the water comes from the extracellular fluid rather than the intracellular fluid (choice E), because sodium ion is not a quantitatively important component of intracellular fluid. Decreased dietary sodium will increase tubular reabsorption of sodium rather than decreasing it (choice A).

A 5-month-old girl is brought to the office by her mother, who states that the girl had an episode following feeding during which she began to breathe deeply, became blue, and then lost consciousness. The mother states that she picked her up and held her, and the infant regained her usual color and became alert. Physical examination reveals a harsh systolic murmur. The remainder of the physical examination is unremarkable. Which of the following is the most likely diagnosis? A. Aortic stenosis B. Coarctation of the aorta C. Patent ductus arteriosus D. Tetralogy of Fallot E. Ventricular septal defect

The correct answer is D. This infant is experiencing a hypoxemic spell, as seen in patients who have tetralogy of Fallot. These hypercyanotic spells, or "Tet spells," usually are self-limited and last less than 10-15 minutes. The spells often occur immediately after feeding or when the child is crying vigorously. Tetralogy of Fallot has the following components: (1) ventricular septal defect, (2) overriding aorta, (3) right ventricular hypertrophy, and (4) pulmonic stenosis.

A 4,000-g male neonate develops severe cyanosis that begins within minutes of birth. Blood drawn 1 hour after birth shows metabolic acidosis with respiratory acidosis. A chest radiograph shows a narrow mediastinum, narrow heart base, and absence of the pulmonary artery. An electrocardiogram is normal. An echocardiogram is ordered and the report is pending. Which of the following is the most likely diagnosis? A. Aortic valve stenosis B. Complete atrioventricular canal defect C. Tetralogy of Fallot D. Transposition of the great arteries E. Underdeveloped (hypoplastic) left ventricle syndrome

The correct answer is D. This is transposition of the great arteries, in which the aorta arises from the right ventricle and the pulmonary artery arises from the left ventricle. Approximately 5% of congenital cardiac anomalies have transposition of the great arteries. Affected infants present within minutes of birth with severe cyanosis and metabolic acidosis secondary to inability to oxygenate tissues. The only exchange of blood between pulmonic and circulatory systems is typically occurring through a patent ductus arteriosus. The chest radiograph changes include a narrow mediastinum, narrow heart base, and absence of the pulmonary artery; these are caused by superposition of the great vessels (rather than the normal side-to-side position). Surgical repair is usually performed within 7 to 10 days of life.

A 15-year-old boy is concerned about multiple stretch marks that have developed on his chest, shoulders, and thighs over the previous 3 to 4 years. He plays center on his high school basketball team and is annoyed by the appearance of the skin lesions. His medical history is remarkable for myopia, upward lens displacement, and mitral valve prolapse. He is quite tall for his age and has long, thin extremities and fingers. He is accompanied by his father, who has the same tall stature and thin extremities. The skin lesions this young man is worried about are multiple striae in the pectoral, deltoid, and thigh regions. His chest is flattened on either side with forward projection of the sternum, and he has long, tapering fingers and toes. Which of the following is the most likely cause of his condition? A. Cushing disease B. Exogenous androgen administration C. Hyperhomocysteinemia D. A genetic defect in the fibrillin gene E. Ehlers-Danlos syndrome

The correct answer is D. This patient has Marfan syndrome, an autosomal dominant inherited genetic defect of fibrillin. Abnormalities appear in 3 organ systems: the eye, the skeletal system, and the cardiovascular system. Skin manifestations may consist of striae (stretch marks). The skeletal features include long, narrow extremities and tapering fingers and toes. The patients are typically tall. They may also have pectus excavatum, pectus carinatum, flat feet, and hyperextensibility of the knees and other joints. The ocular findings include long orbits causing myopia, and ectopia lentis (with upward displacement in 70%) sometimes causing acute glaucoma because of drainage obstruction secondary to lens displacement. Cardiovascular defects consist of weakness of the aortal intima leading to aneurysms and mitral valve prolapse. Acute dissection of the aorta is a common cause of death in these patients.

A 22-year-old man comes to the emergency department with a 3-day history of fever, chills, a cough, pleuritic chest pain, and low back pain. He says that the symptoms came on "out of the blue." He is the son of a wealthy local businesswoman and still lives at home, which he says "is cool because my parents are never around." His temperature is 39°C (102.2°F), blood pressure 120/80 mm Hg, pulse 70/min, and respirations 16/min. Physical examination shows oval retinal hemorrhages with a clear, pale center and pinpoint lesions between his toes. Blood cultures are drawn. A chest radiograph shows multiple patchy infiltrates. Laboratory studies show hemoglobin 11 g/dL, hematocrit 39%, and erythrocyte sedimentation rate 39 mm/h. Which of the following is the most likely pathogen? A. Candida albicans B. Pseudomonas aeruginosa C. Serratia marcescens D. Staphylococcus aureus E. Streptococcus viridans

The correct answer is D. This patient has acute bacterial endocarditis, most likely caused by Staphylococcus aureus, the most common organism causing endocarditis in intravenous drug abusers. The "pinpoint lesions" between his toes are signs of injection drug abuse. Acute endocarditis in drug abusers typically presents with a high fever, pleuritic chest pain, and a cough. The tricuspid valve is commonly affected in these patients. A murmur may not be present in early acute endocarditis or in injection drug abusers who have tricuspid valve disease. Roth spots are oval, pale, retinal lesions that are surrounded by hemorrhages. Other physical manifestations of endocarditis include Janeway lesions, which are hemorrhagic, painless, macular plaques typically located on the palms and soles, and Osler nodes, which are small, painful nodular lesions typically found on the pads of the fingers or toes. Other findings include anemia and an elevated erythrocyte sedimentation rate. Diagnosis is with blood cultures, which are typically positive for S. aureus, and with echocardiography. Treatment is with antibiotics.

A 16-year-old boy who is a recent immigrant is brought to the urgent care clinic with a temperature of 38.4°C (101°F) and low back, wrist, and knee pain. He had a sore throat 1 month earlier. His arthritis is diffuse. Pea-sized swellings are noted over the skin on his knees. He has a serpiginous erythematous area on his anterior trunk. His blood and throat cultures are negative, and his CBC is unremarkable. His antistreptolysin-O (ASO) titer is high. Which of the following is the most appropriate therapy? A. Acetaminophen B. Aspirin C. Penicillin D. Penicillin and aspirin E. Supportive care

The correct answer is D. This patient has acute rheumatic fever from group A streptococci. He has migratory polyarthritis, erythema marginatum, and subcutaneous nodules. Other features absent in this patient are chorea and carditis. His ASO titer indicates recent infection with Streptococcus. It is advisable to administer penicillin for the infection. The arthritis can be managed with salicylates. The patient will need long-term treatment with penicillin as a result of his rheumatic fever.

The EKG of a 60-year-old man reveals widened QRS intervals of 0.14 seconds with distinctly abnormal configurations. Physical examination is significant for paradoxic splitting of the second heart sound. Which of the following valvular defects is likely in this patient? A. Mitral valve prolapse B. Mitral stenosis C. Pulmonic stenosis D. Aortic stenosis E. Aortic regurgitation

The correct answer is D. This patient has bundle branch block, as implied by the QRS interval greater than 0.12 seconds and by paradoxic splitting of the second heart sound. The typical sequence of valve closure is mitral, tricuspid, aortic, and pulmonic. If there is a paradoxic splitting of the second heart sound, the aortic valve closes after the pulmonic valve. This situation can occur with anything that slows the conduction in the left ventricle, such as complete heart block, left bundle branch block, or aortic stenosis. Left bundle branch block results in conduction occurring in a cell-to-cell depolarization pattern that slows the conduction through the left ventricle, which causes the aortic valve to close after the pulmonic valve.

A 57-year-old woman with a history of rheumatic fever as a child comes to the her health care provider complaining of a 6-month history of slowly progressive dyspnea on exertion and orthopnea. Her temperature is 37°C (98.6°F), blood pressure is 110/60 mm Hg, pulse is 93/min and irregular, and respirations are 18/min. Cardiac examination reveals a localized mid-diastolic murmur near the apex. There is a loud opening snap heard after S2. The rhythm appears irregular. Which of the following additional findings will most likely be present on physical examination? A. Delayed carotid upstroke B. De Musset sign C. Corrigan pulse D. Decreased S1 intensity E. Large A-wave

The correct answer is D. This patient has mitral stenosis. Most adults who have mitral stenosis have had rheumatic fever as a child, although not all patients are aware of having had this infection. Mitral stenosis decreases left ventricular filling and elevates left-sided atrial pressures. This causes pulmonary congestion and results in symptoms of left-sided heart failure, such as shortness of breath and dyspnea on exertion. Hemoptysis sometimes occurs as a result of rupture of small pulmonary blood vessels. Later in the course of the disease, patients may develop pulmonary hypertension and cor pulmonale because the right ventricle has to work against increased pressures secondary to chronic pulmonary congestion.

A 75-year-old man with angina pectoris has recurrent episodes of atrial tachycardia (240/min). A rapid sequence of normal QRS waves is seen on EKG. The episodes are controllable by the patient's performance of vagal maneuvers. Which of the following is the most likely etiology of this arrhythmia? A. Atrial reentry B. Automatic atrial conduction C. AV dissociation D. AV nodal reentry E. Wandering atrial pacemaker

The correct answer is D. This patient has paroxysmal supraventricular tachycardia (PSVT), which is a regular, rapid (150-250/min) arrhythmia originating in the atria or AV node. AV nodal reentry is the most common cause of this arrhythmia (about 70% of patients). In this condition, the AV node is pathologically divided into two functional pathways. The electrical impulse usually proceeds anterograde down the slow pathway and retrograde up the fast pathway. The P waves are recorded nearly simultaneously with the QRS complexes (which occur in rapid sequence) and are therefore obscured on EKG. This arrhythmia is commonly seen in older patients, about half of whom have underlying heart disease. Reentry PSVTs can be reverted to normal sinus rhythm by interrupting the reentry pathway. For example, the performance of vagal maneuvers often improves the condition by increasing AV nodal refractoriness.

A 50-year-old man is brought to the emergency department complaining of light-headedness. He has a history of lung cancer, which was diagnosed a month ago and found to be widely metastatic to the bone and pericardium. On physical examination his blood pressure is 70/40 mm Hg and pulse 100/min. Heart sounds are distant and soft. ECG demonstrates low voltage, and electrical alternans is present. Chest radiograph shows that the cardiac silhouette has a "water bottle" appearance. Which of the following is the most appropriate intervention? A. Beta-blockers B. Nonsteroidal anti-inflammatory drugs C. Steroids D. Pericardiocentesis E. Cardiac catheterization

The correct answer is D. This patient has pericardial/cardiac tamponade, most likely as a result of his malignancy. Lung cancer is particularly likely to cause pericardial effusions. Furthermore, because this patient has metastases to the pericardium, he might be bleeding into the pericardial space. This tamponade may be the cause of his significant hypotension and the soft cardiac sounds. Electrical alternans, a phenomenon in which the QRS changes axis, is indicative of pericardial effusion, because the heart is moving freely in the fluid, causing this change in axis noted on the ECG. Emergently, this patient needs decompression of the pericardial space with the aid of pericardiocentesis, in which a catheter directly drains the fluid in the pericardial sac.

A 67-year-old woman comes to the clinic for review of her medications. She has had a history of hypertension for 25 years, type 2 diabetes for 20 years, and congestive heart failure for 5 years. She is a former smoker of two packs of cigarettes per day and her lipid status is not known at this time. Her current medications include nifedipine (Adalat, Procardia), hydralazine (Apresoline), isosorbide dinitrate (Isordil), glyburide (DiaBeta, Micronase), a multivitamin, and conjugated estrogens. Today in the clinic her blood pressure is 160/90 mm Hg, her fasting blood glucose is 210 mg/dL, and her hemoglobin A1c is 7.9%. She reports moderate dyspnea on exertion, unchanged from previous visits. Which of the following is the most appropriate intervention at this time? A. Increase her glyburide dosage B. Increase her nifedipine dosage C. Discontinue her hydralazine therapy D. Discontinue the hydralazine and add captopril E. Begin simvastatin therapy

The correct answer is D. This patient has poorly controlled hypertension and poorly controlled diabetes. She needs improved therapy for both; the issue is how best to do that. The concept underlying this question is the absolute importance of using ACE inhibitor therapy on both type 1 and type 2 diabetic patients. Many clinical trials have shown the beneficial effects of ACE inhibitors on preventing nephropathy and slowing the progression of established nephropathy in diabetics. It is the standard of care that all diabetics be given an ACE inhibitor if they are able to tolerate its blood pressure effects. Given that she has congestive heart failure and hypertension, the ACE inhibitor also will be efficacious in their treatment. In fact, ACE inhibitors have been shown to be superior to hydralazine and isosorbide dinitrate in terms of morbidity and mortality in treatment of CHF; all patients who have symptomatic CHF, regardless of ejection fraction, should be placed on one.

A patient is seen in the office with complaints related to her lower extremities. She claims that the discomfort in her lower extremities is helped with elevation of legs, while recumbency and prolonged standing make the discomfort in her lower extremities worse. Which of the following conditions is most likely to cause this scenario? A. Leriche syndrome B. Acute arterial occlusion C. Cholesterol embolization syndrome D. Mycotic aneurysm E. Chronic venous insufficiency

The correct answer is E. Patients with chronic venous insufficiency have improvement in their symptoms with elevation of legs and worsening of their symptoms with prolonged standing and recumbency.

A 55-year-old man is brought to the emergency department because of increasing incoherence over the past 24 hours. He has a history of hypertension and diabetes. He is disoriented. On physical examination, he is afebrile. His blood pressure is 230/130 mm Hg, pulse is 120/min, and respirations are 24/min. He has an S4 on cardiac examination. The patient is placed on a cardiac monitor, and IV and intra-arterial lines are placed. A head CT scan shows no mass or bleed. Which of the following is the most appropriate next step in management? A. Observe in a quiet room B. Check chemistry for an anion gap C. Administer a diuretic D. Administer labetolol (Trandate) E. Perform a lumbar puncture

The correct answer is D. This patient is having a hypertensive emergency, given the presence of end-organ damage (mental status changes) in the setting of hypertension. Hypertensive encephalopathy allows the definition of hypertensive emergency rather than hypertensive urgency to be used. Immediate therapy is needed, before the laboratory results are known. This requires immediate but not precipitous lowering of the blood pressure over a period of minutes to hours. Labetolol acts as a combined alpha- and beta-blocker and will decrease systemic vascular resistance, mean arterial pressure, and heart rate with minimal change in cardiac output. It is given intravenously with an onset of action in 5 to 10 minutes with a duration of action of 3 to 8 hours.

Which of the following medications used in the setting of non-STEMI (NSTEMI) and unstable angina acts as a glycoprotein IIb/IIIa inhibitor? A. Fondaparinux (Arixtra) B. Enoxaparin (Lovenox) C. Prasugrel (Effient) D. Tirofiban (Aggrastat) E. Bivalirudin (Angiomax)

The correct answer is D. Tirofiban is an antiplatelet medication that binds to platelet glycoprotein IIb/IIIa receptors, which interferes with the final stage of platelet binding.

Which of the following is an advantage of tissue plasminogen activator (tPA) over streptokinase for fibrinolytic therapy? A. It can be used in the setting of acute myocardial infarction B. It cannot cause hemorrhage C. It is less expensive D. It is not likely to produce an allergic reaction E. It results in the activation of plasminogen

The correct answer is D. Tissue plasminogen activator is produced by and secreted from endothelial cells. Because it is not a foreign protein like streptokinase (derived from hemolytic streptococci), tPA is not allergenic.

A 71-year-old man comes to the hospital with an episode of bright red blood per rectum. The patient reports that, a few hours ago, he passed a grossly bloody bowel movement. The passage was associated with some cramping lower abdominal pain. The patient's past medical history is significant for coronary artery disease and a myocardial infarction 3 years ago. He is poorly compliant with his beta-blocker and diuretic therapy, and his blood pressures have run around 140/85 mm Hg. While the patient is in the hospital, he has another episode of large volume, bright red blood per rectum. His blood pressure is 100/60 mm Hg while supine, and his pulse is 120/min. His hematocrit is 28%. He then begins to complain of substernal chest tightness radiating to his left shoulder. An electrocardiogram shows new T-wave inversions in the anterior leads. Which of the following is the most appropriate intervention at this juncture? A. Administration of aspirin by mouth B. Administration of a beta-blocking agent C. Administration of nitroglycerin sublingually D. Administration of nitroglycerin topically E. Blood transfusion

The correct answer is E. According to the available data, the patient is presenting with myocardial ischemia (as manifested by his angina) in the presence of anemia, which is caused by his acute bleeding secondary to diverticulosis. The appropriate treatment, therefore, is a blood transfusion, because he is not tolerating the acute blood loss.

A patient is complaining of some mild dyspnea upon exertion. Examination of her heart reveals a diastolic rumbling murmur located at the apex of the heart, which is best heard with the patient in the left lateral decubitus position. Which of the following is the most important treatment for this patient? A. Mitral valve commissurotomy B. Percutaneous balloon valvuloplasty C. Replacement of the mitral valve D. Diuretics E. Repair of ventricular septal defect

The correct answer is D. Treat mildly symptomatic mitral stenosis with diuretics and antithrombotic medications. Diuretics are helpful in the setting of pulmonary congestion, with loop diuretics being the most effective. Statins are also first-line therapy for mitral stenosis as they help to lower cardiovascular risks in the setting of mitral stenosis.

Patients with heart valve abnormalities can have auscultatory abnormalities when they are examined. Which of the following heart abnormalities has a blowing holosystolic murmur at the lower left sternal border that increases with inspiration? A. Tricuspid stenosis B. Aortic stenosis C. Pulmonic stenosis D. Tricuspid regurgitation E. Pulmonic regurgitation

The correct answer is D. Tricuspid regurgitation is a holosystolic murmur that increases with inspiration. There is increased venous return of blood to the right side of the heart with inspiration, and this will make the holosystolic murmur of tricuspid regurgitation louder and more prominent.

Certain conditions can affect the vasculature of the body. Which of the following is the treatment of choice for women who develop Takayasu (pulseless) disease? A. ACE inhibitors B. Beta-blockers C. Direct renin inhibitors D. Glucocorticoids E. Colchicine

The correct answer is D. Use glucocorticoids to treat Takayasu disease.

A patient sustains trauma to the anterior chest during a motor vehicle collision. The car was an older model that did not have air bags, and the patient's chest impacted on the steering wheel during the collision. Bleeding occurs between the pericardial layers as a result of this trauma. What is the complication of pericardial effusion that is caused by a rapid accumulation of the pericardial fluid? A. Myocardial rupture B. Left ventricular rupture C. Atrial fibrillation D. Rupture of mitral valve papillary muscle E. Cardiac tamponade

The correct answer is E. Cardiac tamponade is a known complication of patients with pericardial effusion who develop rapid fluid accumulation. With this rapid influx of fluid between the pericardial layers, there is compression of the heart because of the inability of these pericardial membranes to dilate. As more fluid accumulates, the heart becomes compressed and is unable to fill, with resultant inability to provide blood to the rest of the body.

Patients having complications of acute myocardial infarction several days later can have sudden loss of blood pressure and loss of consciousness as a result of free wall rupture. Which of the following EKG findings is most likely to be seen in the setting of ventricular free-wall rupture following STEMI? A. Asystole B. Ventricular fibrillation C. Pulseless ventricular tachycardia D. Complete heart block E. Pulseless electrical activity

The correct answer is E. Pulseless electrical activity (PEA) occurs as a complication of STEMI with free wall rupture along with loss of BP and loss of consciousness. PEA results in retained electrical activity, but there is no actual blood being mechanically moved when free wall rupture occurs. This condition is uniformly fatal unless immediate clinical intervention is performed.

A previously healthy 45-year-old woman is in a motor vehicle accident. She suffers multiple internal and external injuries from which she is still actively bleeding when reached by an ambulance. Her blood pressure is 50/20 mm Hg. Which of the following would be the most likely result of a complete blood count performed in the emergency department 15 minutes later, if the normal range for this test is 4.2 to 5.4 million red blood cells/uL in the female population and 5,000-10,000 white blood cells/uL? A. RBC 1.5 million per microliter; WBC 2,000 per microliter B. RBC 1.5 million per microliter; WBC 6,000 per microliter C. RBC 2.5 million per microliter; WBC 3,000 per microliter D. RBC 2.5 million per microliter; WBC 6,000 per microliter E. RBC 4.5 million per microliter; WBC 6,000 per microliter

The correct answer is E. These are the only values within normal limits and are the correct choice. The complete blood count (or alternatively the hematocrit or hemoglobin) cannot be used to assess the size of an acute blood loss, because intravascular volume and blood are both lost as a result of trauma. The reason is that the hematocrit or red blood cell count will drop only as fluid from other body sources (mostly extracellular fluid, with a smaller shift of intracellular fluid to extracellular) enters the capillary beds in response to a decreased intracapillary blood pressure. This takes at least a few hours to happen. When fluid resuscitation is given, the hemoglobin and hematocrit and red blood cell counts will dramatically fall, because they will be hemodiluted out. Low or dropping blood pressure is consequently a much better initial indicator of significant hemorrhage.

A 21-year-old professional dancer complains of several episodes of near loss of consciousness during a performance. She has been in excellent health and is a principal dancer in the New York City Ballet Corps. She has no family history of coronary artery disease. She does not smoke, and a recent cholesterol profile was normal. On physical examination, her blood pressure is 142/88 mm Hg and her pulse is 84/min and regular. She has a brisk carotid upstroke with a double impulse palpable. She has a loud S4 and a harsh systolic murmur heard along the left sternal border. The murmur is accentuated during the Valsalva maneuver and when she stands from a squatting position. An electrocardiogram reveals severe left ventricular hypertrophy. Which of the following is the most appropriate medication in the management of this patient? A. Captopril (Capoten) B. Digoxin (Lanoxin) C. Diltiazem (Cardizem) D. Furosemide (Lasix) E. Metoprolol (Lopressor)

The correct answer is E. This patient has the presentation of hypertrophic cardiomyopathy, a frequent cause of syncope or near syncope in young patients. She has a systolic ejection murmur that is heard at the lower left edge of the sternum, which is exacerbated by exercise and standing and lessened by lying supine or squatting. This murmur can be distinguished from other systolic murmurs by its increase with the Valsalva maneuver or standing from a squatting position. Any maneuver that acts to decrease venous return of blood to the heart and left ventricular size will increase the murmur of hypertrophic cardiomyopathy, because the obstructive component increases as the left ventricular cavity shrinks.

Recommendations for adding ACE inhibitors in the management of chronic stable angina include all of the following EXCEPT: A. Depressed left ventricular (LV) ejection fraction of <40% B. Patient who is also diabetic C. Patient with hypertension D. Patient with chronic kidney disease E. Patient with hyperlipidemia

The correct answer is E. ACE inhibitors have no real indication in a patient who only has hyperlipidemia, as these agents do not affect the lipid profile.

Patients who have hypertension may become pregnant, and patients who are pregnant may become hypertensive during pregnancy. There are a variety of agents used in the management of hypertension during pregnancy. Antihypertensive agents used during pregnancy include all of the following EXCEPT: A. Labetalol (Trandate) B. Methyldopa (Aldomet) C. Nifedipine (Procardia) D. Hydralazine (Apresoline) E. Lisinopril (Zestril)

The correct answer is E. ACE inhibitors like lisinopril are contraindicated for use during pregnancy because of the drop of glomerular filtration pressures. These agents are considered to being teratogenic for kidney development in the second trimester.

Patients with congenital heart disease may have physical examination signs associated with this condition. Which of the following congenital heart conditions will cause delayed and weakened femoral pulses when compared to upper extremity pulses? A. Patent ductus arteriosus B. Aortic regurgitation C. Ventricular septal defect D. Atrial septal defect E. Coarctation of the aorta

The correct answer is E. Coarctation of the aorta causes delayed and weakened femoral pulses due to the narrowing of the aorta, which interferes with blood flow distal to the area of narrowing.

A patient is evaluated for chest pain and dyspnea. Examination reveals significant changes in the person's pulse. An EKG is ordered, revealing low limb voltage. Which of the following cardiac conditions is associated with the development of pulsus alternans and low limb voltage? A. Acute inferior wall STEMI B. Acute anterior wall STEMI C. Restrictive cardiomyopathy D. Dilated cardiomyopathy E. Cardiac tamponade

The correct answer is E. Acute cardiac tamponade can cause low limb voltage and pulsus alternans from the mechanical swinging of the heart, which occurs as a result of a large pericardial effusion. Cardiac tamponade is a known complication of patients with pericardial effusion who develop rapid fluid accumulation. With this rapid influx of fluid between the pericardial layers, there is compression of the heart because of the inability of these pericardial membranes to dilate. As more fluid accumulates, the heart becomes compressed and is unable to fill, with resultant inability to provide blood to the rest of the body.

Pharmacologic stress tests can be utilized to assess coronary artery disease for patients who are not able to exercise. IV adenosine (Adenocard) and dipyridamole (Persantine) are pharmacologic stress agents that primarily work by which of the following mechanisms? A. Increase patient heart rate B. Increase myocardial contraction C. Increased systemic vascular resistance D. Increased vascular volume, which increases diastolic bp E. Coronary artery vasodilation

The correct answer is E. Adenosine and dipyridamole work by causing coronary artery vasodilation. These agents are utilized to produce coronary vasodilation and increase blood flow. In the setting of coronary disease where there is a plaque blocking this artery, increased blood flow into the heart cannot occur and this would indicate coronary artery disease. Both of these agents are used with thallium for imaging of the heart to assess whether there are any areas of poor perfusion.

A patient in the office develops acute dyspnea and palpitations. The patient is placed on the monitor and supraventricular tachycardia is found. Adenosine is prepared for delivery. All of the following adverse effects are associated with the use of this agent EXCEPT: A. Headache B. Flushing C. Chest pressure D. Nausea E. Diarrhea

The correct answer is E. Adenosine causes headache, flushing, chest pressure, nausea, and dyspnea, not diarrhea. Its mechanism of action is to stop conduction through the AV node. The intention of this medication is to cause the fast heart rate to be stopped with the hope that the SA node will reset and cause normal sinus to pick up after conduction stops through the AV node.

A 68-year-old Latin woman is admitted to the medical service for evaluation of her difficult to control hypertension. She has been treated for hypertension for 3 years by her primary care health care provider. She has a documented intolerance to ACE inhibitors manifested by a rapid decline in her renal function. She also has had two episodes of acute pulmonary edema in the past. She has ceased smoking, has altered her diet, and claims she is compliant with her medications. At her last office visit, a mildly elevated creatinine of 1.2 mg/dL was noted, as was microscopic hematuria. Physical examination is remarkable for a blood pressure of 180/100 mm Hg, a prominent apical impulse, and an abdominal examination that reveals audible bruits. Laboratory studies are as follows: Hematocrit 38% (41-50%) Leukocyte count 5,800/mm3 (5,000-10,000/mm3) Sodium 144 mEq/L (135-145 mEq/L) Potassium 3.6 mEq/L (3.5-5.2 mEq/L) Bicarbonate 28 mEq/L (20-29 mEq/L) Urea nitrogen (BUN) 22 mg/dL (7-20 mg/dL) Creatinine 1.3 mg/dL (0.6-1.2 mg/dL in females) Which of the following is the most likely cause of this patient's hypertension? A. Aldosterone-secreting tumor B. Coarctation of the aorta C. Essential hypertension D. Pheochromocytoma E. Renal artery stenosis

The correct answer is E. Although essential hypertension is the most common cause of hypertension, accounting for approximately 91% of all hypertensive diagnoses, in this case there are signs and symptoms suggestive of another diagnosis. For this patient, her sensitivity to ACE inhibitors, her elevated creatinine, abdominal bruits, and congestive heart failure all strongly suggest some degree of renal artery stenosis.

A patient is seen in the emergency department with chest pain, ST-segment elevation in anatomically contiguous leads, and positive troponin levels. The patient's lungs are clear, and the patient is not dyspneic. Which of the following auscultatory findings is most likely to occur in the setting of acute STEMI without co-existing heart failure? A. Widely split S2 B. Paradoxically split S2 C. S3 gallop D. Fixed split S2 E. S4 gallop

The correct answer is E. S4 gallop occurs in the setting of acute STEMI from increased atrial contraction against increased ventricular resistance.

A 61-year-old man is brought to the emergency department for chest pain. The patient has a long history of coronary artery disease and is status post-coronary bypass procedure 6 years ago. The patient has chronic stable angina that is usually precipitated by activity and relieved by rest. About 3 weeks ago, his health care provider prescribed sildenafil (Viagra), and he has been using the drug with success. This morning he developed acute onset of substernal chest pain radiating to his left arm. This pain is not relieved by rest. The patient last took a sildenafil (Viagra) the night before. Which of the following treatments is absolutely contraindicated in this situation? A. Aspirin (ASA) B. Captopril (Capoten) C. Metoprolol (Lopressor) D. Morphine E. Nitroglycerin F. Tissue plasminogen activator (tPA)

The correct answer is E. Although nitrate therapy is typically one of the cornerstones of treatment for cardiac ischemia, the co-administration of nitrates within 24 hours after taking sildenafil is absolutely contraindicated. The vasodilatory effects of nitrates are profoundly amplified when administered in the presence of sildenafil, which can lead to refractory and life-threatening hypotension and cardiovascular collapse because of a synergistic effect. Patients using sildenafil therefore should be instructed to report their use on presentation to any emergency department and to never take nitrates while using the drug.

Patients taking amiodarone on a long-term basis can have all of the following adverse reactions EXCEPT: A. Thyroid abnormalities B. Pulmonary fibrosis C. Corneal deposits D. Bluish tint to the skin E. Chronic renal failure

The correct answer is E. Amiodarone does not cause damage to the kidneys.

Various treatment strategies may be employed as part of the treatment for acute pericarditis. Which of the following medications is typically contraindicated in the management of acute pericarditis? A. Aspirin B. Colchicine C. Indomethacin (Indocin) D. Pericardial catheter drainage E. Warfarin (Coumadin)

The correct answer is E. Anticoagulants such as warfarin are relatively contraindicated in the setting of acute pericarditis due to risk of pericardial hemorrhage.

A patient is seen in the emergency department with syncope and is found to have bradycardia. Clinical evaluation strategies that are commonly performed in this setting consist of all of the following EXCEPT: A. Holter monitor B. 30-day loop recorder C. Long-term implanted monitor D. Continuous EKG monitoring E. Invasive electrophysiologic testing

The correct answer is E. As part of the evaluation strategy, patients do not need invasive electrophysiologic testing as part of the evaluation of bradycardia.

A patient is seen in the office with headache, palpitations, and diaphoresis that occurs spontaneously. During these episodes, the patient is noted to have orthostatic hypotension. The patient now is being treated for long-standing hypertension, which is not responding to conventional medications. Which of the following is the most appropriate next step in his evaluation if he is evaluated for a potential secondary cause of hypertension? A. Serum electrolytes B. Serum creatinine level C. Midnight salivary cortisol levels D. Chest x-ray E. Plasma metanephrine level

The correct answer is E. Assess plasma metanephrine or urinary catecholamine levels for patients suspected of having pheochromocytoma as the initial diagnostic study. CT is performed after the diagnosis to localize this tumor.

A patient sustains an acute myocardial infarction and the clinician is deciding which agent to use for maximum LDL cholesterol lowering. Among the following HMG Co-A reductase agents, which of the following is considered to be the most potent? A. Fluvastatin (Lescol) B. Lovastatin (Mevacor) C. Simvastatin (Zocor) D. Pravastatin (Pravachol) E. Atorvastatin (Lipitor)

The correct answer is E. Atorvastatin and rosuvastatin (Crestor) are the most potent statins available and have the potential to lower the LDL cholesterol levels to the greatest extent on a milligram-to-milligram basis.

A patient is seen in the hospital after sustaining a STEMI. The patient is found to have accelerated idioventricular rhythm while being monitored. The patient is also developing hypotension. Which of the following is the treatment for this patient? A. Amiodarone bolus followed by drip B. IV normal saline push C. ACE inhibitor to act as vasodilator D. Dopamine E. Atropine

The correct answer is E. Atropine is indicated for managing hypotension associated with accelerated idioventricular rhythm. This treatment will improve the rate of the idioventricular rhythm and will also increase the blood pressure as a result of its anticholinergic activity.

Which of the following medications used in the setting of non-STEMI (NSTEMI) and unstable angina acts as a direct thrombin inhibitor? A. Fondaparinux (Arixtra) B. Abciximab (Reopro) C. Prasugrel (Effient) D. Tirofiban (Aggrastat) E. Bivalirudin (Angiomax)

The correct answer is E. Bivalirudin is an anticoagulant that directly, reversibly inhibits thrombin. It is used for patients who have unstable angina and undergo percutaneous transluminal coronary angioplasty and in those who have a history of heparin-induced thrombocytopenia needing anticoagulation.

Which of the following mechanisms of action describes the activity of dihydropyridine calcium-channel blockers in the management of hypertension? A. Decrease force of ventricular contraction B. Decrease renin production C. Dilate the coronary arteries D. Slow the heart rate E. Vasodilate the arterial vasculature

The correct answer is E. Calcium-channel blockers cause arterial vasodilation, which lowers systemic vascular resistance, resulting in the lowering of blood pressure.

A patient is seen for a follow-up appointment in the office in order to obtain results of a continuous loop recorder. Analysis of this recorder reveals that the patient had two runs of nonsustained ventricular tachycardia that occurred while he was sleeping. The loop recorder was in place for a one-month period. The patient was asymptomatic during these episodes. The patient has no history of underlying coronary artery disease, hypertension, hyperlipidemia, or diabetes. Which of the following is the recommended treatment for this patient given the continuous loop recorder results? A. ICD placement (implantable cardiac defibrillator) B. Amiodarone C. Procainamide D. Permanent pacemaker with overdriving capacity E. No further treatment

The correct answer is E. Do not treat nonsustained VT in patients without underlying heart disease. Risk stratification is an important element in determining the appropriate treatment of a patient with nonsustained ventricular tachycardia. Patients who are asymptomatic during the periods of nonsustained ventricular tachycardia and who do not have underlying cardiac disease should adopt healthy lifestyles but should not have additional therapy. It should be remembered that antiarrhythmic medications also have the potential to cause arrhythmias, so these medications should be used for appropriate indications.

Which of the following signs or symptoms is most likely to be an early finding in the setting of pulmonary hypertension? A. Central cyanosis B. Peripheral cyanosis C. Peripheral edema D. Anemia E. Exertional dyspnea

The correct answer is E. Exertional dyspnea, fatigue, angina, and syncope are seen with pulmonary hypertension.

An adult with a long-standing ventricular septal defect (VSD) is seen in the office. Prior to performing the physical examination, the health care provider thinks about the expected historical findings for this patient. Which of the following signs or symptoms is most commonly seen in a symptomatic patient with a VSD? A. Jugular venous distention B. Chest pain C. Peripheral edema D. Palpitations E. Fatigue and reduced exercise tolerance

The correct answer is E. Fatigue and poor exercise tolerance are seen with VSD as a result of the heart having to move additional blood from the left ventricle into both the systemic vascular circulation and into the right ventricle. As this occurs, there is less blood traveling to the systemic circulation, which limits the amount of blood going into the exercising muscles.

A patient is seen in the office after being diagnosed with hypertrophic cardiomyopathy (HCM). Which of the following signs or symptoms is typically associated with this condition? A. Right ventricular hypertrophy B. Hyperkinetic pulse C. Increased susceptibility to atrial fibrillation as the most common heart dysrhythmia D. Diastolic murmur heard best in apex E. Murmur that worsens with Valsalva maneuver

The correct answer is E. HCM produces a systolic murmur that worsens with Valsalva, as there is decreased venous return to the heart with resultant less blood. When the heart has less blood, there is an increase in the obstruction. HCM also causes pulsus bisferiens along with crisp carotid upstroke as the heart initially ejects blood from the left ventricle until the site of obstruction is encountered.

A 29-year-old man is brought to the emergency department in a comatose state a few hours after complaining of sudden onset of excruciating headache. His friend does not know if the patient has any underlying medical conditions. Neurologic examination reveals dilated pupils poorly responsive to light. A CT scan of the head without contrast demonstrates hyperdensity within the suprasellar cistern, whereas MRI scan is unremarkable. Lumbar puncture shows hemorrhagic cerebrospinal fluid. Which of the following is the most likely diagnosis? A. Amyloid angiopathy-related hemorrhage B. Cavernous sinus thrombosis C. Hemorrhagic infarction D. Pituitary apoplexy E. Ruptured berry aneurysm

The correct answer is E. Headache of sudden onset ("thunderclap" headache), rapid deterioration of mental status, and blood in the CSF are virtually diagnostic of ruptured berry aneurysms. Note the characteristic hyperdensity on CT scan of the suprasellar cistern, indicating blood in the subarachnoid space. Rupture of a berry aneurysm is the most common cause of subarachnoid bleeding. Berry aneurysms develop as a result of congenital weakness at branching points of the arteries in the circle of Willis. These outpouchings tend to expand progressively, but in most cases they remain asymptomatic. Hypertension facilitates development and rupture of berry aneurysm. One third of patients recover, one third die, and one third develop re-bleeding. Rapid onset of coma is an ominous sign.

Which of the following electrolyte abnormalities is most likely to occur when ACE inhibitors or angiotensin II receptor blockers (ARBs) are used in order to manage hypertension? A. Hyponatremia B. Hypocalcemia C. Hypercalcemia D. Hypochloremia E. Hyperkalemia

The correct answer is E. Hyperkalemia occurs in the setting of ACE inhibitor or ARB use, as these agents drop the glomerular filtration pressure, resulting in less clearance of potassium.

Following an acute myocardial infarction, a patient develops acute dyspnea, distended neck veins, and chest pain. Lungs examination reveal rales throughout the lung fields. Patients with cardiogenic shock need to have prompt therapy for this condition. When managing this patient, which of the following therapies should be avoided? A. Dopamine B. Dobutamine C. Norepinephrine D. Loop diuretics E. IV fluid resuscitation

The correct answer is E. IV fluids should be avoided in the setting of cardiogenic shock because ventricular pressures are already elevated. Treatment of cardiogenic shock consists of unloading the heart with shrinking of volume with diuretics rather than providing additional fluid.

A patient is seen in consultation following an acute STEMI. The patient is presently pain free. The monitor in the hospital reveals that the patient is in an idioventricular rhythm. Which of the following clinical interventions/treatments is indicated? A. Immediate cardioversion B. Immediate defibrillation C. Amiodarone D. Epinephrine E. No treatment is necessary

The correct answer is E. Idioventricular rhythm is a life-saving rhythm following STEMI. This is a compensatory rhythm that takes over the pacemaking function of the heart, and there should be no attempt at interfering with this compensatory mechanism. In fact, if antiarrhythmic medications such as lidocaine are given, there is a concern that this medication will result in suppression of this protective mechanism. Idioventricular rhythm is expected post MI, and no treatment is indicated in a stable person.

An obese patient is seen in the office for follow-up for potential hypertension. He is concerned that the blood pressure cuff in the office is too small for him, which he knows will falsely elevate his blood pressure. For correctly sizing the cuff, the bladder of the cuff should encircle what percentage of the circumference of the arm? A. 30% B. 40% C. 50% D. 70% E. 80%

The correct answer is E. In order to obtain the most accurate blood pressure (BP), the BP cuff should be appropriately sized. If the BP cuff is too small for the arm, the blood pressure reading may be falsely elevated, with a too-large BP cuff falsely lowering the actual blood pressure. For appropriate sizing of the BP cuff, the bladder of the BP cuff should encircle 80% of the diameter of the arm.

Certain features of an EKG can suggest enlargement of the atria or hypertrophy of the ventricles. Since EKGs measure electrical voltages, actual changes in the muscle mass of these structures is more likely to be confirmed with echocardiography. Which of the following EKG findings would NOT be associated with right ventricular hypertrophy on an EKG? A. Right-axis deviation B. V1 have R wave height greater than depth of S wave C. Deep S wave in V6 D. R wave in V1 larger than 5 mm in height E. Presence of right bundle branch block pattern in V1

The correct answer is E. In the setting of right bundle branch block, right ventricular hypertrophy cannot be diagnosed. Right bundle branch block will upset the repolarization pattern and negates the ability to diagnose right ventricular hypertrophy.

When the right ventricle has to increase its inotropic activity in order to overcome increased pulmonary pressures, cor pulmonale develops. Patients with cor pulmonale may go on to develop which of the following heart abnormalities? A. Fixed split S2 B. Pulmonic stenosis C. Mitral stenosis D. Mitral regurgitation E. Tricuspid regurgitation

The correct answer is E. Increased pulmonary pressures leads to the development of cor pulmonale. When this condition occurs, resultant dilation of right ventricle will result in the development of tricuspid regurgitation.

What is the most common type of myocardial infarction (MI) that results in the development of second-degree type 1 heart block? A. Anterior wall B. Septal wall C. Anterolateral wall MI D. Lateral wall MI E. Inferior wall

The correct answer is E. Inferior wall MI can lead to the development of second-degree type 1 heart block, as can increased vagal tone and use of beta-blockers and digoxin. The inferior wall of the right ventricle is supplied by the right coronary artery, and infarction of this vessel will knock out the pacemaking activity of the heart.

A patient is seen in the office complaining of chest pain. Which of the following signs or symptoms is NOT typically associated with ischemic heart pain as a clinical feature for this condition? A. Chest heaviness B. Chest pressure C. Squeezing D. Tightness E. Sharp pain

The correct answer is E. Ischemic coronary disease pain is not typically described as sharp or stabbing, but rather this pain is more visceral and typically described as heaviness or pressure and is most concerning if this patient has diaphoresis.

A patient is seen who has a history of congenital heart disease. The patient developed secondary hypertension as a result of this congenital issue. Blood pressure is significantly higher in the upper extremity compared to the lower extremity. There is a "3" sign noted in chest film. Where is this murmur classically best heard? A. Second intercostal space to the right of the sternum B. Second intercostal space to the left of the sternum C. Erb's point D. Apex of the heart E. Back

The correct answer is E. It is best to hear the murmur of coarctation of aorta in the back, as this narrowing distal to the left subclavian artery is less obscured when listening to the posterior thorax.

A 63-year-old white man who has recently retired from work as a plumber for over 30 years returns to his health care provider saying that he has been feeling very down lately, and has been having decreased appetite and a loss of interest in activities that used to give him pleasure. He is a smoker, drinks no alcohol, and is being treated by his health care provider for moderate essential hypertension. Physical examination is unchanged from his previous visits. Which of the following is the most appropriate next step in management? A. Discuss activities that will help him enjoy his retirement B. Order a thyroid-stimulating hormone level (TSH) C. Order electroconvulsive therapy (ECT) D. Prescribe an antidepressant E. Review the patient's medication history

The correct answer is E. Many medications used to control hypertension, such as propranolol, and in the past, reserpine, are known to occasionally lead to depressive symptoms. By evaluation of the patient's medication record, the physician can evaluate which medications were started at what time and can make adjustments to dosage or switch to alternative medications to control hypertension. Changing the antihypertensive medication will possibly improve the depressive symptoms without the need to start an antidepressant (choice D).

A patient is evaluated in the office with a known history of peripheral vascular disease. The patient's history reveals that he is having intermittent claudication, which is relieved by rest. On physical examination, the patient also has loss of hair along with thickened toenails. The ankle-brachial index is 1.4. What is the significance of this ankle-brachial index for this patient? A. It predicts that this patient will have rest pain. B. It predicts that this patient already has an arterial ulcer. C. This is a normal finding, and the patient does not have chronic arterial insufficiency. D. This finding reflects chronic venous insufficiency and not arterial insufficiency. E. This finding occurs because the arteries are not compressible, and there is increased concern for severe arterial insufficiency.

The correct answer is E. Patients who have signs and symptoms of peripheral vascular disease with an unexpected elevation in their ankle-brachial index have had a false-negative result. The elevation in the ankle-brachial index means that the peripheral arteries are noncompressible vessels, and this clinical condition is significant for peripheral arterial disease.

Which of the following medications is the most appropriate choice for the treatment of hypertension during pregnancy? A. Lisinopril (Prinivil, Zestril) B. Atenolol (Tenormin) C. Verapamil (Calan, Isoptin, Verelan) D. Furosemide (Lasix) E. Methyldopa (Aldomet)

The correct answer is E. Methyldopa, an alpha-2 adrenergic agonist, decreases blood pressure by decreasing peripheral vascular resistance. It is the drug of choice for treating hypertension during pregnancy that may lead to pre-eclampsia, which is the most common cause of maternal and fetal morbidity and mortality. It acts on the central circulation as an alpha-agonist. It is listed as category B for use in pregnancy, which maintains that it has been proven safe for use in the first trimester of pregnancy.

Various heart valve abnormalities can be identified by auscultation of the heart. Which of the following heart valve abnormalities causes a holosystolic murmur at the apex that radiates into the back or clavicular, axillary area? A. Mitral stenosis B. Aortic stenosis C. Pulmonic stenosis D. Tricuspid stenosis E. Mitral regurgitation

The correct answer is E. Mitral regurgitation is a holosystolic murmur at the apex radiating into the back or to the axilla. This radiation occurs because this is the direction of the blood flow.

A patient is admitted to the hospital with signs and symptoms consistent with acute coronary syndrome (ACS). The patient has a clinical evaluation that reveals nondiagnostic EKG findings along with normal serial troponin levels. The patient has no recurrent symptoms over the next 12 hours. Which of the following is now indicated? A. Beginning a beta-blocker B. Beginning a calcium-channel blocker C. Beginning an ACE inhibitor D. Referral for percutaneous coronary intervention (PCI) E. Referral for stress test and consideration of left ventricular (LV) function

The correct answer is E. Patients with ACS symptoms should have stress test and LV evaluation prior to discharge home. Patients with clinical histories consistent with ACS who do not have a clear answer to explain these symptoms should have appropriate stress testing performed in order to rule out coronary artery disease (CAD). The most common killer in this country is CAD, so patients should have intervention performed when they present in a high-risk situation.

A 32-year-old woman comes to the hospital for an elective repeat cesarean delivery. Four years ago she had a primary cesarean delivery for a nonreassuring fetal heart rate tracing. Two years ago she chose to have an elective repeat cesarean delivery rather than attempt a vaginal birth after cesarean (VBAC). Her prenatal course was uncomplicated except that she has mitral valve prolapse. An echocardiograph demonstrated the mitral valve prolapse but no other structural cardiac disease. Which of the following is the correct management of this patient specifically as related to her mitral valve disease? A. Administer IV antibiotics 30 minutes prior to the procedure B. Administer IV antibiotics for 24 hours after the procedure C. Administer IV antibiotics immediately after the procedure D. Administer oral antibiotics 6 hours after the procedure E. No antibiotics are needed

The correct answer is E. Mitral valve prolapse affects approximately 5% of women of childbearing age. Consequently the issue of mitral valve prolapse and the need for antibiotics is frequently debated. Bacterial endocarditis is a life-threatening infection that can develop in patients who have structural cardiac disease who are exposed to bacteremia. The risk for any given procedure depends on the nature of the procedure itself and on the nature of the cardiac lesion. Periodically the American Heart Association publishes guidelines for the prevention of bacterial endocarditis. According to the American Heart Association guidelines, antibiotic prophylaxis is not necessary for cesarean delivery or normal vaginal delivery. The possible exception to this is patients who have "high risk" cardiac conditions, which include women who have a history of endocarditis or who have prosthetic heart valves, complex cyanotic congenital heart disease, or surgically corrected systemic pulmonary shunts. Mitral valve prolapse, even if associated with mitral regurgitation (demonstrated by Doppler or a murmur), is considered a moderate risk condition and, therefore, antibiotic prophylaxis is not necessary for elective cesarean delivery. This patient therefore does not require antibiotics prior to, during, or after her cesarean delivery.

Patients with chronic heart failure are treated with a variety of agents. To what class of medication does nesiritide (Natrecor) belong? A. Inotrope B. Loop diuretic C. Aldosterone antagonist D. Combined alpha- and beta-blocker E. Vasodilator

The correct answer is E. Nesiritide is an engineered natriuretic peptide that relaxes smooth muscle, resulting in vasodilation. This medication is typically used IV in the setting of acutely decompensated heart failure.

When managing patients for STEMI, it is important to know indications and contraindications for utilizing various medications. Which of the following is a contraindication for using nitroglycerin as part of the management of STEMI? A. Chronic kidney disease B. Acute hepatitis C. Active psoriasis D. Pulmonary edema E. Hypotension

The correct answer is E. Nitroglycerin is a venodilator and dilates the coronary arteries. Nitroglycerin will lower BP, so it cannot be used in the setting of hypotension.

A patient who is being treated for hypertension related to a myocardial infarction that occurred 2 hours ago is medicated with IV nitroprusside (Nipride). Which of the following is the expected action of this drug? A. Constriction of arterioles alone B. Constriction of both arterioles and venules C. Constriction of venules alone D. Dilatation of arterioles alone E. Dilatation of arterioles and venules

The correct answer is E. Nitroprusside is a very useful IV agent that causes dilatation of both arterioles and venules. It has a very rapid onset of action and is typically used in an emergency department or intensive care unit situation. It can be titrated to effect on a drip-to-drip basis. It is used in settings in which the blood pressure needs to be carefully controlled. It is listed on ACLS protocols as one of the treatments (along with the combined alpha- and beta-blocker labetolol) for hypertensive emergency/hypertensive urgency.

A patient is seen by another provider and was told that a screening echocardiogram revealed aortic stenosis. The patient is 65 years old and does not have signs or symptoms consistent with this echocardiogram finding. Which of the following is the treatment of choice at this time? A. Transcatheter aortic valve replacement B. Open aortic valve replacement C. Aortic valve balloon valvuloplasty D. Beta-blockers E. No specific treatment at this time

The correct answer is E. No treatment is needed for aortic stenosis when the patient is asymptomatic. Patients with aortic stenosis are monitored for development of symptoms, but heart surgery is not indicated until the patient becomes symptomatic with a low ejection fraction, or until there is critical reduction of the patient's cardiac output resulting in severe symptoms.

A patient is seen in the office and admits a history of IV drug use. He is febrile and has a newly discovered cardiac murmur. The patient is noted to have painful red lesions on his fingers, toes, and feet. What are these lesions known as? A. Herald patches B. Janeway lesions C. Roth spots D. Target lesions E. Osler nodes

The correct answer is E. Osler nodes are painful lesions on the fingers, toes, and feet in the setting of endocarditis.

A 59-year-old man chooses a health care provider. The health care provider discovers that the patient was recently admitted to the hospital for palpitations and was found to have newly diagnosed atrial fibrillation (AF). He was placed on digoxin (Lanoxin) and verapamil (Calan, Isoptin, Verelan) for rate control and was loaded with warfarin (Coumadin) for anticoagulation. After several trips to the anticoagulation clinic, he decided to stop taking his warfarin, as it was difficult for him to leave work during the day to have the INR checked. He now calls the office complaining of a cold, pale, and painful right leg starting an hour ago. Which of the following is the most likely explanation for his symptoms? A. Cerebrovascular accident (CVA) B. Deep vein thrombosis (DVT) C. Multiple sclerosis (MS) D. Raynaud syndrome E. Thromboembolism

The correct answer is E. Overall, atrial fibrillation confers about a 1% annual risk for a thromboembolic event, which, among other things, may cause stroke, intestinal ischemia, renal infarcts, or a threatened limb. Specific etiologies of AF such as valvular disease are associated with higher rates of embolic events.

A patient developed atrial fibrillation while he was vacationing in the Caribbean the previous week. The patient developed a feeling of heart palpitations and then took his pulse and found that his pulse was irregular. His pulse has remained irregular, and the patient continues to complain of feelings of his heart beating irregularly. Blood pressure in the office is 110/78, and pulse is 108. Which of the following interventions is most appropriate? A. Immediate cardioversion B. Immediate defibrillation C. Beta blockers alone D. Anticoagulate the patient for 3 weeks then cardiovert the patient E. Anticoagulate the patient for 3 weeks, cardiovert, and anticoagulate for 4 weeks following the cardioversion

The correct answer is E. Patients having elective cardioversions for converting atrial fibrillation to normal sinus rhythm when atrial fibrillation occurs more than 48 hours ago should have anticoagulation 3 weeks prior to cardioversion and 4 weeks after cardioversion. The danger of patients having atrial fibrillation is that the lack of atrial kick can allow a clot to form in the atrium. If normal sinus rhythm is restored, the atrial contraction can result in the atrial clot being sent into the ventricles and then into the systemic circulation. For this reason, the patient is anticoagulated before either medical or electrical cardioversion is attempted, and this anticoagulation continues for several weeks in order to ensure that new clot formation does not occur. Following chemical or electrical cardioversion, the patient may have recurrence of atrial fibrillation, which is why the anticoagulation is continued.

Searching for secondary causes of hypertension is prudent to perform in certain clinical settings. When secondary hypertension occurs, correcting the underlying cause for this condition can cure the patient's hypertension. When certain causes of secondary hypertension are identified, there should be an evaluation for other entities that may exist with coarctation of the aorta. Patients with coarctation of the aorta may concurrently have which of the following heart abnormalities? A. Ventricular septal defect B. Atrial septal defect C. Patent ductus arteriosus D. Congenital prolonged QT syndrome E. Bicuspid aortic valve

The correct answer is E. Patients with coarctation of the aorta may have concomitant bicuspid aortic valve. Bicuspid aortic valve does not have the endurance of a tricuspid valve, so these patients may go on to develop aortic stenosis or aortic regurgitation and need to have surgical intervention for this condition.

A patient is seen in the office with light-headedness and palpitations. Her pulse is irregular. EKG performed in the office reveals atrial fibrillation with a rapid ventricular response. All of the following are known precipitants of this condition EXCEPT: A. Alcohol intake B. Thyrotoxicosis C. Mitral stenosis D. Pulmonary embolism E. Pulmonary edema

The correct answer is E. Pulmonary edema is not a risk factor for the development of atrial fibrillation. Pulmonary edema typically occurs as a result of left ventricular failure in the setting of a patient being volume-overloaded. The volume is centralized into the pulmonary system, which leads to increased pulmonary pressures. This condition occurs acutely, so it does not produce a large, floppy atrium, which is a common reason for atrial fibrillation to occur.

Which of the following types of pulses is most commonly associated with aortic stenosis? A. Bisferiens pulse B. Bounding (hyperkinetic pulse) C. Pulsus alternans D. Pulsus paradoxus E. Pulse parvus and tardus

The correct answer is E. Pulse parvus and tardus is most commonly associated with aortic stenosis, causing slow, low volume from impaired cardiac outflow due to the narrowed aortic valve orifice.

For patients in the office who have hypertension, which of the following is the most common secondary cause of hypertension? A. Coarctation of the aorta B. Conn syndrome C. Cushing syndrome D. Pheochromocytoma E. Renal artery stenosis

The correct answer is E. Renal artery stenosis is the most common cause of secondary hypertension. Renal artery stenosis can result from isolated narrowing of the renal artery or due to fibromuscular dysplasia, which is often seen in young females and is identified as a string of pearls on imaging using contrast.

It is essential for health care providers to understand the physiologic and pathophysiologic processes that occur when the right ventricle is damaged as a result of acute myocardial infarction. Which of the following signs or symptoms would typically be absent in the setting of right ventricular failure secondary to STEMI? A. Hepatic engorgement B. Increased jugular distention C. Elevation of right heart pressures D. Signs of inferior or posterior wall STEMI on EKG E. Rales on auscultation of the lungs

The correct answer is E. Right ventricular infarct will not have rales or elevated pulmonary wedge pressures because those findings are downstream, and right ventricular failure causes upstream increased pressures.

For patients who have chronic heart failure with preserved ejection fraction, which of the following heart sounds is most likely to be heard? A. Murmur of aortic stenosis B. Murmur of aortic regurgitation C. Holosystolic murmur of VSD D. Widely split S2 E. S4 gallop

The correct answer is E. S4 gallop is heard in the setting of chronic heart failure with preserved ejection fraction due to left atrial contraction against a stiff left ventricle.

A patient is seen in the office with consistently elevated blood pressures. In order to fully evaluate this patient, a review of her comorbidities and medications is undertaken. The patient is noted to have hypothyroidism, hyperparathyroidism, obstructive sleep apnea, and Cushing disease, and she has been found to have a prolactinoma. Which of the following conditions is not typically a secondary cause of hypertension? A. Hypothyroidism B. Hyperparathyroidism C. Obstructive sleep apnea D. Cushing disease E. Prolactinoma

The correct answer is E. Secondary causes of hypertension include thyroid and parathyroid disease, sleep apnea, and Cushing disease. Prolactinomas lead to infertility in both males and females with this condition, but this is not a cause of secondary hypertension.

A patient is seen with highly abnormal laboratory values including a serum triglyceride level elevated at 3,000 mg/dL. Which of the following is the primary complication of this hypertriglyceridemia? A. Renal failure B. Induces coronary artery spasm C. Liver failure D. Diabetes E. Pancreatitis

The correct answer is E. Severe hypertriglyceridemia can result in pancreatitis and patients should be monitored for this complication.

Which of the following cardiac conditions would be most likely lead to the formation of pulsus alternans? A. Aortic regurgitation B. Aortic stenosis C. Mitral regurgitation D. Patent ductus arteriosus E. Severe left ventricular failure

The correct answer is E. Severe left ventricular failure leads to pulsus alternans with the heart failing as a pump. With this condition, the heart attempts to eject blood from the left ventricle and is only able to forcefully contract on an every other beat basis.

Patients who undergo valve replacement surgery are at risk for infection of the newly placed valve. This infection can occur at various times in the postoperative period. Which of the following pathogens is most likely to cause early-onset endocarditis in prosthetic valves (symptoms appearing within 60 days of placement of the prosthetic valve)? A. Staphylococcus aureus B. Streptococcus viridans C. Enterococcus species D. Streptococcus bovis E. Staphylococcus epidermidis

The correct answer is E. Staphylococcus epidermidis is most likely to cause early onset endocarditis in prosthetic valves, as this organism can be introduced at the time of surgery.

A man complains to his health care provider that he feels lightheaded and has even fainted during defecation. This is most probably an example of syncope due to which of the following mechanisms? A. Anoxia B. Hyperventilation C. Hypovolemia D. Sinus node disease E. Vagal stimulation

The correct answer is E. Syncope has a broad differential diagnosis, because fainting can be produced by a wide variety of mechanisms. All of the mechanisms listed in the answers can produce syncope, but only vagal stimulation is caused by the Valsalva maneuver as the patient strained to defecate. This mechanism can also produce fainting during weightlifting and with the use of wind instruments. Vagal maneuvers result in the slowing or stoppage of cardiac conduction through the AV node and this can be utilized to treat a patient who has symptoms associated with supraventricular tachycardia.

A 12-year-old African American boy is brought to the office for a well child examination. He has been in good health and only complains of an occasional headache. He has been doing reasonably well in school but has some social problems that his mother attributes to his physical appearance. She has tried persuading her son to eat healthier, but he seems to enjoy eating fast food much more than home cooked meals. His past medical history is unremarkable, and the family history is significant for adult-onset hypertension in his father's family. He takes no medication. On physical examination, the patient is in no acute distress. He is 145 centimeters tall and weighs 92 kilograms. His vital signs are within normal limits, but his blood pressure is 145/90 mm Hg. The health care provider checks that the cuff size is appropriate and remeasures his blood pressure in all four extremities, only to confirm that it is above the ninety-fifth percentile for his age. After discussing the finding with his mother, he is scheduled for several more blood pressure evaluations over the following 6 weeks. All readings yield results mildly above the ninety-fifth percentile for his age. Which of the following recommendations is most appropriate at this time? A. A low salt and low potassium diet should be started B. No intervention is necessary at this time C. The child should be enrolled in a sports program that involves strenuous physical activity D. The child should immediately be started on a diuretic and angiotensin-converting enzyme inhibitor E. The child should initiate a weight reduction diet with limited salt intake and regular exercise

The correct answer is E. Systemic hypertension is defined as blood pressure above the ninety-fifth percentile for age on repeated measurements over a 6-week period. Primary (essential) hypertension usually has no known underlying cause. Predisposing factors include heredity, salt intake, stress, and obesity. It is more commonly seen in adolescents and adults. Secondary hypertension is caused by an associated disease. It is more common in infants and younger children. The most common cause of secondary hypertension in children is renal disease (75-80%). A prior urinary tract infection is seen in 25 to 50% of cases, often related to an obstructive lesion of the urinary tract. In newborns, a history of umbilical artery catheterization may be elicited, with resultant thrombosis of the renal artery. Hypertension usually presents with no symptoms, especially in adolescents who have essential hypertension, and it is diagnosed on routine examination. Headaches, dizziness, vision changes, and seizures may be present. Blood pressure should be measured over several visits and compared with normal values for age. It should be measured in all extremities to rule out coarctation of the aorta. All children who have secondary hypertension should have a renal evaluation, including culture, ultrasound, renin levels, blood urea nitrogen, and creatinine. Echocardiography should be used to assess ventricular function and size. Therapy of hypertension in children is the same as in adults: diet, exercise, and medication. Pharmacologic management includes angiotensin-converting enzyme inhibitors, calcium-channel blockers, and diuretics.

Patients who have various heart valve abnormalities can have physical manifestations of these disorders. The de Musset, Mueller, and Duroziez signs are all associated with which of the following cardiac valvular abnormalities? A. Mitral stenosis B. Aortic stenosis C. Pulmonic stenosis D. Tricuspid stenosis E. Aortic regurgitation

The correct answer is E. The de Musset sign is head bobbing that occurs with each heartbeat. Duroziez sign consists of hearing both a systolic and diastolic murmur when the femoral artery is partially compressed. Mueller sign is the visualization of pulsations in the uvula due to the large cardiac outflow that is being sent into the systemic circulation. Aortic regurgitation consists of a large volume of blood being forcefully contracted out of the heart with a rapid de-escalation of blood in the systemic circulation because of blood reentering the heart rather than being maintained in the systemic circulation.

Which of the following is the first treatment typically employed for hypertrophic cardiomyopathy (HCM)? A. Clipping of the redundant mitral valve B. Septum injection with alcohol C. Digoxin (Lanoxin) D. ACE inhibitor E. Beta blocker

The correct answer is E. The first-line treatment for HCM are beta blockers. Beta blockers slow the heart rate, which allows more time for ventricular filling and increased heart volume in the setting of obstructive cardiomyopathy. These are the medications of choice for patients with hypertrophic obstructive cardiomyopathy by allowing increased blood in the left ventricle along with causing less inotropic contraction of the left ventricle.

A 35-year-old woman presents to her health care provider with paresthesias of the left shoulder and arm. She previously had an X-ray that showed that she has a cervical rib on the left. Which of the following diagnoses explains the cause of her symptoms? A. Horner syndrome B. Osteopetrosis C. Pancoast tumor D. Shoulder dislocation E. Thoracic outlet obstruction

The correct answer is E. The patient has a left cervical rib. This anatomic variant, which is typically bilateral but can occur on only one side, is caused by formation of an extra rib at the C7 level. Cervical ribs, while often asymptomatic, can cause thoracic outlet obstruction. This results in pain caused by distortion of blood vessels; pain or paresthesias related to brachial plexus impingement (notably sensory disturbances in the distribution of the ulnar nerve), and palpable abnormalities in the greater supraclavicular fossa.

A 68-year-old man presents with complaints of chronic fatigue, exertional and nocturnal dyspnea, orthopnea, and a chronic nonproductive cough. On examination, respiratory wheezing and rhonchi are noted. Cardiac examination reveals a diminished first heart sound and an S3 gallop. The patient indicates that he was recently treated for hypertension and vasospastic angina. Which of the following medications is typically given for this type of patient? A. Amlodipine (Norvasc) B. Captopril (Capoten) C. Furosemide (Lasix) D. Hydralazine (Apresoline) E. Verapamil (Isoptin, Calan)

The correct answer is E. The patient is presenting with classic signs and symptoms of congestive heart failure: chronic fatigue, exertional and nocturnal dyspnea, orthopnea, a chronic nonproductive cough, respiratory wheeze and rhonchi, as well as a diminished first heart sound and an S3 gallop. Verapamil is a non-dihydropyridine calcium channel blocker used to treat both hypertension and vasospastic angina. It has, however, a strong negative inotropic effect on the heart that can cause signs and symptoms of heart failure. Furthermore, some clinical studies have shown that congestive heart failure can develop in a small percentage of individuals taking verapamil.

A patient with long-standing kidney failure develops shortness of breath, a pericardial friction rub, distant heart sounds, low voltage on EKG, and jugular venous distention. He has increased chest pain when he takes a deep breath. Which of the following is the preferred treatment for him? A. Creation of pericardial window B. Pericardiocentesis to remove fluid C. Loop diuretics D. Aspirin E. Dialysis

The correct answer is E. The treatment of choice for uremic pericardial effusion is dialysis more than pericardiocentesis. Dialysis will resolve the azotemia, which will cause resolution of the inflammatory cells in the pericardium, making dialysis the treatment of choice for uremic pericarditis.

A 25-year-old man is shot with a .22-caliber revolver. The entrance wound is in the anteromedial aspect of the upper thigh, while the exit wound is approximately 3 inches lower, in the posterolateral aspect of the thigh. He has a large, expanding hematoma in the upper inner thigh. There are no palpable pulses in the foot. The bone is intact by physical examination and x-ray film. Which of the following is the most appropriate next step in management? A. Arteriogram B. CT angiography C. Doppler studies D. Embolectomy E. Surgical exploration F. Venogram

The correct answer is E. There is no question that this patient's femoral vessels are injured, and the expanding hematoma plus absent pulses indicate that the femoral artery is involved (the vein may or may not be injured). Surgical exploration starts with proximal and distal control. Once the hematoma is safely entered, the extent of the injuries can be ascertained and the proper repair performed.

A 4-week-old boy is brought to the emergency department with tachycardia, tachypnea, and poor weight gain. His arterial blood gas shows a pH of 7.34 (normal 7.35-7.45 in this age population), PaCO2 of 41 mm Hg (normal 27-44 mm Hg in neonatal period), and PaO2 of 74 mm Hg (normal 83-100 mm Hg in the neonatal period). Chest radiograph shows cardiomegaly. Echocardiography reveals a structurally normal heart, left ventricular dilatation, a left ventricular ejection fraction of 20%, and mild mitral and tricuspid regurgitation. Intravenous administration of which of the following medications is the best initial step in management? A. Lisinopril (Zestril) B. Corticosteroid C. Digoxin (Lanoxin) D. Epinephrine E. Furosemide (Lasix)

The correct answer is E. This infant has acute congestive heart failure (CHF). Heart failure may be defined as inadequate contractile heart function for the specific hemodynamic needs. Clinical manifestations include respiratory distress, tachycardia and/or hyperdynamic precordium, and cardiac enlargement evidenced by echocardiogram. This patient is developing acute symptom manifestation as a result of the patient not being able to adequately meet the metabolic demands of his body. The differential diagnosis of CHF includes left-to-right shunt (ASD, VSD, PDA, atrioventricular canal, or AV fistula), left-sided obstruction leading to myocardial dysfunction (severe coarctation or AS), or intrinsic myocardial dysfunction (myocarditis, cardiomyopathy, or infarct caused by anomalous coronary artery). In acute CHF, treatment should begin immediately. IV furosemide is the drug of choice because its onset is very rapid. It can provide quick symptomatic relief and improve respiratory distress.

A 50-year-old surgical nurse consults a health care provider because of a rash above both her ankles. She says that her ankles have been swollen for a few months but she only recently noticed the rash. Physical examination demonstrates marked ankle edema with erythema, mild scaling, and brown discoloration of the overlying skin of the distal lower legs. Varicose veins are also noted. Which of the following is the most likely diagnosis? A. Atopic dermatitis B. Cellulitis C. Lichen simplex chronicus D. Nummular dermatitis E. Stasis dermatitis

The correct answer is E. This is stasis dermatitis, which is a persistent inflammation of the skin of the lower legs. The condition is often related to varicose veins, although it has been postulated that the true cause may instead be perivascular fibrin deposition and abnormal small vessel vasoconstrictive reflexes. When there is venous pooling of the blood, hemosiderin deposition will occur which leads to the brawny discoloration. This presentation is typical. Most patients are relatively asymptomatic and may not seek medical attention until the edema becomes severe or the lesions become complicated by secondary bacterial infection or ulceration. It is important to increase the venous return to the heart by elevating the ankles while resting and using properly fitted compression support hose. Unna boot application can be helpful if ulcer formation occurs. Purulent lesions can be treated with hydrocolloid dressings. Ulcers are treated with compresses and bland dressings, such as zinc oxide paste.

A 68-year-old man is brought to the emergency department following a high-speed automobile accident. He is alert and complains of chest pain and mild back pain. His blood pressure is 80/60 mm Hg. Chest radiograph shows a widened mediastinum, tracheal deviation, bronchial displacement, and loss of the aortic knob. Which of the following is the most likely diagnosis? A. Cardiac tamponade B. Myocardial contusion C. Pulmonary contusion D. Tension pneumothorax E. Traumatic aortic rupture

The correct answer is E. This patient has a traumatic aortic rupture, which is the most common cause of immediate death following a motor vehicle accident. Deceleration causes twisting of the aorta and may lead to rupture. Up to 90% of individuals die at the scene, however a high index of suspicion and early intervention may save the remaining few. The symptoms include chest pain, back pain, and hypotension. Angiography is the gold standard and immediate surgery is mandatory. Radiographic findings include widening of the mediastinum, alteration of the aortic knob, pleural cap, tracheal deviation, bronchial displacement, and esophageal deviation.

A 10-year-old girl is brought to the office because of fever and chills for 3 days. She had been complaining of a headache and feeling tired for 10 days before she developed fever. The parents had tried giving her acetaminophen, but the child's condition was not improving. Her past medical history is significant for frequent streptococcal throat infections over the past 2 years and a new onset heart murmur detected at her last well child visit 2 months earlier. She is on no medication currently, except for acetaminophen. Otherwise, she has been wearing dental braces for the past year and had a primary tooth extracted 2 weeks earlier in the attempt of liberating space for permanent teeth. On physical examination, the patient is diaphoretic, in moderate distress with a temperature of 39.0°C (102.2°F), blood pressure of 90/60 mm Hg, pulse of 110/min, and respirations of 24/min. On her fingernails, you note several splinter hemorrhages. Auscultation confirms a grade 2/6 high-pitched, blowing, systolic ejection murmur, best heard at the apex that radiates to the left axilla. A chest radiograph is unremarkable, but the electrocardiogram shows signs of left ventricular strain. An echocardiogram reveals vegetations on the mitral valve. Which of the following microorganisms is the most likely cause of this patient's current condition? A. Candida species B. Pseudomonas aeruginosa C. Serratia marcescens D. Staphylococcus aureus E. Streptococcus viridans

The correct answer is E. This patient has developed acute endocarditis most likely caused by Streptococcus viridans that entered the bloodstream during the recent dental procedure. The predisposing factor for her current illness is rheumatic heart disease, a sequela of prior Group A β-hemolytic streptococcal pharyngitis, which was complicated by involvement of the mitral valve.

A 61-year-old woman comes to the health care provider for her first physical examination in more than 10 years. She reports that she has been in excellent health, does not smoke or drink, and runs 3 miles daily. She is a retired accountant and has 3 healthy grown children. She has been taking 81 mg of aspirin daily after reading about its cardioprotective effects in the newspaper. On physical examination she appears well. Blood pressure is 122/76 mm Hg, pulse 70/min, and respirations 14/min. She is afebrile. Head and neck examination is normal. There is no jugulovenous distention. Lungs are clear. On cardiac examination she has a regular S1 and S2, and II/VI crescendo blowing diastolic murmur is heard at the aortic area. Abdominal examination is normal. Rectal examination shows no masses and brown, guaiac-negative stool. Which of the following most likely explains the cardiac findings on physical examination? A. Prior Bacteroides infection B. Prior Escherichia coli infection C. Prior gonococcal infection D. Prior Haemophilus ducreyi infection E. Prior streptococcal infection

The correct answer is E. This patient has findings of asymptomatic aortic insufficiency on physical examination. This lesion may result from a number of causes, several of which are infectious in etiology. Aortic insufficiency may result as a sequela of rheumatic heart disease, which occurs as an immunologic response to a streptococcal infection. Acute rheumatic fever is typically characterized by cardiac involvement that may cause pericarditis, myocarditis, or endocarditis. Often the initial cardiac manifestations are asymptomatic and become apparent only years later with the development of cardiac valvular disease.

A 62-year-old man comes to the health care provider with symptoms of worsening congestive heart failure. He has a history of rheumatic heart disease as a child. Over the past 3 years he has had progressive symptoms of dyspnea on exertion, paroxysmal nocturnal dyspnea, and orthopnea. He has been maintained on digoxin, furosemide, and enalapril for symptoms of his congestive heart failure. Cardiac examination reveals a loud blowing decrescendo diastolic murmur. He has bounding peripheral pulses. Which of the following additional findings would most likely be found on physical examination? A. Bradycardia B. Isolated systolic hypertension C. Pulsus paradoxus D. Tachycardia E. Wide pulse pressure

The correct answer is E. This patient has the characteristic physical findings of an aortic insufficiency, including the loud blowing decrescendo murmur and the bounding peripheral pulses. These patients typically will have a wide pulse pressure (i.e., elevated systolic blood pressure) related to a large left ventricular volume being ejected (until left ventricular failure supervenes). The low diastolic pressure is a result of the rapid run-off from the aorta caused by the regurgitant flow across the aortic valve into the left ventricle as well as the forward flow to the aorta.

In addition to utilizing blood cultures as part of the assessment for endocarditis, which of the following diagnostic studies is considered to be the best to identify bacterial endocarditis? A. CT of the chest B. MRI C. PET scan D. Transthoracic echocardiogram E. Transesophageal echocardiogram

The correct answer is E. Transesophageal echocardiogram (TEE) is the diagnostic test of choice for endocarditis, as this test is best at identifying all of the heart valves. It is essential that a high sensitivity test is utilized because this is not a diagnosis that should be missed.

A patient is seen in the office with complaints related to pain in his hands and feet. The patient has a history of smoking cigarettes, and he also has hypertension and coronary artery disease. CT angiogram is performed and reveals smooth tapering distal vessels but no associated signs of atherosclerosis. Which of the following treatment strategies will be most effective for this patient? A. Nitrates B. Beta-blockers C. Alpha-1 antagonists D. ACE inhibitors E. Smoking cessation

The correct answer is E. Treat thromboangiitis obliterans by smoking cessation. Buerger disease is an inflammatory reaction caused by smoking cigarettes typically occurs in young men and affects both upper and lower extremities. Patients with this disorder are extremely sensitive to the effects of nicotine, with their blood vessels narrowing as a result of nicotine intake.

A patient is seen in the office with complaints of a new onset of a racing heart. The patient is placed on the monitor in the office, and atrial fibrillation with a rapid ventricular response is noted. The patient is complaining of chest heaviness and becomes unstable. Blood pressure is not audible with a stethoscope. Which of the following is the treatment of choice? A. Beta blockers IV to control the rate B. Calcium channel blockers IV to control the rate C. Adenosine IV D. Ibutalide (Corvert) IV E. Immediate cardioversion

The correct answer is E. Treat unstable A fib with immediate cardioversion. Unstable atrial fibrillation is defined as a patient having chest pain symptoms in the setting of hypotension or poor organ perfusion. Hemodynamic instability includes patients who are hypotensive, patients who have symptomatic heart failure, or those with ongoing myocardial ischemia.

Which of the following diuretic agents is most likely to cause hyperkalemia? A. Acetazolamide (Diamox) B. Furosemide (Lasix) C. Hydrochlorothiazide D. Metolazone (Zaroxolyn) E. Triamterene (Dyrenium)

The correct answer is E. Triamterene is grouped in the category of potassium-sparing diuretics. Along with spironolactone (Aldactone) and amiloride (Midamor), these diuretic agents may cause excess renal retention of potassium. In this manner, use of these drugs may increase potassium levels and cause hyperkalemia.

You are assessing a patient in the ICU with acute hypovolemic shock. The patient's blood pressure is not sufficient to maintain tissue perfusion to the systemic circulation. Which of the following is the initial therapy given? A. Whole blood products B. Packed red blood cells C. Dopamine D. Norepinephrine E. Normal saline

The correct answer is E. Volume expansion with normal saline or lactated Ringer's is the initial therapy for hypovolemic shock prior to using vasopressors. This is so that there is volume in the blood vessels prior to vasopressors contracting them.

A 19-year-old college student with a history of panic disorder and anxiety comes to the emergency department complaining of shortness of breath and anxiousness before finals. She denies any chest pain, palpitations, diaphoresis, or light-headedness. These episodes are similar to "panic attacks" that she had in the past. She has recently stopped all of her outpatient medications because she believed that she no longer needed them. Her temperature is 37.0°C (98.6°F), blood pressure is 120/70 mm Hg, pulse is 113/min, and respirations are 18/min. Her oxygen saturation is 99% on room air. She appears anxious but in no acute distress. Her heart is tachycardic without any murmurs. Her lungs are clear. The remainder of her examination is unremarkable. She is treated appropriately and her symptoms resolve. Just as she is about to be discharged with a diagnosis of anxiety and panic disorder, a nurse points out the following rhythm strip: 2nd degree AV block - Wenckebach Which of the following is the most appropriate management? A. Administer amiodarone B. Anticoagulation C. Cardioversion D. Electrophysiologic study E. Pacemaker placement F. Reassurance

The correct answer is F. This patient has a second-degree atrioventricular (AV) block of the Mobitz type I or Wenckebach type. In this condition, there is a progressive prolongation of the PR interval until there is eventually a blocked beat. This rhythm can be seen in normal patients or can be seen in patients on drugs such as digoxin, beta-blockers, or calcium channel antagonists. Rarely this rhythm can be secondary to ischemia, infarction, or inflammation. The patient should be reassured that the prognosis is good, and treatment is not routinely needed.


संबंधित स्टडी सेट्स

World Geography Chapter 7 Review

View Set

A Raisin In the Sun - Final test

View Set

PS103, PS104, and PS102 - Reviewer

View Set

Chapter 8 Interest Rates and Bond Valuation

View Set